You are on page 1of 150

PASS PRO - Copyright 2004 Inmarkets Ltd - Licensed to Raymond chen

Print PASS PRO Questions 1.

/150 1
Close

If a cash flow of $10,000 in two years' time has a PV of $8,455, the annual percentage rate, assuming continuous compounding is CLOSEST to: A. 8.13%. B. 8.39%. C. 8.75%. D. 8.95%. 2. The present value of $700 received in 9 month's time,, using a continuous discount rate of 9%, is CLOSEST to: A. $653. B. $654. C. $655. D. $656. 3. If a cash flow of $100 in 2 years' time has a PV of $75, the annual percentage rate, assuming continuous compounding is CLOSEST to: A. 11.1%. B. 12.2%. C. 13.5%. D. 14.4%. 4. If a cash flow of $100 in 2 years' time has a PV of $75, the annual percentage rate, assuming continuous compounding is CLOSEST to: A. 11.1%. B. 12.2%. C. 13.5%. D. 14.4%. 5. The price of a stock has risen from $60 to $70 over the past 4 years. If the stock did not pay any dividends during this period, its continuously compounded return is CLOSEST to: A. 2.18%.

http://64.106.152.240/inmarkets/passpro/index.cfm?event=questions....

2004-08-02

PASS PRO - Copyright 2004 Inmarkets Ltd - Licensed to Raymond chen

/150 2

B. 2.25%. C. 3.35%. D. 3.85%. 6. The price of a stock has risen from $13 to $35 over the past five years. If the stock did not pay any dividends during this period, its continuously compounded return is CLOSEST to: A. 20%. B. 22%. C. 24%. D. 26%. 7. The present value of $800 received in 12 month's time,, using a continuous discount rate of 20%, is CLOSEST to: A. $621. B. $638. C. $655. D. $667. 8. The price of a stock has risen from $60 to $70 over the past 4 years. If the stock did not pay any dividends during this period, its continuously compounded return is CLOSEST to: A. 2.18%. B. 2.25%. C. 3.35%. D. 3.85%. 9. The price of a stock has risen from $13 to $35 over the past five years. If the stock did not pay any dividends during this period, its continuously compounded return is CLOSEST to: A. 20%. B. 22%. C. 24%. D. 26%. 10. The price of a stock has risen from $30 to $45 over the past 5 years. If the stock did not pay any dividends during this period, its continuously compounded return is CLOSEST to:

http://64.106.152.240/inmarkets/passpro/index.cfm?event=questions....

2004-08-02

PASS PRO - Copyright 2004 Inmarkets Ltd - Licensed to Raymond chen


A. 7.2%. B. 8.1%. C. 8.3%. D. 8.6%. 11.

/150 3

The present value of $4,500 received in 6 month's time, using a continuous discount rate of 9%, is CLOSEST to: A. $3,290 B. $4,302 C. $4,306 D. $4,310 12. If a cash flow of $700 in 3 years' time has a PV of $400, the annual percentage rate, assuming continuous compounding is CLOSEST to: A. 8.52%. B. 12.16%. C. 18.65%. D. 25.44%. 13. The present value of $800 received in 12 month's time,, using a continuous discount rate of 20%, is CLOSEST to: A. $621. B. $638. C. $655. D. $667. 14. The price of a stock has risen from $15 to $40 over the past 2 years. If the stock did not pay any dividends during this period, its continuously compounded return is CLOSEST to: A. 9.15% B. 14.94% C. 29.24% D. 49.04% 15. If a cash flow of $800 in 6 years' time has a PV of $500, the annual percentage rate, assuming

http://64.106.152.240/inmarkets/passpro/index.cfm?event=questions....

2004-08-02

PASS PRO - Copyright 2004 Inmarkets Ltd - Licensed to Raymond chen

/150 4

continuous compounding is CLOSEST to: A. 4.48%. B. 5.23%. C. 7.83%. D. 9.12%. 16. The present value of $700 received in 9 month's time,, using a continuous discount rate of 9%, is CLOSEST to: A. $653. B. $654. C. $655. D. $656. 17. Given that a stock price rises on two days out of five and falls on three days out of five, what is the probability that it will rise on exactly seven out of the next eight days? A. 0.8%. B. 4.5%. C. 9.0%. D. 12.0%. 18. A portfolio has a mean value of $75 million and a daily standard deviation of $4.27 million. Assuming that the portfolio values are normally distributed, the probability of the portfolio value falling below $40 million within the next seven days is CLOSEST to: A. 0.10%. B. 1.00%. C. 5.00%. D. 15.87%. 19. A standard normal distribution has: A. no tails. B. fat tails. C. infinite tails. D. asymmetric tails. 20.

http://64.106.152.240/inmarkets/passpro/index.cfm?event=questions....

2004-08-02

PASS PRO - Copyright 2004 Inmarkets Ltd - Licensed to Raymond chen

/150 5

The covariance between the return from two securities is 4 and the correlation between them is 0.5. If the variance of the first return is 16, the variance of the second return will be CLOSEST to: A. 0.25. B. 0.50. C. 2.00. D. 4.00. 21. Which of the following are characteristics of a normal distribution? I. Skewness equal to zero. II. Mean less than median. III. Kurtosis greater than zero. IV. Continuous and unbounded. A. I and IV. B. II and III. C. I, II and III. D. I, III and IV. 22. What is the quantile corresponding to the 95% confidence interval? A. -3.715. B. -2.326. C. -1.645. D. -1.282. 23. Consider a portfolio whose expected return is normally distributed with a mean of 20 percent and a standard deviation of 10 percent. The probability that the return will lie between 0 percent and 10 percent is CLOSEST to: A. 2%. B. 6%. C. 14%. D. 19%. 24. Assume a variable with normal distribution. The mean is 100. The variance is 100. The probability of observing an outcome of 77 and below is approximately: A. 1.00%. B. 10.35%.

http://64.106.152.240/inmarkets/passpro/index.cfm?event=questions....

2004-08-02

PASS PRO - Copyright 2004 Inmarkets Ltd - Licensed to Raymond chen

/150 6

C. 12.35%. D. 50.35%. 25. An analyst determines that the S&P500 falls every 3 out of 8 days but without any serial dependency, i.e. probability of a fall on a certain date is not affected by its movement on the previous days. Given that there are 285 trading days in a year, the standard deviation in the number of days that the S&P500 will fall is CLOSEST to: A. 4 B. 6 C. 8 D. 14 26. A portfolio has a mean value of $90 million and a daily standard deviation of $9 million. Assuming that the portfolio values are normally distributed, the lowest value that the portfolio will fall to over the next ten days and within 99.9% probability is: A. $1.8 million. B. $23.8 million. C. $43.2 million. D. $61.5 million. 27. The covariance between the return from two securities is 4.2 and the correlation between them is 0.6. If the variance of the first return is 5, the variance of the second return will be CLOSEST to: A. 1.40. B. 1.96. C. 3.13. D. 9.80. 28. What is the quantile corresponding to the 99% confidence interval? A. -3.715. B. -2.326. C. -1.645. D. -1.282. 29. Which of the following are characteristic of a normal distribution?

http://64.106.152.240/inmarkets/passpro/index.cfm?event=questions....

2004-08-02

PASS PRO - Copyright 2004 Inmarkets Ltd - Licensed to Raymond chen

/150 7

I. It is bell shaped. II. It is a continuous distribution. III. It is symmetrical about the mean. IV. It peaks at the mean expected value. A. I and II. B. I and IV. C. I, II and III. D. I, II, III and IV. 30. The mean age of the 80 employees in a company is 35 and the standard deviation is 15. Assuming that the ages are normally distributed and using 95 percent confidence, we can say that the employees within the firm fall between: A. 20.0 and 50.0 years. B. 31.7 and 38.3 years. C. 33.8 and 36.2 years. D. 34.6 and 35.4 years. 31. Which of the following is the most appropriate for modeling stock prices? A. Normal distribution. B. Poisson distribution. C. Random distribution. D. Lognormal distribution. 32. Which of the following are characteristics of a normal distribution? I. Mean less than median. II. Skewness equal to three. III. Kurtosis greater than zero. IV. Continuous and unbounded. A. I and II. B. III and IV. C. I, III and IV. D. II, III and IV. 33. Given that a stock price rises on two days out of five and falls on three days out of five, what is the probability that it will rise on exactly seven out of the next eight days? A. 0.8%.

http://64.106.152.240/inmarkets/passpro/index.cfm?event=questions....

2004-08-02

PASS PRO - Copyright 2004 Inmarkets Ltd - Licensed to Raymond chen

/150 8

B. 4.5%. C. 9.0%. D. 12.0%. 34. The covariance between the return from two securities is 4 and the correlation between them is 0.5. If the variance of the first return is 16, the variance of the second return will be CLOSEST to: A. 0.25. B. 0.50. C. 2.00. D. 4.00. 35. The covariance between the return from two securities is 15 and the correlation between them is 0.5. If the variance of the first return is 25, the variance of the second return will be CLOSEST to: A. 1.20. B. 1.44. C. 6 D. 36 36. An analyst determines that in the current market environment the S&P500 goes up five out of every seven days but without any serial dependency, i.e. probability of going up on a certain date is not affected by its movement on the previous days. Given that 280 trading days in a year the standard deviation in the number of days that the S&P500 will go up is CLOSEST to: A. 7.56. B. 11.95. C. 57.18. D. 142.74. 37. A standard normal distribution has: A. no tails. B. fat tails. C. infinite tails. D. asymmetric tails. 38.

http://64.106.152.240/inmarkets/passpro/index.cfm?event=questions....

2004-08-02

PASS PRO - Copyright 2004 Inmarkets Ltd - Licensed to Raymond chen

/150 9

A portfolio has a mean value of $140 million and a daily standard deviation of $1.5 million. Assuming that the portfolio values are normally distributed, the lowest value that the portfolio will fall to over the next 250 days and within 95% probability is: A. $66.5 million. B. $84.9 million. C. $101 million. D. $116.3 million. 39. Which of the following is the most appropriate for modeling stock prices? A. Normal distribution. B. Poisson distribution. C. Random distribution. D. Lognormal distribution. 40. A portfolio has a mean value of $100 million and a daily standard deviation of $19 million. Assuming that the portfolio values are normally distributed, the lowest value that the portfolio will fall to over the next five days and within 95% probability is: A. -$31.7 million. B. $1.2 million. C. $30.1 million. D. $57.5 million. 41. The covariance between the return from two securities is 22 and the correlation between them is 0.8. If the variance of the first return is 25, the variance of the second return will be CLOSEST to: A. 1.10. B. 1.21. C. 5.50. D. 30.25. 42. Pat Fineman is due to receive $6,000 one year from now, which he can invest at an interest rate of 4 percent for a further four years. The value of this payment at the end of this period will be CLOSEST to: A. $6,960 B. $7,019 C. $7,200

http://64.106.152.240/inmarkets/passpro/index.cfm?event=questions....

2004-08-02

PASS PRO - Copyright 2004 Inmarkets Ltd - Licensed to Raymond chen


D. $7,300 43.

0/150 1

Assuming continuous compounding and an annual percentage rate of 8%, the present value of $8,400 received in six months' time is CLOSEST to: A. $6,203. B. $8,071. C. $8,077. D. $8,083. 44. If a cash flow of $21,000 in two years' time has a PV of $16,000, the annual percentage rate, assuming continuous compounding is CLOSEST to: A. 13.60%. B. 13.80%. C. 14.24%. D. 14.56%. 45. An investor is being promised an annuity of $4,000 starting in 8 years from now and continuing for 10 years. Assuming a constant interest rate of 5.25 percent per annum, the value of this annuity today is CLOSEST to: A. $21,328. B. $26,563. C. $27,958. D. $30,515. 46. What is the standard deviation of the following set of numbers? {2, 4, 6, 8, 10}. A. 2. B. 2.8. C. 3.2. D. 3.6. 47. A bank is offering loans with monthly interest payments based on a stated annual interest rate of 9 percent. The effective annual rate of these loans is CLOSEST to: A. 9.20%.

http://64.106.152.240/inmarkets/passpro/index.cfm?event=questions....

2004-08-02

PASS PRO - Copyright 2004 Inmarkets Ltd - Licensed to Raymond chen


B. 9.31%. C. 9.34%. D. 9.38%. 48.

1/150 1

If a cash flow of $10,000 in five years' time has a PV of $7,500, the annual percentage rate, assuming continuous compounding, is CLOSEST to: A. 5.65%. B. 5.75%. C. 5.85%. D. 5.92%. 49. If all the cash flows in an investment are positive what is its IRR? A. Positive. B. Zero. C. Negative. D. Cannot be determined. 50. Peter Roche is due to receive $4,000 one year from now, which he can invest at an interest rate of 7 percent for a further ten years. The value of this payment at the end of this period will be CLOSEST to: A. $6,800. B. $7,080. C. $7,869. D. $8,419. 51. An investor is being promised an annuity of $900 starting in two years from now and continuing for 25 years. Assuming a constant interest rate of 8 percent per annum, the value of this annuity today is CLOSEST to: A. $8,895. B. $9,607. C. $19,290. D. $20,833. 52. An investor is being promised an annuity of $1,500 starting in 15 years from now and continuing for ten years. Assuming a constant interest rate of 8 percent per annum, the value

http://64.106.152.240/inmarkets/passpro/index.cfm?event=questions....

2004-08-02

PASS PRO - Copyright 2004 Inmarkets Ltd - Licensed to Raymond chen

2/150 1

of this annuity today is CLOSEST to: A. $3,427. B. $4,729. C. $5,107. D. $10,065. 53. Sam Walsh is due to receive $7,000 one year from now, which he can invest at an interest rate of 6 percent for a further eight years. The value of this payment at the end of this period will be CLOSEST to: A. $10,360. B. $10,780. C. $11,157. D. $11,826. 54. If a cash flow of $8,000 in four year's time has a PV of $7,000, the annual percentage rate, assuming continuous compounding is CLOSEST to: A. 3.24%. B. 3.29%. C. 3.34%. D. 3.39%. 55. What is the standard deviation of the following set of numbers? {2, 4, 6, 8, 10}. A. 2. B. 2.8. C. 3.2. D. 3.6. 56. If a cash flow of $21,000 in two years' time has a PV of $16,000, the annual percentage rate, assuming continuous compounding is CLOSEST to: A. 13.60%. B. 13.80%. C. 14.24%. D. 14.56%.

http://64.106.152.240/inmarkets/passpro/index.cfm?event=questions....

2004-08-02

PASS PRO - Copyright 2004 Inmarkets Ltd - Licensed to Raymond chen

3/150 1

57. An investor is being promised an annuity of $2,000 starting in two years from now and continuing for eight years. Assuming a constant interest rate of 4 percent per annum, the value of this annuity today is CLOSEST to: A. $12,947. B. $13,465. C. $14,793. D. $15,385. 58. A bank is offering loans with monthly interest payments based on a stated annual interest rate of 7.79 percent. The effective annual rate of these loans is CLOSEST to: A. 8.02%. B. 8.05%. C. 8.07%. D. 8.09%. 59. If a cash flow of $8,000 in four year's time has a PV of $7,000, the annual percentage rate, assuming continuous compounding is CLOSEST to: A. 3.24%. B. 3.29%. C. 3.34%. D. 3.39%. 60. Suppose that Gene owns a perpetuity, issued by an insurance company that pays $1,250 at the end of each year. The insurance company now wishes to replace it with a decreasing perpetuity of $1,500 decreasing at 1% p.a. without any change in the payment dates. At what rate of interest (assuming a flat yield curve) would Gene be indifferent between the choices? A. 4%. B. 5%. C. 6%. D. 9%. 61. Sam Walsh is due to receive $7,000 one year from now, which he can invest at an interest rate of 6 percent for a further eight years. The value of this payment at the end of this period will be CLOSEST to: A. $10,360.

http://64.106.152.240/inmarkets/passpro/index.cfm?event=questions....

2004-08-02

PASS PRO - Copyright 2004 Inmarkets Ltd - Licensed to Raymond chen


B. $10,780. C. $11,157. D. $11,826. 62.

4/150 1

Pat Fineman is due to receive $6,000 one year from now, which he can invest at an interest rate of 4 percent for a further four years. The value of this payment at the end of this period will be CLOSEST to: A. $6,960 B. $7,019 C. $7,200 D. $7,300 63. The inflation rate in an economy where the prices are doubling ever eight years is CLOSEST to: A. 6%. B. 9%. C. 13%. D. 25%. 64. What is the approximate price of a zero coupon bond whose par value is 100 and term is two years if its yields is 5%? A. 86. B. 91. C. 96. D. 101. 65. Consider two cash flows, the first of $10,000 receivable after one year and the second one of $11,000 receivable after three years. At what rate of semi-annual interest would you be indifferent between the two? A. 3.8%. B. 4.8%. C. 5.8%. D. 6.0%. 66. The semi-annual yield in the market for one year is 5%. An options trader is pricing 1-year options under Black Scholes. What is the rate of interest he should input in his model?

http://64.106.152.240/inmarkets/passpro/index.cfm?event=questions....

2004-08-02

PASS PRO - Copyright 2004 Inmarkets Ltd - Licensed to Raymond chen


A. 4.8500%. B. 4.9385%. C. 5.0000%. D. 5.0625%. 67. Which of the following statements are TRUE?

5/150 1

I. Correlation coefficient falls between -1 and +1. II. Independent stochastic variables have a correlation coefficient of -1. III. Correlation coefficient measures the non-linear relationship between two variables. IV. Correlation coefficient can be calculated by scaling the covariance between two variables. A. I and IV. B. II and III. C. I, II and III. D. I, III and IV. 68. A trader in your firm is convinced that the stock index in country X is perfectly negatively correlated to the S&P 500. In order to profit from this analysis, he has taken a long position on index X and shorted S&P 500 futures. Which of the following is TRUE? A. This is a riskless trade because the stocks have negative correlation. B. Apart from the currency risk and cash flow risks on margin calls, this is almost a riskless strategy. C. This trading strategy has the same risk as shorting the S&P 500. This is not a hedged position. D. There will be some small residual risk due to the currency conversion. Otherwise, it is a virtually riskless strategy. 69. The covariance between the return from two securities is 5 and the correlation between them is 0.5. If the variance of the first return is 8, the variance of the second return will be CLOSEST to: A. 1.25. B. 1.56. C. 3.54. D. 12.50. 70. What is the mean for the following probability data? Probability Value 60% -2

http://64.106.152.240/inmarkets/passpro/index.cfm?event=questions....

2004-08-02

PASS PRO - Copyright 2004 Inmarkets Ltd - Licensed to Raymond chen

6/150 1

20% 10 20% 20 A. 1.60. B. 2.80. C. 3.60. D. 4.80. 71. Given the following data for a market variable what is the best estimate of its variance? Probability Value 24% -12 40% 4 36% 14 A. 6 B. 10 C. 32 D. 97 72. Which of the following statements about the correlation coefficient are FALSE? I. Correlation coefficient ranges between 0 and +1. II. If two random variables are independent they will have a correlation coefficient of zero. III. Correlation coefficient can be calculated by scaling the covariance between two variables. IV. Correlation coefficient is a measure of the linear relationship between a dependent and an independent variable. A. I and II. B. I and IV. C. II and III. D. III and IV. 73. The covariance between the return from two securities is 5 and the correlation between them is 0.5. If the variance of the first return is 8, the variance of the second return will be CLOSEST to: A. 1.25. B. 1.56. C. 3.54. D. 12.50. 74.

http://64.106.152.240/inmarkets/passpro/index.cfm?event=questions....

2004-08-02

PASS PRO - Copyright 2004 Inmarkets Ltd - Licensed to Raymond chen

7/150 1

Consider a stock whose 5-day volatility has been estimated as 0.89%. Assuming that the returns from the stock are uncorrelated over time, the volatility over 22 days will be CLOSEST to: A. 0.42%. B. 1.87%. C. 3.92%. D. 4.17%. 75. What is the mean for the following probability data? Probability Value 50% -2 25% 8 25% 15 A. 1.58. B. 2.25. C. 4.75. D. 5.38. 76. What is the standard deviation of the following data? Probability Value 60% -2 20% 10 20% 20 A. 5.14. B. 8.91. C. 26.45. D. 79.36. 77. What is the mean for the following probability data? Probability Value 30% -10 40% 5 30% 25 A. 1.15. B. 2.17. C. 3.34. D. 6.50.

http://64.106.152.240/inmarkets/passpro/index.cfm?event=questions....

2004-08-02

PASS PRO - Copyright 2004 Inmarkets Ltd - Licensed to Raymond chen

8/150 1

78. Consider a stock whose 10-day volatility has been estimated as 1.4%. Assuming that the returns from the stock are uncorrelated over time, the volatility over 23 days will be CLOSEST to: A. 0.92%. B. 2.12%. C. 3.22%. D. 6.71%. 79. Consider a stock whose 5-day volatility has been estimated as 1.2%. Assuming that the returns from the stock are uncorrelated over time, the volatility over 22 days will be CLOSEST to: A. 0.57%. B. 2.52%. C. 5.28%. D. 5.63%. 80. What is the standard deviation of the following data? Probability Value 30% -15 40% 5 30% 25 A. 8.94. B. 15.49. C. 80.00. D. 240.00. 81. Asset 1 has correlation of 0.5 with asset 2. A portfolio with equal weights of these two assets has a standard deviation of 13. The standard deviation of asset 2 is 19.50. What is the approximate standard deviation of asset 1? A. 5. B. 10. C. 20. D. Insufficient Information. 82. Asset 1 has correlation of 0.5 with asset 2. A portfolio with equal weights of these two assets has a standard deviation of 13. The standard deviation of asset 2 is 19.50. What is the

http://64.106.152.240/inmarkets/passpro/index.cfm?event=questions....

2004-08-02

PASS PRO - Copyright 2004 Inmarkets Ltd - Licensed to Raymond chen

9/150 1

approximate standard deviation of asset 1? A. 5. B. 10. C. 20. D. Insufficient Information. 83. Which of the following statements about the correlation coefficient are FALSE? I. Correlation coefficient ranges between -1 and +1. II. Correlation coefficient is a measure of non-linear relationship between two random variables. III. Correlation coefficient can be calculated by scaling the covariance between two variables. IV. If the correlation coefficient between two random variables is zero they will be independent. A. I and II. B. I and III. C. II and IV. D. I, III and IV. 84. What is the mean for the following probability data? Probability Value 20% -4 40% 6 40% 12 A. 2.13. B. 6.40. C. 12.22. D. 16.17. 85. Which of the following statements are TRUE? I. Correlation coefficient ranges between -1 and +1. II. Correlation coefficient is a measure of linear relationship between two random variables. III. Correlation coefficient can be calculated by scaling the covariance between two variables. IV. If the correlation coefficient between two random variables is zero they will be independent. A. I and II. B. III and IV. C. I, II and III. D. I, II, III and IV. 86.

http://64.106.152.240/inmarkets/passpro/index.cfm?event=questions....

2004-08-02

PASS PRO - Copyright 2004 Inmarkets Ltd - Licensed to Raymond chen

0/150 2

Consider a stock whose 5-day volatility has been estimated as 0.89%. Assuming that the returns from the stock are uncorrelated over time, the volatility over 22 days will be CLOSEST to: A. 0.42%. B. 1.87%. C. 3.92%. D. 4.17%. 87. The covariance between the return from two securities is 5 and the correlation between them is 0.5. If the variance of the first return is 8, the variance of the second return will be CLOSEST to: A. 1.25. B. 1.56. C. 3.54. D. 12.50. 88. Which of the following statements are TRUE? I. Correlation coefficient ranges between -1 and +1. II. If two random variables are independent they will have a correlation coefficient of zero. III. Correlation coefficient is a measure of the linear relationship between a dependent and an independent variable. IV. Covariance between two variables can be calculated from the variances of the variables and their correlation coefficient. A. I and II. B. I and III. C. II and IV. D. I, II, III and IV. 89. Consider a stock whose 5-day volatility has been estimated as 1.5%. Assuming that the returns from the stock are uncorrelated over time, the volatility over 250 days will be CLOSEST to: A. 0.21%. B. 10.61%. C. 23.72%. D. 75.00%. 90. What is the mean for the following probability data?

http://64.106.152.240/inmarkets/passpro/index.cfm?event=questions....

2004-08-02

PASS PRO - Copyright 2004 Inmarkets Ltd - Licensed to Raymond chen

1/150 2

Probability Value 20% -4 40% 6 40% 12 A. 2.13. B. 6.40. C. 12.22. D. 16.17. 91. What is the mean for the following probability data? Probability Value 50% -2 25% 8 25% 15 A. 1.58. B. 2.25. C. 4.75. D. 5.38. 92. An analyst is studying a stock that is currently trading at $35. The analyst estimates that there is 33 percent probability that the stock will trade at $50 after one year, a 20 percent probability that the stock will trade at $42, and a 47 percent chance that the stock will trade at $20. What is the implied volatility of this stock price? A. 13%. B. 24%. C. 31%. D. 39%. 93. Which of the following investments has the highest co-efficient of variation? A. An investment with an expected return of 15% and a variance of return of 0.008. B. An investment with an expected return of 12% and a variance of return of 0.005. C. An investment with an expected return of 8% and standard deviation of return of 3%. D. An investment with an expected return of 3% and standard deviation of return of 2%. 94. An analyst regresses the returns of 60 stocks in a stock market and finds that the best fitting line is:

http://64.106.152.240/inmarkets/passpro/index.cfm?event=questions....

2004-08-02

PASS PRO - Copyright 2004 Inmarkets Ltd - Licensed to Raymond chen

2/150 2

Return = 8% + 9% x Beta If the standard error of the estimate is 6% and the standard error of the coefficient of Beta is 4%, the test statistic for the coefficient is CLOSEST to: A. 1.33. B. 1.43. C. 1.50. D. 2.25. 95. Which of the following investment has the highest co-efficient of variation? A. An investment with an expected return of 19% and a variance of return of 0.005. B. An investment with an expected return of 15% and a variance of return of 0.002. C. An investment with an expected return of 10% and standard deviation of return of 3%. D. An investment with an expected return of 3% and standard deviation of return of 1%. 96. Which of the following investments has the highest co-efficient of variation? A. An investment with an expected return of 15% and a variance of of return of 0.008. B. An investment with an expected return of 12% and a variance of of return of 0.005. C. An investment with an expected return of 8% and standard deviation of return of 3%. D. An investment with an expected return of 3% and standard deviation of return of 2%. 97. An analyst collects the data for interest rate expectations. The mean expected rate is 2.5 percent, the lowest expectation is 1 percent and the highest expectation is 5 percent. This distribution is: A. sparse. B. normal. C. skewed. D. abnormal. 98. Using the returns from 64 stocks, an analyst determines that the best fitting line for the capital market relationship is: Return = 2% + 12% x Beta If the standard error of the estimate is 3% and the standard error of the coefficient of Beta is 4%, the test statistic for the coefficient is CLOSEST to: A. 2

http://64.106.152.240/inmarkets/passpro/index.cfm?event=questions....

2004-08-02

PASS PRO - Copyright 2004 Inmarkets Ltd - Licensed to Raymond chen


B. 3 C. 4 D. 6 99.

3/150 2

An analyst regresses the returns of 60 stocks in a stock market and finds that the best fitting line is: Return = 8% + 9% x Beta If the standard error of the estimate is 6% and the standard error of the coefficient of Beta is 4%, the test statistic for the coefficient is CLOSEST to: A. 1.33. B. 1.43. C. 1.50. D. 2.25. 100. An analyst regresses the returns of 16 stocks in a stock market and finds that the best fitting line is: Return = 6.5% + 11.8% x Beta If the standard error of the estimate is 4% and the standard error of the coefficient of Beta is 3%, the test statistic for the coefficient is CLOSEST to: A. 1.63. B. 2.95. C. 2.80. D. 3.93. 101. If the mean P/E of 30 stocks in a certain industrial sector is 18 and the sample standard deviation is 3.5, standard error of the mean is CLOSEST to: A. 0.12. B. 0.34. C. 0.64. D. 1.56. 102. An analyst regresses the returns of 50 stocks in a stock market and finds that the best fitting line is: Return = 8% + 12.25% x Beta

http://64.106.152.240/inmarkets/passpro/index.cfm?event=questions....

2004-08-02

PASS PRO - Copyright 2004 Inmarkets Ltd - Licensed to Raymond chen

4/150 2

If the standard error of the estimate is 10% and the standard error of the coefficient of Beta is 8%, the test statistic for the coefficient is CLOSEST to: A. 0.80. B. 1.16. C. 1.23. D. 1.53. 103. Which of the following investments has the highest co-efficient of variation? A. An investment with an expected return of 15% and a variance of return of 0.008. B. An investment with an expected return of 12% and a variance of return of 0.005. C. An investment with an expected return of 8% and standard deviation of return of 3%. D. An investment with an expected return of 3% and standard deviation of return of 2%. 104. Using the returns from 64 stocks, an analyst determines that the best fitting line for the capital market relationship is: Return = 2% + 12% x Beta If the standard error of the estimate is 3% and the standard error of the coefficient of Beta is 4%, the test statistic for the coefficient is CLOSEST to: A. 2 B. 3 C. 4 D. 6 105. Which of the following investments has the highest co-efficient of variation? A. An investment with an expected return of 12% and a variance of of return of 0.005. B. An investment with an expected return of 10% and a variance of of return of 0.002. C. An investment with an expected return of 15% and standard deviation of return of 3%. D. An investment with an expected return of 6% and standard deviation of return of 5%. 106. Which of the following investments has the highest co-efficient of variation? A. An investment with an expected return of 10% and a variance of of return of 0.002. B. An investment with an expected return of 15% and a variance of of return of 0.003. C. An investment with an expected return of 8% and standard deviation of return of 3%. D. An investment with an expected return of 5% and standard deviation of return of 1%.

http://64.106.152.240/inmarkets/passpro/index.cfm?event=questions....

2004-08-02

PASS PRO - Copyright 2004 Inmarkets Ltd - Licensed to Raymond chen

5/150 2

107. If the mean P/E of 40 stocks in a certain industrial sector is 12 and the sample standard deviation is 4, standard error of the mean is CLOSEST to: A. 0.10. B. 0.32. C. 0.63. D. 1.58. 108. Which of the following investments has the highest co-efficient of variation? A. An investment with an expected return of 15% and a variance of of return of 0.008. B. An investment with an expected return of 12% and a variance of of return of 0.005. C. An investment with an expected return of 8% and standard deviation of return of 3%. D. An investment with an expected return of 3% and standard deviation of return of 2%. 109. If the mean P/E of 90 stocks in a certain industrial sector is 12 and the sample standard deviation is 6, standard error of the mean is CLOSEST to: A. 0.07. B. 0.26. C. 0.63. D. 1.58. 110. If the mean P/E of 50 stocks in a certain industrial sector is ten and the sample standard deviation is six, standard error of the mean is CLOSEST to: A. 0.12. B. 0.35. C. 0.85. D. 1.18. 111. If the mean P/E of 60 stocks in a certain industrial sector is 18 and the sample standard deviation is 7, standard error of the mean is CLOSEST to: A. 0.12. B. 0.34. C. 0.90. D. 1.11.

http://64.106.152.240/inmarkets/passpro/index.cfm?event=questions....

2004-08-02

PASS PRO - Copyright 2004 Inmarkets Ltd - Licensed to Raymond chen

6/150 2

112. If the mean P/E of 90 stocks in a certain industrial sector is 12 and the sample standard deviation is 6, standard error of the mean is CLOSEST to: A. 0.07. B. 0.26. C. 0.63. D. 1.58. 113. An analyst regresses the returns of 50 stocks in a stock market and finds that the best fitting line is: Return = 8% + 9% x Beta If the standard error of the estimate is 5.5% and the standard error of the coefficient of Beta is 3.25%, the test statistic for the coefficient is CLOSEST to: A. 1.45. B. 1.64. C. 1.55. D. 2.77. 114. An analyst wants to test whether the variance of return from pharmaceutical stocks is different from that of the overall market. For this purpose, he obtains the following data from a sample of 21 pharmaceutical stocks and a sample of 41 stocks that are representative of the market. Mean return from pharmaceutical stocks = 8% Standard deviation of return from pharmaceutical stocks = 9.2% Mean return from market stocks = 12% Standard deviation of return from market stocks = 13% Based on this information and a 0.05 significance level: A. there is sufficient evidence for a difference between the variance of pharmaceutical stocks and the variance of the market stocks. B. there is insufficient evidence for a difference between the variance of pharmaceutical stocks and the variance of the market stocks. C. there is sufficient evidence that there is no difference between the variance of pharmaceutical stocks and the variance of the market stocks. D. there is insufficient evidence that there is no difference between the variance of pharmaceutical stocks and the variance of the market stocks. 115. An analyst wants to test whether the mean spending by tourists coming to a holiday resort is equal to or less than $2,000 with a 1 percent level of significance. He finds that the average spending by 16 tourists is $2,200 and the standard deviation of the population is $400. The critical value of the Z statistic for this study is: A. .65. 1 B. -1.96.

http://64.106.152.240/inmarkets/passpro/index.cfm?event=questions....

2004-08-02

PASS PRO - Copyright 2004 Inmarkets Ltd - Licensed to Raymond chen

7/150 2

C. 2.33. D. .58. 2 116. A t-test is used instead of a z-test when: A. the sample size is small. B. greater accuracy is required. C. the variance of the population is known. D. the standard deviation is larger than the mean. 117. An analyst wants to test whether the standard deviation of return from pharmaceutical stocks is lower than 0.2. For this purpose, he obtains the following data from a sample of 30 pharmaceutical stocks. Mean return from pharmaceutical stocks = 8%. Standard deviation of return from pharmaceutical stocks = 12%. Mean return from the market = 12%. Standard deviation of return from the market = 16%. What is the appropriate test statistic for this test? A. t-statistic. B. z-statistic. C. F-statistic. D. Chi-squared statistic. 118. Using a sample size of 61 observations, an analyst determines that the standard deviation of the returns from a stock is 21 percent. Using a 0.05 significance level, the analyst: A. can conclude that the standard deviation of returns is higher than 14%. B. cannot conclude that the standard deviation of returns is higher than 14%. C. can conclude that the standard deviation of returns is not higher than 14%. D. none of the above. 119. An analyst wants to test whether the return from utility stocks are less volatile than that of the overall market. For this purpose, he obtains the following data from a sample of 21 utility stocks and a sample of 41 stocks that are representative of the market. Mean return from utility stocks = 9%. Standard deviation of return from utility stocks = 11%. Mean return from the market stocks = 12%. Standard deviation of return from the market stocks = 15%. Based on this information and a 0.05 significance level: A. there is sufficient evidence to say that the standard deviation of utility stocks lower than that of the market. B. there is sufficient evidence to say that the standard deviation of utility stocks higher than that of the market. C. there is insufficient evidence to say that the standard deviation of utility stocks lower than that of the market.

http://64.106.152.240/inmarkets/passpro/index.cfm?event=questions....

2004-08-02

PASS PRO - Copyright 2004 Inmarkets Ltd - Licensed to Raymond chen

8/150 2

D. none of the above. 120. Using a sample size of 20 observations, an analyst determines that the standard deviation of the returns from a stock is 12 percent. Using a 0.05 significance level, the analyst: A. can conclude that the standard deviation of returns is same as 18%. B. can conclude that the standard deviation of returns is different from 18%. C. cannot conclude that the standard deviation of returns is different from 18%. D. none of the above. 121. An analyst wants to test whether the return from transportation sector stocks is different from that of the utility stocks. For this purpose, he obtains the following data from a sample of 21 transportation stocks and a sample of 41 utility stocks. Mean return from transportation stocks = 15%. Standard deviation of return from transportation stocks = 12%. Mean return from utility stocks = 12%. Standard deviation of return from utility stocks = 13%. Based on this information and using a 0.05 significance level: A. there is no evidence for a difference between the means. B. there is sufficient evidence for a difference between the means. C. there is insufficient evidence for a difference between the means. D. none of the above. 122. Using a sample size of 15 observations, an analyst determines that the standard deviation of the returns from a stock is 10 percent. Using a 0.05 significance level, the analyst: A. can conclude that the standard deviation of returns is same as 15%. B. can conclude that the standard deviation of returns is different from 15%. C. cannot conclude that the standard deviation of returns is different from 15%. D. none of the above. 123. An analyst wants to test whether the return from transportation sector stocks is different from that of the utility stocks. For this purpose, he obtains the following data from a sample of 21 transportation stocks and a sample of 41 utility stocks. Mean return from transportation stocks = 9%. Standard deviation of return from transportation stocks = 13%. Mean return from utility stocks = 12%. Standard deviation of return from utility stocks = 15%. What is the value of the test statistic for this test? A. 0.78. B. 0.96. C. 1.13. D. 1.34.

http://64.106.152.240/inmarkets/passpro/index.cfm?event=questions....

2004-08-02

PASS PRO - Copyright 2004 Inmarkets Ltd - Licensed to Raymond chen

9/150 2

124. An analyst wants to test whether the standard deviation of return from utility stocks is lower than 0.14. For this purpose, he obtains the following data from a sample of 25 utility stocks. Mean return from utility stocks = 8%. Standard deviation of return from utility stocks = 12%. Mean return from the market = 10%. Standard deviation of return from the market = 15%. What is the critical value of the statistic for this test at a 0.05 level of significance? A. 13.85. B. 23.77. C. 36.42. D. 39.36. 125. An analyst wants to test whether the standard deviation of return from food and drinks stocks is lower than 0.18. For this purpose, he obtains the following data from a sample of 15 food and drinks stocks. Mean return from food and drinks stocks = 8%. Standard deviation of return from food and drinks stocks = 13%. Mean return from the market = 12%. Standard deviation of return from the market = 17% What is the value of the test statistic for this test? A. 6.83. B. 6.11. C. 7.30. D. 7.82. 126. An analyst is given the task of determining whether a group of 16 active portfolio managers have achieved a significantly higher performance (using a significance level of 0.05) than the average for all portfolio managers over a certain period. Over the period of the study, the active portfolio managers achieved a mean return of 15 percent. Over the same period the mean return for all portfolio managers was 12 percent and their standard deviation was 8 percent. The correct conclusion from this study is that: A. the performance of active portfolio managers is significantly higher than the average for all portfolio managers. B. the performance of active portfolio managers is not significantly higher than the average for all portfolio managers. C. the performance of active portfolio managers is significantly lower than the average for all portfolio managers. D. the performance of active portfolio managers is the same as the average for all portfolio managers. 127. The z-statistic cannot be used to test the mean of a population when the population variance is: A. known and the sample size is large. B. known and the sample size is small. C. unknown and the sample size is large.

http://64.106.152.240/inmarkets/passpro/index.cfm?event=questions....

2004-08-02

PASS PRO - Copyright 2004 Inmarkets Ltd - Licensed to Raymond chen


D. unknown and the sample size is small. 128. Which of the following is NOT a step in the hypothesis testing process? A. State a hypothesis. B. Identify the test statistic and its probability distribution. C. Obtain the significance level from the data. D. Make the statistical decision. 129.

0/150 3

Rutherford Enterprises has a sales force of 1,000 sales people. The average turnover generated by each sales person is $800,000 and the standard deviation of this population is $100,000. If Peter, a salesman at Rutherford, has generated a turnover of $1,000,000, we can conclude, using a significance level of 0.05, that Peter's performance: A. is significantly higher than average. B. is not significantly higher than average. C. is lower or not significantly higher than average. D. cannot be determined on the basis of this data alone. 130. An analyst wants to test whether the return from utility stocks are less volatile than that of the overall market. For this purpose, he obtains the following data from a sample of 21 utility stocks and a sample of 41 stocks that are representative of the market. Mean return from utility stocks = 9%. Standard deviation of return from utility stocks = 11%. Mean return from the market stocks = 12%. Standard deviation of return from the market stocks = 15%. What is the critical value of the statistic for this test at a 0.05 level of significance? A. 1.84. B. 1.99. C. 2.07. D. 2.29. 131. An analyst wants to test whether the variance of return from oil and gas stocks is higher than 0.045. For this purpose, he obtains the following data from a sample of 60 oil and gas stocks. Mean return from oil and gas stocks = 16%. Standard deviation of return from oil and gas stocks = 22% Mean return from the market = 12%. Standard deviation of return from the market = 14% Based on this information, we can say that at a 0.05 significance level the variance of oil and gas firms is: A. lower than 0.045. B. higher than 0.045. C. not higher than 0.045. D. none of the above.

http://64.106.152.240/inmarkets/passpro/index.cfm?event=questions....

2004-08-02

PASS PRO - Copyright 2004 Inmarkets Ltd - Licensed to Raymond chen

1/150 3

132. Using a sample size of 20 observations, an analyst determines that the standard deviation of the returns from a stock is 9 percent. Using a 0.05 significance level, the analyst: A. can conclude that the standard deviation of returns is same as 16%. B. can conclude that the standard deviation of returns is different from 16%. C. cannot conclude that the standard deviation of returns is different from 16%. D. none of the above. 133. You want to test at the 0.05 level of significance that the mean price of luxury cars is greater than $80,000. A random sample of 50 cars has a mean price of $88,000. The population standard deviation is $15,000. What is the alternative hypothesis? A. The population mean is greater than or equal to $80,000. B. The population mean is less than $80,000. C. The population mean is not equal to $80,000. D. The population mean is greater than is $80,000. 134. An analyst is studying the impact of the dot-com bust on the beta of business services sector stocks. From price data for 10 stocks in this sector, she has obtained the following results: Mean beta of the stocks before dot-com bust = 1.3 Mean beta of the stocks after dot-com bust = 1.4 Standard deviation before and after dot-com bust = 0.25. Mean difference in betas before and after dot-com bust = 0.18 The standard deviation of differences in betas before and after dot-com bust = 0.2. Based on these results and using a 0.05 significance level, the analyst: A. can conclude that the dot-com bust has affected the beta of these stocks. B. cannot conclude that the dot-com bust has affected the beta of these stocks. C. can conclude that the dot-com bust has not affected the beta of these stocks. D. none of the above. 135. An analyst wants to test whether the variance of return from telecom stocks is higher than 0.04. For this purpose, he obtains the following data from a sample of 31 telecom stocks. Mean return from telecom stocks = 15% Standard deviation of return from telecom stocks = 24% Mean return from market = 12% Standard deviation of return from market = 13% What is the critical value of the statistic for this test at a 0.05 level of significance? A. 16.79. B. 18.49. C. 43.73. D. 46.98. 136.

http://64.106.152.240/inmarkets/passpro/index.cfm?event=questions....

2004-08-02

PASS PRO - Copyright 2004 Inmarkets Ltd - Licensed to Raymond chen

2/150 3

An analyst wants to test whether the variance of return from oil and gas stocks is higher than 0.045. For this purpose, he obtains the following data from a sample of 60 oil and gas stocks. Mean return from oil and gas stocks = 16%. Standard deviation of return from oil and gas stocks = 22% Mean return from the market = 12%. Standard deviation of return from the market = 14% What is the value of the test statistic for this test? A. 61.19. B. 63.46. C. 64.53. D. 93.33. 137. An analyst is studying the impact of the 1990 deregulation on the beta of financial services sector stocks. From price data for 12 stocks in this sector she has obtained the following results: Mean beta of the stocks before 1990 deregulation = 1.2 Mean beta of the stocks after 1990 deregulation = 1.4 The standard deviation before and after 1990 deregulation = 0.5. Mean difference in betas before and after 1990 deregulation = 0.25 Standard deviation of differences in betas before and after 1990 deregulation = 0.3. Based on these results and using a 0.01 significance level, the analyst: A. can conclude that the 1990 deregulation has affected the beta of these stocks. B. cannot conclude that the 1990 deregulation has affected the beta of these stocks. C. can conclude that the 1990 deregulation has not affected the beta of these stocks. D. none of the above. 138. Which of the following statements are NOT true? I. Type I error occurs when the null hypothesis is not rejected when it is actually false. II. Type II error occurs when the null hypothesis is rejected when it is actually true. III. Type I error occurs when the alternate hypothesis is wrongly accepted. IV. Minimizing the probability of Type II error maximizes the power of the test. A. I and II. B. I and III. C. II and IV. D. I, II and IV. 139. An analyst is using a statistical package to perform a linear regression between the risk and return from securities in an emerging market country. The original data and intermediate statistics are shown on the right. The value of coefficient of determination for this regression is CLOSEST to:

http://64.106.152.240/inmarkets/passpro/index.cfm?event=questions....

2004-08-02

PASS PRO - Copyright 2004 Inmarkets Ltd - Licensed to Raymond chen


A. 0.043. B. 0.084. C. 0.916. D. 0.957.

3/150 3

140. An analyst has constructed the following t-test for a portfolio of financial securities whose returns are normally distributed: Number of securities = 40. H0: Mean return >= 18 percent. Significance level = 0.1 What is the rejection point for this test? A. 1.304. B. 1.684. C. 2.021. D. 2.023. 141. A fund manager is using a statistical package to perform a linear regression between the number of securities that she uses to replicate an index and the 95 percent shortfall risk. The original data and intermediate statistics are shown on the right. The correlation between the price and volume is CLOSEST to:

A. 0.120. B.

http://64.106.152.240/inmarkets/passpro/index.cfm?event=questions....

2004-08-02

PASS PRO - Copyright 2004 Inmarkets Ltd - Licensed to Raymond chen


0.062. C. 0.880. D. 0.938.

4/150 3

142. An analyst is using a statistical package to perform a linear regression between the risk and return from securities in an emerging market country. The original data and intermediate statistics are shown on the right. The value of the intercept for this regression is CLOSEST to:

A. -2.97. B. -1.09. C. 3.07. D. 5.62.

143. A fund manager is using a statistical package to perform a linear regression between the number of securities that she uses to replicate an index and the 95 percent shortfall risk. The original data and intermediate statistics are shown on the right. The value of the intercept for this regression is CLOSEST to:

http://64.106.152.240/inmarkets/passpro/index.cfm?event=questions....

2004-08-02

PASS PRO - Copyright 2004 Inmarkets Ltd - Licensed to Raymond chen

5/150 3

A. -51.85. B. 0. C. 20.51. D. 38.22.

144. An analyst is using a statistical package to perform a linear regression between price of a new semiconductor cleaning product and its price. The original data and intermediate statistics are shown on the right. The correlation between the price and volume is CLOSEST to:

A. 0.038. B. 0.074. C. 0.926. D. 0.962.

145. An analyst is using a statistical package to perform a linear regression between the features offered in a product and its sales volume at a constant price. The original data and intermediate statistics are shown on the right. The standard error of the estimate from this regression is CLOSEST to:

http://64.106.152.240/inmarkets/passpro/index.cfm?event=questions....

2004-08-02

PASS PRO - Copyright 2004 Inmarkets Ltd - Licensed to Raymond chen

6/150 3

A. 53. B. 59. C. 2,852. D. 3,423.

146. A fund manager is using a statistical package to perform a linear regression between the number of securities that she uses to replicate an index and the 95 percent shortfall risk. The original data and intermediate statistics are shown on the right. The value of the intercept for this regression is CLOSEST to:

A. -51.85. B. 0. C. 20.51. D. 38.22.

147. An analyst is using a statistical package to perform a linear regression between the risk and return from securities in an emerging market country. The original data and intermediate statistics are shown on the right. The correlation between the price and volume is CLOSEST to:

http://64.106.152.240/inmarkets/passpro/index.cfm?event=questions....

2004-08-02

PASS PRO - Copyright 2004 Inmarkets Ltd - Licensed to Raymond chen

7/150 3

A. 0.043. B. 0.084. C. 0.916. D. 0.957.

148. If the correlation coefficient of a linear regression is 0.6, the percentage of variation of the dependent variable that is not explained by the independent variable is CLOSEST to: A. 36%. B. 40%. C. 60%. D. 64%. 149. A fund manager is using a statistical package to perform a linear regression between the number of securities that she uses to replicate an index and the 95 percent shortfall risk. The original data and intermediate statistics are shown on the right. The correlation between the price and volume is CLOSEST to:

A. 0.120. B. 0.062. C. 0.880. D. 0.938.

http://64.106.152.240/inmarkets/passpro/index.cfm?event=questions....

2004-08-02

PASS PRO - Copyright 2004 Inmarkets Ltd - Licensed to Raymond chen

8/150 3

150. If the correlation coefficient of a linear regression is 0.75, the percentage of variation of the dependent variable that is not explained by the independent variable is CLOSEST to: A. 25%. B. 44%. C. 56%. D. 75%. 151. If the correlation coefficient of a linear regression is 0.85, the percentage of variation of the dependent variable that is not explained by the independent variable is CLOSEST to: A. 15%. B. 28%. C. 72%. D. 85%. 152. If the correlation coefficient of a linear regression is 0.25, the percentage of variation of the dependent variable that is not explained by the independent variable is CLOSEST to: A. 6%. B. 25%. C. 75%.

http://64.106.152.240/inmarkets/passpro/index.cfm?event=questions....

2004-08-02

PASS PRO - Copyright 2004 Inmarkets Ltd - Licensed to Raymond chen


D. 94%. 153. An analyst is using a statistical package to perform a linear regression between the risk and return from securities in an emerging market country. The original data and intermediate statistics are shown on the right. The value of the intercept for this regression is CLOSEST to:

9/150 3

A. -2.97. B. -1.09. C. 3.07. D. 5.62.

154. If the correlation coefficient of a linear regression is 0.75, the percentage of variation of the dependent variable that is not explained by the independent variable is CLOSEST to: A. 25%. B. 44%. C. 56%. D. 75%. 155. Which of the following test statistics is most appropriate for conducting the hypothesis test given below? H0: variance A = variance B; where the test is based on two random independent samples from two normally distributed populations. A. t-statistic. B. z-statistic. C. F-statistic. D. chi-square. 156. An analyst is using a statistical package to perform a linear regression between price of a new semi-

http://64.106.152.240/inmarkets/passpro/index.cfm?event=questions....

2004-08-02

PASS PRO - Copyright 2004 Inmarkets Ltd - Licensed to Raymond chen

0/150 4

conductor cleaning product and its price. The original data and intermediate statistics are shown on the right. The value of coefficient of determination for this regression is CLOSEST to:

A. 0.038. B. 0.074. C. 0.926. D. 0.962.

157. An analyst is using a statistical package to perform a linear regression between the risk and return from securities in an emerging market country. The original data and intermediate statistics are shown on the right. The standard error of the estimate from this regression is CLOSEST to:

A. 0.225. B. 0.247. C. 0.051. D. 0.061.

158. An analyst is using a statistical package to perform a linear regression between price of a new semiconductor cleaning product and its price. The original data and intermediate statistics are shown on the right. The standard error of the estimate from this regression is CLOSEST to:

http://64.106.152.240/inmarkets/passpro/index.cfm?event=questions....

2004-08-02

PASS PRO - Copyright 2004 Inmarkets Ltd - Licensed to Raymond chen

1/150 4

A. 55. B. 60. C. 3,009. D. 3,611.

159. An analyst is using a statistical package to perform a linear regression between the risk and return from securities in an emerging market country. The original data and intermediate statistics are shown on the right. The value of the slope coefficient for this regression is CLOSEST to:

A. 0.37. B. 1.24. C. 2.09. D. 7.11.

160. If the correlation coefficient of a linear regression is 0.25, the percentage of variation of the dependent variable that is not explained by the independent variable is CLOSEST to: A. 6%. B. 25%. C. 75%.

http://64.106.152.240/inmarkets/passpro/index.cfm?event=questions....

2004-08-02

PASS PRO - Copyright 2004 Inmarkets Ltd - Licensed to Raymond chen


D. 94%. 161. An analyst is using a statistical package to perform a linear regression between the features offered in a product and its sales volume at a constant price. The original data and intermediate statistics are shown on the right. The value of the slope coefficient for this regression is CLOSEST to:

2/150 4

A. 7.0. B. 3.6. C. 58.6. D. 113.9.

162. An analyst is using a statistical package to perform a linear regression between price of a new semiconductor cleaning product and its price. The original data and intermediate statistics are shown on the right. The correlation between the price and volume is CLOSEST to:

A. 0.038. B. 0.074. C. 0.926. D. 0.962.

163. An analyst is using a statistical package to perform a linear regression between the features offered in a product and its sales volume at a constant price. The original data and intermediate statistics are shown on the right. The correlation between the price and volume is CLOSEST to:

http://64.106.152.240/inmarkets/passpro/index.cfm?event=questions....

2004-08-02

PASS PRO - Copyright 2004 Inmarkets Ltd - Licensed to Raymond chen

3/150 4

A. 0.079. B. 0.151. C. 0.849. D. 0.921.

164. A portfolio contains two perfectly negatively correlated investments with volatilities of 5 percent and 7 percent. The proportion of these two securities that would lead to the lowest risk are: A. 58% and 42%. B. 42% and 58%. C. 34% and 66%. D. 66% and 34%. 165. If Security A and Security B are positively correlated, and the price of Security A increases, the price of Security B: A. will increase. B. will decrease. C. is most likely to increase than to decrease. D. may decrease or remain unchanged, but will not increase. 166. A portfolio contains two perfectly negatively correlated investments with volatilities of 5 percent and 7 percent. The proportion of these two securities that would lead to the lowest risk are: A. 34% and 66%. B. 66% and 34%. C. 58% and 42%. D. 42% and 58%.

http://64.106.152.240/inmarkets/passpro/index.cfm?event=questions....

2004-08-02

PASS PRO - Copyright 2004 Inmarkets Ltd - Licensed to Raymond chen

4/150 4

167. A portfolio contains two perfectly negatively correlated investments with volatilities of 5 percent and 7 percent. The proportion of these two securities that would lead to the lowest risk are: A. 34% and 66%. B. 66% and 34%. C. 58% and 42%. D. 42% and 58%. 168. A portfolio contains two perfectly negatively correlated investments with volatilities of 5 percent and 7 percent. The proportion of these two securities that would lead to the lowest risk are: A. 58% and 42%. B. 42% and 58%. C. 34% and 66%. D. 66% and 34%. 169. If Security A and Security B are positively correlated, and the price of Security A increases, the price of Security B: A. will increase. B. will decrease. C. may increase or decrease, but is most likely to increase. D. may decrease or remain unchanged, but will not increase. 170. The lowest level of risk, measured by volatility, of a portfolio containing two perfectly negatively correlated investments with volatilities of 3 percent and 10 percent respectively is: A. 0% B. 3% C. 10% D. 13% 171. The lowest level of risk, measured by volatility, of a portfolio containing two perfectly negatively correlated investments with volatilities of 5 percent and 7 percent is: A. 0%. B. 5%. C. 7%. D. 12%.

http://64.106.152.240/inmarkets/passpro/index.cfm?event=questions....

2004-08-02

PASS PRO - Copyright 2004 Inmarkets Ltd - Licensed to Raymond chen

5/150 4

172. If Security A and Security B are positively correlated, and the price of Security A increases, the price of Security B: A. will increase. B. will decrease. C. may increase or decrease, but is most likely to increase. D. may decrease or remain unchanged, but will not increase. 173. The variance of the returns from stock A is 0.018 and that of the market is 0.025. If the covariance between the stock and the index is -0.002, their correlation coefficient is CLOSEST to: A. -0.23. B. -0.11. C. -0.09. D. -0.08. 174. A positive covariance means that: A. there is no relationship between asset returns. B. the correlation coefficient is zero. C. asset returns move in the opposite direction. D. asset returns move in the same direction. 175. Which of the following model incorporates both mean reversion and no-arbitrage? A. Vasicek. B. Black Scholes. C. Cox, Ingersoll and Ross. D. Heath, Jarrow and Morton. 176. Given two variables X and Y that follow geometric Brownian motion, which of the following variable will also follow geometric Brownian motion? A. X * Y. B. X + Y. C. log(X) + log(Y). D. exp(X) + exp(Y).

http://64.106.152.240/inmarkets/passpro/index.cfm?event=questions....

2004-08-02

PASS PRO - Copyright 2004 Inmarkets Ltd - Licensed to Raymond chen

6/150 4

177. The standard deviation of a Wiener process over a short period of time would be proportional to: A. the time period. B. square of the time period. C. square-root of the time period. D. none of the above. 178. For which of the following process can the mean change be not equal to zero? I. Ito process. II. Wiener process. III. Generalized Wiener process. A. I and II. B. I and III. C. II and III. D. I, II and III. 179. Which of the following model incorporates both mean reversion and no-arbitrage? A. Vasicek. B. Ho and Lee. C. Black Scholes. D. Hull and White. 180. The standard deviation of a Wiener process over a short period of time would be proportional to: A. the time period. B. square of the time period. C. square-root of the time period. D. none of the above. 181. Given two variables X and Y that are lognormally distributed, what is the distribution of X * Y? A. Normal. B. Lognormal. C. Exponential.

http://64.106.152.240/inmarkets/passpro/index.cfm?event=questions....

2004-08-02

PASS PRO - Copyright 2004 Inmarkets Ltd - Licensed to Raymond chen


D. None of the above. 182. Which of the following are no-arbitrage models for fixed income valuation? I. Ho and Lee. II. Hull and White. III. Cox, Ingersoll and Ross. IV. Heath, Jarrow and Morton. A. I and II. B. III and IV. C. I, II and IV. D. II, III and IV. 183. Which of the following models for interest rates would allow for mean reversion? A. dr = a * dt + s * dz B. dr = a * dt - b * dt + s * dz C. dr = a * (b - r) * dt + s * dz D. dr = a * (r - b) * dt + s * dz 184. Vasicek proposed the following model for interest rates: dr = a * (b - r) * dt + s * dz What is the long term mean of interest rates in this model? A. a B. b C. r D. s 185.

7/150 4

Given two variables X and Y that are normally distributed, what of the following will also be normally distributed? A. X * Y. B. X + Y. C. log(X) + log(Y). D. exp(X) + exp(Y). 186. Assuming continuous compounding and an annual percentage rate of 6.5%, the present value

http://64.106.152.240/inmarkets/passpro/index.cfm?event=questions....

2004-08-02

PASS PRO - Copyright 2004 Inmarkets Ltd - Licensed to Raymond chen

8/150 4

of a cash amount of $8,500 that we will receive after 5 years is CLOSEST to: A. $6,372. B. $6,141. C. $8,022. D. $8,030. 187. Assuming continuous compounding and an annual percentage rate of 9%, the present value of a cash amount of $600 that we will receive after 8 years is CLOSEST to: A. $292. B. $301. C. $323. D. $349. 188. Assuming continuous compounding and an annual percentage rate of 9%, the present value of a cash amount of $1,500 that we will receive after 4 years is CLOSEST to: A. $1,047. B. $1,063. C. $1,103. D. $1,141. 189. If a cash flow of $700 in 3 years' time has a PV of $400, the annual percentage rate, assuming continuous compounding is CLOSEST to: A. 8.52%. B. 12.16%. C. 18.65%. D. 25.44%. 190. The price of a stock has risen from $15 to $40 over the past 2 years. If the stock did not pay any dividends during this period, its continuously compounded return is CLOSEST to: A. 9.15% B. 14.94% C. 29.24% D. 49.04% 191.

http://64.106.152.240/inmarkets/passpro/index.cfm?event=questions....

2004-08-02

PASS PRO - Copyright 2004 Inmarkets Ltd - Licensed to Raymond chen

9/150 4

The present value of $4,500 received in 6 month's time, using a continuous discount rate of 9%, is CLOSEST to: A. $3,290 B. $4,302 C. $4,306 D. $4,310 192. The price of a stock has risen from $15 to $40 over the past 2 years. If the stock did not pay any dividends during this period, its continuously compounded return is CLOSEST to: A. 9.15% B. 14.94% C. 29.24% D. 49.04% 193. The present value of $500 received in 9 month's time, using a continuous discount rate of 9%, is CLOSEST to: A. $467. B. $468. C. $469. D. $470. 194. If a cash flow of $10,000 in two years' time has a PV of $8,455, the annual percentage rate, assuming continuous compounding is CLOSEST to: A. 8.13%. B. 8.39%. C. 8.75%. D. 8.95%. 195. If a cash flow of $10,000 in two years' time has a PV of $8,455, the annual percentage rate, assuming continuous compounding is CLOSEST to: A. 8.13%. B. 8.39%. C. 8.75%. D. 8.95%.

http://64.106.152.240/inmarkets/passpro/index.cfm?event=questions....

2004-08-02

PASS PRO - Copyright 2004 Inmarkets Ltd - Licensed to Raymond chen

0/150 5

196. The present value of $4,500 received in 6 month's time, using a continuous discount rate of 9%, is CLOSEST to: A. $3,290 B. $4,302 C. $4,306 D. $4,310 197. If a cash flow of $100 in 2 years' time has a PV of $75, the annual percentage rate, assuming continuous compounding is CLOSEST to: A. 11.1%. B. 12.2%. C. 13.5%. D. 14.4%. 198. If a cash flow of $800 in 6 years' time has a PV of $500, the annual percentage rate, assuming continuous compounding is CLOSEST to: A. 4.48%. B. 5.23%. C. 7.83%. D. 9.12%. 199. Assuming continuous compounding and an annual percentage rate of 9%, the present value of a cash amount of $400 that we will receive after 8 years is CLOSEST to: A. $195. B. $201. C. $233. D. $252. 200. Assuming continuous compounding and an annual percentage rate of 6.5%, the present value of a cash amount of $8,500 that we will receive after 5 years is CLOSEST to: A. $6,372. B. $6,141. C. $8,022. D. $8,030.

http://64.106.152.240/inmarkets/passpro/index.cfm?event=questions....

2004-08-02

PASS PRO - Copyright 2004 Inmarkets Ltd - Licensed to Raymond chen

1/150 5

201. Assuming continuous compounding and an annual percentage rate of 6.5%, the present value of a cash amount of $8,500 that we will receive after 5 years is CLOSEST to: A. $6,372. B. $6,141. C. $8,022. D. $8,030. 202. Given that a stock price rises on six days out of ten and falls on four days out of ten, what is the probability that it will rise on exactly eight out of the next twelve days? A. 4.2%. B. 5.8%. C. 20.2%. D. 21.3%. 203. A sports firm produces two different sets of golf clubs and five types of high-tech golf balls. If the firm bundles each set of golf club with three different balls, how may unique packages can it make? A. 20. B. 40. C. 120. D. 250. 204. An analyst determines that in the current market environment the S&P500 goes up 7 out of every 15 days but without any serial dependency, i.e. probability of going up on a certain date is not affected by its movement on the previous days. Given that 275 trading days in a year the standard deviation in the number of days that the S&P500 will go up is CLOSEST to: A. 7.74. B. 8.27. C. 59.97. D. 68.45. 205. Which of the following is the most appropriate for modeling stock prices? A. Normal distribution. B. Random distribution.

http://64.106.152.240/inmarkets/passpro/index.cfm?event=questions....

2004-08-02

PASS PRO - Copyright 2004 Inmarkets Ltd - Licensed to Raymond chen


C. Bernoulli distribution. D. Lognormal distribution. 206. The lognormal distribution is: A. not skewed. B. positively skewed. C. negatively skewed.

2/150 5

D. skewed, but the degree of skew is dependent on the a relative mean of the distribution. 207. The lognormal distribution is: A. not skewed. B. positively skewed. C. negatively skewed. D. skewed, but the degree of skew is dependent on the a relative mean of the distribution. 208. The mean age of the 200 employees in a company is 32 and the standard deviation is 12. Assuming that the ages are normally distributed and using 95 percent confidence we can say that the employees within the firm fall between: A. 8.5 and 55.5 years. B. 29.7 and 34.3 years. C. 30.3 and 33.7 years. D. 31.9 and 32.1 years. 209. A portfolio has a mean value of $60 million and a daily standard deviation of $8 million. Assuming that the portfolio values are normally distributed, the lowest value that the portfolio will fall to over the next five days and within 99% probability is: A. $4.5 million. B. $18.4 million. C. $30.6 million. D. $42.1 million. 210. Which of the following are characteristic of a normal distribution? I. It is bell shaped. II. It is a continuous distribution.

http://64.106.152.240/inmarkets/passpro/index.cfm?event=questions....

2004-08-02

PASS PRO - Copyright 2004 Inmarkets Ltd - Licensed to Raymond chen

3/150 5

III. It is symmetrical about the mean. IV. It peaks at the mean expected value. A. I and II. B. I and IV. C. I, II and III. D. I, II, III and IV. 211. What is the quantile corresponding to the 95% confidence interval? A. -3.715. B. -2.326. C. -1.645. D. -1.282. 212. The covariance between the return from two securities is 16 and the correlation between them is 0.5. If the variance of the first return is 8, the variance of the second return will be CLOSEST to: A. 4.00. B. 11.31. C. 16.00. D. 128.00. 213. The mean age of the 80 employees in a company is 35 and the standard deviation is 15. Assuming that the ages are normally distributed and using 95 percent confidence, we can say that the employees within the firm fall between: A. 20.0 and 50.0 years. B. 31.7 and 38.3 years. C. 33.8 and 36.2 years. D. 34.6 and 35.4 years. 214. The covariance between the return from two securities is 16 and the correlation between them is 0.5. If the variance of the first return is 8, the variance of the second return will be CLOSEST to: A. 4.00. B. 11.31. C. 16.00. D. 128.00.

http://64.106.152.240/inmarkets/passpro/index.cfm?event=questions....

2004-08-02

PASS PRO - Copyright 2004 Inmarkets Ltd - Licensed to Raymond chen

4/150 5

215. What is the quantile corresponding to the 99.99% confidence interval? A. -3.715. B. -2.326. C. -1.645. D. -1.282. 216. Which of the following are characteristic of a normal distribution? I. Its kurtosis is equal to zero. II. Its skewness is less than zero. III. It is symmetrical about the mean. IV. It has the same mean, median and mode. A. I and III. B. III and IV. C. I, III and IV. D. I, II, III and IV. 217. Consider a portfolio whose expected return is normally distributed with a mean of 20 percent and a standard deviation of 10 percent. The probability that the return will lie between 0 percent and 10 percent is CLOSEST to: A. 2%. B. 6%. C. 14%. D. 19%. 218. A portfolio has a mean value of $70 million and a daily standard deviation of $9.62 million. Assuming that the portfolio values are normally distributed, the probability of the portfolio value falling below $20 million within the next five days is CLOSEST to: A. 0.10%. B. 1.00%. C. 5.00%. D. 15.87%. 219. The covariance between the return from two securities is 5 and the correlation between them is 0.5. If the variance of the first return is 8, the variance of the second return will be CLOSEST to:

http://64.106.152.240/inmarkets/passpro/index.cfm?event=questions....

2004-08-02

PASS PRO - Copyright 2004 Inmarkets Ltd - Licensed to Raymond chen


A. 1.25. B. 1.56. C. 3.54. D. 12.50. 220.

5/150 5

Which of the following is FALSE in relation to the distribution of prices of an asset that is found to be lognormal? A. The skewness of the returns distribution will be zero. B. The mean and median of the price distribution will not be equal. C. The mean, median and mode of the returns distribution will be equal. D. If volatility increases but the mean stays constant, the median of price distribution will shift to the right. 221. The covariance between the return from two securities is 4.2 and the correlation between them is 0.6. If the variance of the first return is 5, the variance of the second return will be CLOSEST to: A. 1.40. B. 1.96. C. 3.13. D. 9.80. 222. A portfolio has a mean value of $100 million and a daily standard deviation of $2.5 million. Assuming that the portfolio values are normally distributed, the lowest value that the portfolio will fall to over the next 14 days and within 95% probability is: A. $18.6 million. B. $42.4 million. C. $78.3 million. D. $84.6 million. 223. A sports firm produces two different sets of golf clubs and five types of high-tech golf balls. If the firm bundles each set of golf club with three different balls, how may unique packages can it make? A. 20. B. 40. C. 120. D. 250.

http://64.106.152.240/inmarkets/passpro/index.cfm?event=questions....

2004-08-02

PASS PRO - Copyright 2004 Inmarkets Ltd - Licensed to Raymond chen

6/150 5

224. A sports firm produces three different sets of golf clubs and three types of high-tech golf balls. If the firm bundles each set of golf club with two different balls, how may unique packages can it make? A. 9. B. 18. C. 27. D. 81. 225. An analyst observes that the closing price of a stock during a week as $33, $43, $45, $48, $46. On the corresponding days the S&P 500 closed at 1150, 1125, 1140, 1160, 1170. Based on this data the covariance of the stock with the market is CLOSEST to: A. 15.8 B. 18.0 C. 37.4 D. 54.4 226. A sports firm produces two different sets of golf clubs and five types of high-tech golf balls. If the firm bundles each set of golf club with three different balls, how may unique packages can it make? A. 20. B. 40. C. 120. D. 250. 227. If a loan is being offered at a rate of interest of 14.49 percent compounded monthly, the borrower will pay: I. a nominal rate of 13.61%. II. a periodic rate of 14.49%. III. a stated rate of 14.49%. IV. an effective rate of 15.49%. A. I and II. B. II and III. C. III and IV. D. I, III and IV. 228.

http://64.106.152.240/inmarkets/passpro/index.cfm?event=questions....

2004-08-02

PASS PRO - Copyright 2004 Inmarkets Ltd - Licensed to Raymond chen

7/150 5

As the discount rate rises, the NPV of a set of cash flows: A. increases. B. decreases. C. stays constant. D. cannot be determined without further information. 229. An investor is being promised an annuity of $6,000 starting in five years from now and continuing for 15 years. Assuming a constant interest rate of 6 percent per annum, the value of this annuity today is CLOSEST to: A. $46,158. B. $58,273. C. $67,253. D. $71,288. 230. An investor is being promised an annuity of $900 starting in two years from now and continuing for 25 years. Assuming a constant interest rate of 8 percent per annum, the value of this annuity today is CLOSEST to: A. $8,895. B. $9,607. C. $19,290. D. $20,833. 231. Assuming continuous compounding and an annual percentage rate of 7%, the present value of $4,700 received in nine month's time is CLOSEST to: A. $3,506. B. $4,460. C. $4,466. D. $4,467. 232. What is the approximate price of a zero coupon bond whose par value is 100 and term is two years if its yields is 5%? A. 86. B. 91. C. 96. D. 101.

http://64.106.152.240/inmarkets/passpro/index.cfm?event=questions....

2004-08-02

PASS PRO - Copyright 2004 Inmarkets Ltd - Licensed to Raymond chen

8/150 5

233. Assuming continuous compounding and an annual percentage rate of 8%, the present value of $7,000 received in eight months' time is CLOSEST to: A. $6,636. B. $6,646. C. $6,650. D. $6,665. 234. An investor is being promised an annuity of $2,000 starting in two years from now and continuing for eight years. Assuming a constant interest rate of 4 percent per annum, the value of this annuity today is CLOSEST to: A. $12,947. B. $13,465. C. $14,793. D. $15,385. 235. The inflation rate in an economy where the prices are doubling ever 15 years is CLOSEST to: A. 3%. B. 5%. C. 7%. D. 13%. 236. If all the cash flows in an investment are positive what is its IRR? A. Positive. B. Zero. C. Negative. D. Cannot be determined. 237. An investor is being promised an annuity of $6,000 starting in five years from now and continuing for 15 years. Assuming a constant interest rate of 6 percent per annum, the value of this annuity today is CLOSEST to: A. $46,158. B. $58,273. C. $67,253. D. $71,288.

http://64.106.152.240/inmarkets/passpro/index.cfm?event=questions....

2004-08-02

PASS PRO - Copyright 2004 Inmarkets Ltd - Licensed to Raymond chen

9/150 5

238. If a cash flow of $15,000 in two years' time has a PV of $12,000, the annual percentage rate, assuming continuous compounding is CLOSEST to: A. 11.16%. B. 11.44%. C. 11.69%. D. 11.80%. 239. Suppose that you need to borrow $1 million for 24 months. Two large US-based international banks with equal credit ratings offer deposit rates of 2%. To choose between the two banks, you would need all of the following except: A. day count basis. B. compounding basis. C. currency of deposit. D. balance sheets of the banks. 240. An investor is being promised an annuity of $1,500 starting in 15 years from now and continuing for ten years. Assuming a constant interest rate of 8 percent per annum, the value of this annuity today is CLOSEST to: A. $3,427. B. $4,729. C. $5,107. D. $10,065. 241. Suppose that you need to borrow $1 million for 24 months. Two large US-based international banks with equal credit ratings offer deposit rates of 2%. To choose between the two banks, you would need all of the following except: A. day count basis. B. compounding basis. C. currency of deposit. D. balance sheets of the banks. 242. A bank is offering loans with monthly interest payments based on a stated annual interest rate of 7.75 percent. The effective annual rate of these loans is CLOSEST to: A. 7.90%.

http://64.106.152.240/inmarkets/passpro/index.cfm?event=questions....

2004-08-02

PASS PRO - Copyright 2004 Inmarkets Ltd - Licensed to Raymond chen


B. 7.98%. C. 8.00%. D. 8.03%. 243.

0/150 6

If a cash flow of $10,000 in five years' time has a PV of $7,500, the annual percentage rate, assuming continuous compounding, is CLOSEST to: A. 5.65%. B. 5.75%. C. 5.85%. D. 5.92%. 244. Consider a 4.25 percent semi-annual coupon bond with a par value of $100 and three remaining coupons, which is trading at a yield of 3.975 percent. There are 45 days remaining in the current period that has a total of 180 days. The clean price of this bond is CLOSEST to: A. 99.70. B. 100.30. C. 101.89. D. 102.32. 245. As the discount rate rises, the NPV of a set of cash flows: A. increases. B. decreases. C. stays constant. D. cannot be determined without further information. 246. Assuming continuous compounding and an annual percentage rate of 8%, the present value of $8,400 received in six months' time is CLOSEST to: A. $6,203. B. $8,071. C. $8,077. D. $8,083. 247. A bank is offering loans with monthly interest payments based on a stated annual interest rate of 7.75 percent. The effective annual rate of these loans is CLOSEST to:

http://64.106.152.240/inmarkets/passpro/index.cfm?event=questions....

2004-08-02

PASS PRO - Copyright 2004 Inmarkets Ltd - Licensed to Raymond chen


A. 7.90%. B. 7.98%. C. 8.00%. D. 8.03%. 248.

1/150 6

Peter Roche is due to receive $4,000 one year from now, which he can invest at an interest rate of 7 percent for a further ten years. The value of this payment at the end of this period will be CLOSEST to: A. $6,800. B. $7,080. C. $7,869. D. $8,419. 249. A bank is offering loans with monthly interest payments based on a stated annual interest rate of 9 percent. The effective annual rate of these loans is CLOSEST to: A. 9.20%. B. 9.31%. C. 9.34%. D. 9.38%. 250. If a cash flow of $15,000 in two years' time has a PV of $12,000, the annual percentage rate, assuming continuous compounding is CLOSEST to: A. 11.16%. B. 11.44%. C. 11.69%. D. 11.80%. 251. The inflation rate in an economy where the prices are doubling ever eight years is CLOSEST to: A. 6%. B. 9%. C. 13%. D. 25%. 252. What is the standard deviation of the following data?

http://64.106.152.240/inmarkets/passpro/index.cfm?event=questions....

2004-08-02

PASS PRO - Copyright 2004 Inmarkets Ltd - Licensed to Raymond chen

2/150 6

Probability Value 40% -4 25% 8 35% 15 A. 4.80. B. 8.32. C. 23.08. D. 69.23. 253. Consider a stock whose 10-day volatility has been estimated as 1.8%. Assuming that the returns from the stock are uncorrelated over time, the volatility over 250 days will be CLOSEST to: A. 0.36%. B. 9.00%. C. 28.46%. D. 45.00%. 254. Assets A and B have a correlation of +1. The volatility of asset A is 15%. The volatility of asset B is 25%. The daily return on asset A is observed at 1.50%. What is the most likely return on asset B? A. -1.50%. B. 1.50%. C. 2.50%. D. Cannot the predicted. 255. Which of the following statements about the correlation coefficient are FALSE? I. Correlation coefficient ranges between -1 and +1. II. Correlation coefficient is a measure of non-linear relationship between two random variables. III. Correlation coefficient can be calculated by scaling the covariance between two variables. IV. If the correlation coefficient between two random variables is zero they will be independent. A. I and II. B. I and III. C. II and IV. D. I, III and IV. 256. What is the standard deviation of the following data?

http://64.106.152.240/inmarkets/passpro/index.cfm?event=questions....

2004-08-02

PASS PRO - Copyright 2004 Inmarkets Ltd - Licensed to Raymond chen

3/150 6

Probability Value 15% -6 40% 6 45% 12 A. 3.51. B. 6.08. C. 12.33. D. 36.98. 257. What is the mean for the following probability data? Probability Value 30% -10 40% 5 30% 25 A. 1.15. B. 2.17. C. 3.34. D. 6.50. 258. Which of the following statements are TRUE? I. Correlation coefficient ranges between 0 and +1. II. Correlation coefficient is a measure of non-linear relationship between two random variables. III. Correlation coefficient can be calculated by scaling the covariance between two variables. IV. If two random variables are independent they will have a correlation coefficient of zero. A. I and II. B. I and IV. C. II and III. D. III and IV. 259. What is the mean for the following probability data? Probability Value 25% -8 40% 12 35% 24 A. 3.73. B. 5.73. C. 11.20.

http://64.106.152.240/inmarkets/passpro/index.cfm?event=questions....

2004-08-02

PASS PRO - Copyright 2004 Inmarkets Ltd - Licensed to Raymond chen


D. 20.10. 260.

4/150 6

The covariance between the return from two securities is 5 and the correlation between them is 0.5. If the variance of the first return is 8, the variance of the second return will be CLOSEST to: A. 1.25. B. 1.56. C. 3.54. D. 12.50. 261. What is the standard deviation of the following data? Probability Value 25% -8 40% 12 35% 24 A. 7.07. B. 12.24. C. 49.92. D. 149.76. 262. Which of the following statements about the correlation coefficient are FALSE? I. Correlation coefficient ranges between -1 and +1. II. Correlation coefficient can be calculated by scaling the covariance between two variables. III. If the correlation coefficient between two random variables is zero they will be independent. IV. Correlation coefficient is a measure of the non-linear relationship between a dependent and an independent variable. A. I and II. B. I and IV. C. II and III. D. III and IV. 263. What is the standard deviation of the following data? Probability Value 45% -4 40% 8 15% 25 A. 5.77.

http://64.106.152.240/inmarkets/passpro/index.cfm?event=questions....

2004-08-02

PASS PRO - Copyright 2004 Inmarkets Ltd - Licensed to Raymond chen


B. 10.00. C. 33.34. D. 100.03. 264. Which of the following statements are TRUE?

5/150 6

I. Correlation coefficient ranges between -1 and +1. II. Correlation coefficient is a measure of linear relationship between two random variables. III. Correlation coefficient can be calculated by scaling the covariance between two variables. IV. If the correlation coefficient between two random variables is zero they will be independent. A. I and II. B. III and IV. C. I, II and III. D. I, II, III and IV. 265. What is the mean for the following probability data? Probability Value 25% -8 40% 12 35% 24 A. 3.73. B. 5.73. C. 11.20. D. 20.10. 266. What is the standard deviation of the following data? Probability Value 15% -6 40% 6 45% 12 A. 3.51. B. 6.08. C. 12.33. D. 36.98. 267. Which of the following statements about the correlation coefficient are FALSE? I. Correlation coefficient ranges between 0 and +1.

http://64.106.152.240/inmarkets/passpro/index.cfm?event=questions....

2004-08-02

PASS PRO - Copyright 2004 Inmarkets Ltd - Licensed to Raymond chen

6/150 6

II. If two random variables have a correlation coefficient of zero they are independent. III. Correlation coefficient can be calculated by scaling the covariance between two variables. IV. Correlation coefficient is a measure of the linear relationship between a dependent and an independent variable. A. I and II. B. I and IV. C. II and III. D. III and IV. 268. Consider a stock whose 10-day volatility has been estimated as 1.4%. Assuming that the returns from the stock are uncorrelated over time, the volatility over 23 days will be CLOSEST to: A. 0.92%. B. 2.12%. C. 3.22%. D. 6.71%. 269. Which of the following statements about the correlation coefficient are FALSE? I. Correlation coefficient ranges between 0 and +1. II. If two random variables have a correlation coefficient of zero they are independent. III. Correlation coefficient can be calculated by scaling the covariance between two variables. IV. Correlation coefficient is a measure of the linear relationship between a dependent and an independent variable. A. I and II. B. I and IV. C. II and III. D. III and IV. 270. Assets A and B have a correlation of +1. The volatility of asset A is 15%. The volatility of asset B is 25%. The daily return on asset A is observed at 1.50%. What is the most likely return on asset B? A. -1.50%. B. 1.50%. C. 2.50%. D. Cannot the predicted. 271. Consider a stock whose 7-day volatility has been estimated as 1.05%. Assuming that the

http://64.106.152.240/inmarkets/passpro/index.cfm?event=questions....

2004-08-02

PASS PRO - Copyright 2004 Inmarkets Ltd - Licensed to Raymond chen

7/150 6

returns from the stock are uncorrelated over time, the volatility over 30 days will be CLOSEST to: A. 0.51%. B. 2.17%. C. 4.50%. D. 5.75%. 272. What is the mean for the following probability data? Probability Value 60% -2 20% 10 20% 20 A. 1.60. B. 2.80. C. 3.60. D. 4.80. 273. What is the standard deviation of the following data? Probability Value 40% -4 25% 8 35% 15 A. 4.80. B. 8.32. C. 23.08. D. 69.23. 274. What is the standard deviation of the following data? Probability Value 60% -2 20% 10 20% 20 A. 5.14. B. 8.91. C. 26.45. D. 79.36. 275.

http://64.106.152.240/inmarkets/passpro/index.cfm?event=questions....

2004-08-02

PASS PRO - Copyright 2004 Inmarkets Ltd - Licensed to Raymond chen

8/150 6

Consider a stock whose 7-day volatility has been estimated as 1.05%. Assuming that the returns from the stock are uncorrelated over time, the volatility over 30 days will be CLOSEST to: A. 0.51%. B. 2.17%. C. 4.50%. D. 5.75%. 276. What is the standard deviation of the following data? Probability Value 30% -15 40% 5 30% 25 A. 8.94. B. 15.49. C. 80.00. D. 240.00. 277. Which of the following investments has the highest co-efficient of variation? A. Expected return = 18%; variance of return = 0.004. B. Expected return = 25%; variance of return = 0.005. C. Expected return = 12%; standard deviation of return = 4%. D. Expected return = 8%; standard deviation of return = 2.5%. 278. Which of the following investments has the highest co-efficient of variation? A. An investment with an expected return of 12% and a variance of of return of 0.005. B. An investment with an expected return of 10% and a variance of of return of 0.002. C. An investment with an expected return of 15% and standard deviation of return of 3%. D. An investment with an expected return of 6% and standard deviation of return of 5%. 279. An analyst regresses the returns of 18 stocks in a stock market and finds that the best fitting line is: Return = 3.75% + 6% x Beta If the standard error of the estimate is 4% and the standard error of the coefficient of Beta is 3%, the test statistic for the coefficient is CLOSEST to:

http://64.106.152.240/inmarkets/passpro/index.cfm?event=questions....

2004-08-02

PASS PRO - Copyright 2004 Inmarkets Ltd - Licensed to Raymond chen


A. 0.94. B. 1.43. C. 1.50. D. 2.00. 280. Which of the following investments has the highest co-efficient of variation?

9/150 6

A. An investment with an expected return of 24% and a variance of of return of 0.005. B. An investment with an expected return of 18% and a variance of of return of 0.002. C. An investment with an expected return of 15% and standard deviation of return of 5%. D. An investment with an expected return of 25% and standard deviation of return of 8%. 281. If the mean P/E of 40 stocks in a certain industrial sector is 12 and the sample standard deviation is 4, standard error of the mean is CLOSEST to: A. 0.10. B. 0.32. C. 0.63. D. 1.58. 282. An analyst regresses the returns of 15 stocks in a stock market and finds that the best fitting line is: Return = 13% + 15% x Beta If the standard error of the estimate is 6% and the standard error of the coefficient of Beta is 9%, the test statistic for the coefficient is CLOSEST to: A. 1.67. B. 2.17. C. 2.38. D. 2.50. 283. Which of the following investments has the highest co-efficient of variation? A. Expected return = 18%; variance of return = 0.004. B. Expected return = 25%; variance of return = 0.005. C. Expected return = 12%; standard deviation of return = 4%. D. Expected return = 8%; standard deviation of return = 2.5%.

http://64.106.152.240/inmarkets/passpro/index.cfm?event=questions....

2004-08-02

PASS PRO - Copyright 2004 Inmarkets Ltd - Licensed to Raymond chen

0/150 7

284. An analyst regresses the returns of 50 stocks in a stock market and finds that the best fitting line is: Return = 8% + 12.25% x Beta If the standard error of the estimate is 10% and the standard error of the coefficient of Beta is 8%, the test statistic for the coefficient is CLOSEST to: A. 0.80. B. 1.16. C. 1.23. D. 1.53. 285. Which of the following investments has the highest co-efficient of variation? A. An investment with an expected return of 24% and a variance of of return of 0.005. B. An investment with an expected return of 18% and a variance of of return of 0.002. C. An investment with an expected return of 15% and standard deviation of return of 5%. D. An investment with an expected return of 25% and standard deviation of return of 8%. 286. If the mean P/E of 30 stocks in a certain industrial sector is 18 and the sample standard deviation is 3.5, standard error of the mean is CLOSEST to: A. 0.12. B. 0.34. C. 0.64. D. 1.56. 287. An analyst regresses the returns of 15 stocks in a stock market and finds that the best fitting line is: Return = 13% + 15% x Beta If the standard error of the estimate is 6% and the standard error of the coefficient of Beta is 9%, the test statistic for the coefficient is CLOSEST to: A. 1.67. B. 2.17. C. 2.38. D. 2.50. 288.

http://64.106.152.240/inmarkets/passpro/index.cfm?event=questions....

2004-08-02

PASS PRO - Copyright 2004 Inmarkets Ltd - Licensed to Raymond chen

1/150 7

An analyst regresses the returns of 50 stocks in a stock market and finds that the best fitting line is: Return = 8% + 9% x Beta If the standard error of the estimate is 5.5% and the standard error of the coefficient of Beta is 3.25%, the test statistic for the coefficient is CLOSEST to: A. 1.45. B. 1.64. C. 1.55. D. 2.77. 289. An analyst collects the data for interest rate expectations. The mean expected rate is 2.5 percent, the lowest expectation is 1 percent and the highest expectation is 5 percent. This distribution is: A. sparse. B. normal. C. skewed. D. abnormal. 290. An analyst is studying a stock that is currently trading at $35. The analyst estimates that there is 33 percent probability that the stock will trade at $50 after one year, a 20 percent probability that the stock will trade at $42, and a 47 percent chance that the stock will trade at $20. What is the implied volatility of this stock price? A. 13%. B. 24%. C. 31%. D. 39%. 291. An analyst regresses the returns of 16 stocks in a stock market and finds that the best fitting line is: Return = 6.5% + 11.8% x Beta If the standard error of the estimate is 4% and the standard error of the coefficient of Beta is 3%, the test statistic for the coefficient is CLOSEST to: A. 1.63. B. 2.95. C. 2.80. D. 3.93.

http://64.106.152.240/inmarkets/passpro/index.cfm?event=questions....

2004-08-02

PASS PRO - Copyright 2004 Inmarkets Ltd - Licensed to Raymond chen

2/150 7

292. If the mean P/E of 60 stocks in a certain industrial sector is 18 and the sample standard deviation is 7, standard error of the mean is CLOSEST to: A. 0.12. B. 0.34. C. 0.90. D. 1.11. 293. If the mean P/E of 50 stocks in a certain industrial sector is ten and the sample standard deviation is six, standard error of the mean is CLOSEST to: A. 0.12. B. 0.35. C. 0.85. D. 1.18. 294. An analyst regresses the returns of 18 stocks in a stock market and finds that the best fitting line is: Return = 3.75% + 6% x Beta If the standard error of the estimate is 4% and the standard error of the coefficient of Beta is 3%, the test statistic for the coefficient is CLOSEST to: A. 0.94. B. 1.43. C. 1.50. D. 2.00. 295. Which of the following investment has the highest co-efficient of variation? A. An investment with an expected return of 19% and a variance of return of 0.005. B. An investment with an expected return of 15% and a variance of return of 0.002. C. An investment with an expected return of 10% and standard deviation of return of 3%. D. An investment with an expected return of 3% and standard deviation of return of 1%. 296. An analyst regresses the returns of 40 stocks in a stock market and finds that the best fitting line is: Return = 6% + 13.6% x beta

http://64.106.152.240/inmarkets/passpro/index.cfm?event=questions....

2004-08-02

PASS PRO - Copyright 2004 Inmarkets Ltd - Licensed to Raymond chen

3/150 7

If the standard error of the estimate is 3% and the standard error of the coefficient of beta is 5%, the test statistic for the coefficient is CLOSEST to: A. 2.00. B. 2.72. C. 4.31. D. 4.53. 297. Which of the following investments has the highest co-efficient of variation? A. An investment with an expected return of 10% and a variance of of return of 0.002. B. An investment with an expected return of 15% and a variance of of return of 0.003. C. An investment with an expected return of 8% and standard deviation of return of 3%. D. An investment with an expected return of 5% and standard deviation of return of 1%. 298. An analyst regresses the returns of 40 stocks in a stock market and finds that the best fitting line is: Return = 6% + 13.6% x beta If the standard error of the estimate is 3% and the standard error of the coefficient of beta is 5%, the test statistic for the coefficient is CLOSEST to: A. 2.00. B. 2.72. C. 4.31. D. 4.53. 299. Which of the following statements are TRUE? I. Lowering the level of significance reduces the probability of Type I error. II. Lowering the level of significance increases the probability of Type I error. III. Minimizing the probability of Type II error minimizes the power of the test. IV. Minimizing the probability of Type II error maximizes the power of the test. A. I and III. B. I and IV. C. II and III. D. II and IV. 300. Which of the following statements is TRUE? A. Statistical significance does not suggest economic significance.

http://64.106.152.240/inmarkets/passpro/index.cfm?event=questions....

2004-08-02

PASS PRO - Copyright 2004 Inmarkets Ltd - Licensed to Raymond chen


B. Economic significance and statistical significance are not related. C. A result that is statistically significant may not be economically significant. D. Economic significance statistical significance refer to the same evidence. 301.

4/150 7

An analyst wants to test whether the standard deviation of return from pharmaceutical stocks is lower than 0.2. For this purpose, he obtains the following data from a sample of 30 pharmaceutical stocks. Mean return from pharmaceutical stocks = 8%. Standard deviation of return from pharmaceutical stocks = 12%. Mean return from the market = 12%. Standard deviation of return from the market = 16%. Based on this information, we can say that at a 0.05 significance level the standard deviation of pharmaceutical stocks is: A. lower than 20%. B. higher than 20%. C. not lower than 20%. D. none of the above. 302. Which of the following statements are TRUE? I. Lowering the level of significance reduces the probability of Type I error. II. Type I error occurs when the null hypothesis is rejected when it is actually true. III. Type I error occurs when the null hypothesis is accepted when it is actually false. IV. Type I error occurs when the alternate hypothesis is wrongly accepted. A. I and II. B. I and IV. C. II and III. D. I, II and IV. 303. An analyst is testing a hypothesis that firms with a P/B ratio 25 percent below average tend to outperform the market. If the test statistic calculated from the market data is more than the critical value of the statistic, the analyst should: A. reject the null hypothesis. B. accept the null hypothesis. C. reject the results of the hypothesis test. D. none of the above. 304. An analyst wants to test whether the return from pharmaceutical stocks is less volatile than that of the overall market. For this purpose, he obtains the following data from a sample of 21 pharmaceutical stocks and a sample of 41 stocks that are representative of the market. Mean return from pharmaceutical stocks = 8% Standard deviation of return from pharmaceutical stocks = 10% Mean return from market stocks = 12% Standard deviation of return from market stocks = 13% What is the value of the test statistic for this test? A. 1.30.

http://64.106.152.240/inmarkets/passpro/index.cfm?event=questions....

2004-08-02

PASS PRO - Copyright 2004 Inmarkets Ltd - Licensed to Raymond chen


B. 1.50. C. 1.69. D. 2.25. 305.

5/150 7

An analyst wants to test whether the variance of return from telecom stocks is higher than 0.04. For this purpose, he obtains the following data from a sample of 51 telecom stocks. Mean return from telecom stocks = 15% Standard deviation of return from telecom stocks = 24% Mean return from market = 12% Standard deviation of return from market = 13% Based on this information and a 0.05 significance level: A. we can say that the variance of telecom firms is lower than 0.04. B. we can say that the variance of telecom firms is higher than 0.04. C. we cannot say that the variance of telecom firms is lower than 0.04. D. none of the above. 306. Which of the following is NOT a step in the hypothesis testing process? A. State a hypothesis. B. Identify the population mean and probability distribution. C. Specify the significance level. D. Make the investment/economic decision based on the results of the test. 307. An analyst wants to test whether the standard deviation of return from pharmaceutical stocks is lower than 0.2. For this purpose, he obtains the following data from a sample of 30 pharmaceutical stocks. Mean return from pharmaceutical stocks = 8%. Standard deviation of return from pharmaceutical stocks = 12%. Mean return from the market = 12%. Standard deviation of return from the market = 16%. What is the critical value of the statistic for this test at a 0.05 level of significance? A. 17.71. B. 28.81. C. 42.56. D. 45.72. 308. An analyst wants to test whether the return from transportation sector stocks is different from that of the utility stocks. For this purpose, he obtains the following data from a sample of 21 transportation stocks and a sample of 41 utility stocks. Mean return from transportation stocks = 15%. Standard deviation of return from transportation stocks = 12%. Mean return from utility stocks = 12%. Standard deviation of return from utility stocks = 13%. Based on this information and using a 0.05 significance level: A. there is no evidence for a difference between the means. B. there is sufficient evidence for a difference between the means.

http://64.106.152.240/inmarkets/passpro/index.cfm?event=questions....

2004-08-02

PASS PRO - Copyright 2004 Inmarkets Ltd - Licensed to Raymond chen

6/150 7

C. there is insufficient evidence for a difference between the means. D. none of the above. 309. An analyst wants to test whether the standard deviation of return from pharmaceutical stocks is lower than 0.2. For this purpose, he obtains the following data from a sample of 40 pharmaceutical stocks. Mean return from pharmaceutical stocks = 14% Standard deviation of return from pharmaceutical stocks = 11% Mean return from market = 12% Standard deviation of return from market = 13% What is the value of the test statistic for this test? A. 11.80. B. 12.10. C. 21.45. D. 22.00. 310. Using a sample size of 15 observations, an analyst determines that the standard deviation of the returns from a stock is 10 percent. Using a 0.05 significance level, the analyst: A. can conclude that the standard deviation of returns is same as 15%. B. can conclude that the standard deviation of returns is different from 15%. C. cannot conclude that the standard deviation of returns is different from 15%. D. none of the above. 311. An analyst wants to test whether the variance of return from oil and gas stocks is higher than 0.045. For this purpose, he obtains the following data from a sample of 61 oil and gas stocks. Mean return from oil and gas stocks = 16%. Standard deviation of return from oil and gas stocks = 22% Mean return from the market = 12%. Standard deviation of return from the market = 14% Based on this information, we can say that at a 0.05 significance level the variance of oil and gas firms is: A. lower than 0.045. B. higher than 0.045. C. not higher than 0.045. D. none of the above. 312. An analyst wants to test whether the variance of return from pharmaceutical stocks different from that of the overall market. For this purpose, he obtains the following data from a sample of 21 pharmaceutical stocks and a sample of 41 stocks that are representative of the market. Mean return from pharmaceutical stocks = 8%. Standard deviation of return from pharmaceutical stocks = 9.2%. Mean return from the market stocks = 12%. Standard deviation of return from the market stocks = 13%. What is the critical value of the statistic for this test at a 0.05 level of significance? A. 1.84.

http://64.106.152.240/inmarkets/passpro/index.cfm?event=questions....

2004-08-02

PASS PRO - Copyright 2004 Inmarkets Ltd - Licensed to Raymond chen


B. 1.99. C. 2.07. D. 2.29. 313.

7/150 7

Which of the following statements are NOT true? I. Type I error occurs when the alternate hypothesis is wrongly accepted. II. Minimizing the probability of Type II error maximizes the power of the test. III. Type II error occurs when the null hypothesis is rejected when it is actually true. IV. Type I error occurs when the null hypothesis is not rejected when it is actually false. A. I and II. B. I and III. C. III and IV. D. I, II and IV. 314. An analyst collects a sample of 50 P/E ratios of stocks that are representative of the market. The mean P/E of these stocks is 20 and the standard deviation is 8.5. What is the 95 percent confidence interval for the mean P/E of stocks in this market? A. 17.21 to 22.79. B. 17.64 to 22.36. C. 18.02 to 21.98. D. 18.31 to 21.69. 315. An analyst has calculated that the variance of returns of stock A based on 20 observations is 0.005 and the variance of returns of stock B based on 15 observations is 0.012. Using a 0.05 level of significance, the analyst: A. can conclude that the variance of Stock A is the same as that of Stock B. B. can conclude that the variance of Stock A is different from that of Stock B. C. cannot conclude that the variance of Stock A is different from that of Stock B. D. none of the above. 316. Which of the following statements are NOT true? I. Type I error occurs when the null hypothesis is not rejected when it is actually false. II. Type II error occurs when the null hypothesis is rejected when it is actually true. III. Type I error occurs when the alternate hypothesis is wrongly accepted. IV. Minimizing the probability of Type II error maximizes the power of the test. A. I and II. B. I and III. C. II and IV.

http://64.106.152.240/inmarkets/passpro/index.cfm?event=questions....

2004-08-02

PASS PRO - Copyright 2004 Inmarkets Ltd - Licensed to Raymond chen

8/150 7

D. I, II and IV. 317. Using a sample size of 101 observations, an analyst determines that the standard deviation of the returns from a stock is 10 percent. Using a 0.05 significance level, the analyst: A. can conclude that the standard deviation of returns is lower than 18%. B. cannot conclude that the standard deviation of returns is lower than 18%. C. can conclude that the standard deviation of returns is not lower than 18%. D. none of the above. 318. An analyst believes that hedge funds have significantly (using a significance level of 0.05) outperformed the S&P 500 over the past five years. So she picks a random group of 15 hedge funds and finds that their mean return over this period is 85 percent and their standard deviation is 45 percent. During the same period the S&P 500 has risen by 75 percent. The critical value of the t-statistic for this study is: A. 1.21. B. 1.65. C. 1.76. D. 1.96. 319. An analyst believes that a group of 16 active portfolio managers have achieved a significantly higher performance (using a significance level of 0.05) than the average for all portfolio managers over a certain period. Over the period of the study, the active portfolio managers achieved a mean return of 15 percent. Over the same period the mean return for all portfolio managers was 12 percent and their standard deviation was 8 percent. What is the null hypothesis in this study? A. The performance of active portfolio managers is higher than or equal to the average for all portfolio managers. B. The performance of active portfolio managers is higher than the average for all portfolio managers. C. The performance of active portfolio managers is lower than or equal to the average for all portfolio managers. D. The performance of active portfolio managers is lower than the average for all portfolio managers. 320. An analyst collects a sample of 40 P/E ratios of stocks that are representative of the market. The mean P/E of these stocks is 18 and the standard error of this estimate 7.25. What is the 95 percent confidence interval for the mean P/E of stocks in this market? A. 3.79 to 32.21. B. 6.04 to 29.96.

http://64.106.152.240/inmarkets/passpro/index.cfm?event=questions....

2004-08-02

PASS PRO - Copyright 2004 Inmarkets Ltd - Licensed to Raymond chen


C. 1.18 to 34.82. D. 7.78 to 28.22. 321.

9/150 7

An analyst wants to test whether the mean spending by tourists coming to a holiday resort is greater than $2,000 with a 1 percent level of significance. He finds that the average spending by 16 tourists is $2,200 and the standard deviation of the population is $400. This study is a: A. one-tailed test. B. two-tailed test. C. four-tailed test. D. alternate test. 322. An analyst wants to test whether the return from transportation sector stocks is different from that of the utility stocks. For this purpose, he obtains the following data from a sample of 21 transportation stocks and a sample of 41 utility stocks. Mean return from transportation stocks = 9%. Standard deviation of return from transportation stocks = 13%. Mean return from utility stocks = 12%. Standard deviation of return from utility stocks = 15%. Based on this information and using a 0.05 significance level: A. there is no evidence for a difference between the means. B. there is sufficient evidence for a difference between the means. C. there is insufficient evidence for a difference between the means. D. none of the above. 323. An analyst believes that a group of 16 active portfolio managers have achieved a significantly higher performance (using a significance level of 0.05) than the average for all portfolio managers over a certain period. Over the period of the study, the active portfolio managers achieved a mean return of 15 percent. Over the same period the mean return for all portfolio managers was 12 percent and their standard deviation was 8 percent. What is the null hypothesis in this study? A. The performance of active portfolio managers is higher than or equal to the average for all portfolio managers. B. The performance of active portfolio managers is higher than the average for all portfolio managers. C. The performance of active portfolio managers is lower than or equal to the average for all portfolio managers. D. The performance of active portfolio managers is lower than the average for all portfolio managers. 324. A fund manager is using a statistical package to perform a linear regression between the number of securities that she uses to replicate an index and the 95 percent shortfall risk. The original data and

http://64.106.152.240/inmarkets/passpro/index.cfm?event=questions....

2004-08-02

PASS PRO - Copyright 2004 Inmarkets Ltd - Licensed to Raymond chen

0/150 8

intermediate statistics are shown on the right. The value of coefficient of determination for this regression is CLOSEST to:

A. 0.062. B. 0.120. C. 0.880. D. 0.938.

325. An analyst is using a statistical package to perform a linear regression between the features offered in a product and its sales volume at a constant price. The original data and intermediate statistics are shown on the right. The standard error of the estimate from this regression is CLOSEST to:

A. 53. B. 59. C. 2,852. D. 3,423.

326. If the correlation coefficient of a linear regression is 0.6, the percentage of variation of the dependent variable that is not explained by the independent variable is CLOSEST to: A. 36%. B. 40%. C. 60%.

http://64.106.152.240/inmarkets/passpro/index.cfm?event=questions....

2004-08-02

PASS PRO - Copyright 2004 Inmarkets Ltd - Licensed to Raymond chen


D. 64%. 327.

1/150 8

If the correlation coefficient of a linear regression is 0.85, the percentage of variation of the dependent variable that is not explained by the independent variable is CLOSEST to: A. 15%. B. 28%. C. 72%. D. 85%. 328. An analyst is using a statistical package to perform a linear regression between price of a new semiconductor cleaning product and its price. The original data and intermediate statistics are shown on the right. The value of the slope coefficient for this regression is CLOSEST to:

A. -24.81. B. -14.59. C. 24.83. D. 42.21.

329. If the correlation coefficient of a linear regression is 0.33, the percentage of variation of the dependent variable that is not explained by the independent variable is CLOSEST to: A. 11%. B. 33%. C. 67%. D. 89%. 330. A fund manager is using a statistical package to perform a linear regression between the number of securities that she uses to replicate an index and the 95 percent shortfall risk. The original data and intermediate statistics are shown on the right. The value of the slope coefficient for this regression is

http://64.106.152.240/inmarkets/passpro/index.cfm?event=questions....

2004-08-02

PASS PRO - Copyright 2004 Inmarkets Ltd - Licensed to Raymond chen

2/150 8

CLOSEST to:

A. -8.51. B. -2.47. C. 3.68. D. 5.71.

331. An analyst is using a statistical package to perform a linear regression between the features offered in a product and its sales volume at a constant price. The original data and intermediate statistics are shown on the right. The correlation between the price and volume is CLOSEST to:

A. 0.079. B. 0.151. C. 0.849. D. 0.921.

332. A fund manager is using a statistical package to perform a linear regression between the number of securities that she uses to replicate an index and the 95 percent shortfall risk. The original data and intermediate statistics are shown on the right. The value of coefficient of determination for this regression is CLOSEST to:

http://64.106.152.240/inmarkets/passpro/index.cfm?event=questions....

2004-08-02

PASS PRO - Copyright 2004 Inmarkets Ltd - Licensed to Raymond chen

3/150 8

A. 0.062. B. 0.120. C. 0.880. D. 0.938.

333. An analyst is using a statistical package to perform a linear regression between the features offered in a product and its sales volume at a constant price. The original data and intermediate statistics are shown on the right. The value of coefficient of determination for this regression is CLOSEST to:

A. 0.079. B. 0.151. C. 0.849. D. 0.921.

334. A fund manager is using a statistical package to perform a linear regression between the number of securities that she uses to replicate an index and the 95 percent shortfall risk. The original data and intermediate statistics are shown on the right. The standard error of the estimate from this regression is CLOSEST to:

http://64.106.152.240/inmarkets/passpro/index.cfm?event=questions....

2004-08-02

PASS PRO - Copyright 2004 Inmarkets Ltd - Licensed to Raymond chen

4/150 8

A. 4. B. 17. C. 21. D. 26.

335. A fund manager is using a statistical package to perform a linear regression between the number of securities that she uses to replicate an index and the 95 percent shortfall risk. The original data and intermediate statistics are shown on the right. The standard error of the estimate from this regression is CLOSEST to:

A. 4. B. 17. C. 21. D. 26.

336. An analyst is using a statistical package to perform a linear regression between the risk and return from securities in an emerging market country. The original data and intermediate statistics are shown on the right. The correlation between the price and volume is CLOSEST to:

http://64.106.152.240/inmarkets/passpro/index.cfm?event=questions....

2004-08-02

PASS PRO - Copyright 2004 Inmarkets Ltd - Licensed to Raymond chen

5/150 8

A. 0.043. B. 0.084. C. 0.916. D. 0.957.

337. An analyst is using a statistical package to perform a linear regression between price of a new semiconductor cleaning product and its price. The original data and intermediate statistics are shown on the right. The value of the intercept for this regression is CLOSEST to:

A. 0. B. 258.51. C. 843.96. D. 995.77.

338. If the average return of 60 stocks in a certain industrial sector is 25 percent and the sample standard deviation is 15 percent, standard error of the mean is CLOSEST to: A. 1.76. B. 1.82. C. 1.88.

http://64.106.152.240/inmarkets/passpro/index.cfm?event=questions....

2004-08-02

PASS PRO - Copyright 2004 Inmarkets Ltd - Licensed to Raymond chen


D. 1.94. 339. An analyst is using a statistical package to perform a linear regression between the risk and return from securities in an emerging market country. The original data and intermediate statistics are shown on the right. The value of coefficient of determination for this regression is CLOSEST to:

6/150 8

A. 0.043. B. 0.084. C. 0.916. D. 0.957.

340. An analyst is using a statistical package to perform a linear regression between price of a new semiconductor cleaning product and its price. The original data and intermediate statistics are shown on the right. The standard error of the estimate from this regression is CLOSEST to:

A. 55. B. 60. C. 3,009. D. 3,611.

341. If the average return of 60 stocks in a certain industrial sector is 25 percent and the sample standard deviation is 15 percent, standard error of the mean is CLOSEST to: A. 1.76. B. 1.82.

http://64.106.152.240/inmarkets/passpro/index.cfm?event=questions....

2004-08-02

PASS PRO - Copyright 2004 Inmarkets Ltd - Licensed to Raymond chen

7/150 8

C. 1.88. D. 1.94. 342. An analyst is using a statistical package to perform a linear regression between the risk and return from securities in an emerging market country. The original data and intermediate statistics are shown on the right. The standard error of the estimate from this regression is CLOSEST to:

A. 0.225. B. 0.247. C. 0.051. D. 0.061.

343. If the correlation coefficient of a linear regression is 0.33, the percentage of variation of the dependent variable that is not explained by the independent variable is CLOSEST to: A. 11%. B. 33%. C. 67%. D. 89%. 344. An analyst is using a statistical package to perform a linear regression between the risk and return from securities in an emerging market country. The original data and intermediate statistics are shown on the right. The value of the slope coefficient for this regression is CLOSEST to:

A. 0.37. B. 1.24.

http://64.106.152.240/inmarkets/passpro/index.cfm?event=questions....

2004-08-02

PASS PRO - Copyright 2004 Inmarkets Ltd - Licensed to Raymond chen


C. 2.09. D. 7.11.

8/150 8

345. An analyst is using a statistical package to perform a linear regression between price of a new semiconductor cleaning product and its price. The original data and intermediate statistics are shown on the right. The value of the intercept for this regression is CLOSEST to:

A. 0. B. 258.51. C. 843.96. D. 995.77.

346. A fund manager is using a statistical package to perform a linear regression between the number of securities that she uses to replicate an index and the 95 percent shortfall risk. The original data and intermediate statistics are shown on the right. The value of the slope coefficient for this regression is CLOSEST to:

http://64.106.152.240/inmarkets/passpro/index.cfm?event=questions....

2004-08-02

PASS PRO - Copyright 2004 Inmarkets Ltd - Licensed to Raymond chen

9/150 8

A. -8.51. B. -2.47. C. 3.68. D. 5.71.

347. An analyst is using a statistical package to perform a linear regression between the features offered in a product and its sales volume at a constant price. The original data and intermediate statistics are shown on the right. The value of the intercept for this regression is CLOSEST to:

A. 0. B. 221. C. 441. D. 690.

348. Which of the following test statistics is most appropriate for conducting the hypothesis test given below? H0: variance A = variance B; where the test is based on two random independent samples from two normally distributed populations. A. t-statistic. B. z-statistic.

http://64.106.152.240/inmarkets/passpro/index.cfm?event=questions....

2004-08-02

PASS PRO - Copyright 2004 Inmarkets Ltd - Licensed to Raymond chen

0/150 9

C. F-statistic. D. chi-square. 349. Stock A has a standard deviation of 0.30 and Stock B has a standard deviation of 0.45. If the covariance is 0.1013 what is the correlation coefficient? A. 0.01. B. 0.23. C. 0.34. D. 0.75. 350. The variance of the returns from stock A is 0.018 and that of the market is 0.025. If the covariance between the stock and the index is -0.002, their correlation coefficient is CLOSEST to: A. -0.23. B. -0.11. C. -0.09. D. -0.08. 351. Stock A has a standard deviation of 0.30 and Stock B has a standard deviation of 0.45. If their correlation coefficient is 0.75 what is their covariance? A. 0.1013. B. 0.1350. C. 0.2250. D. 0.3375. 352. A portfolio contains two perfectly negatively correlated investments with volatilities of 3 percent and 10 percent. The proportion of these two securities that would lead to the lowest risk are: A. 8% and 92%. B. 92% and 8%. C. 23% and 77%. D. 77% and 23%. 353. A portfolio contains two perfectly negatively correlated investments with volatilities of 3 percent and 10 percent. The proportion of these two securities that would lead to the lowest risk are:

http://64.106.152.240/inmarkets/passpro/index.cfm?event=questions....

2004-08-02

PASS PRO - Copyright 2004 Inmarkets Ltd - Licensed to Raymond chen


A. 8% and 92%. B. 92% and 8%. C. 23% and 77%. D. 77% and 23%. 354.

1/150 9

Stock A has a standard deviation of 0.30 and Stock B has a standard deviation of 0.45. If their correlation coefficient is 0.75 what is their covariance? A. 0.1013. B. 0.1350. C. 0.2250. D. 0.3375. 355. The lowest level of risk, measured by volatility, of a portfolio containing two perfectly negatively correlated investments with volatilities of 5 percent and 7 percent is: A. 0%. B. 5%. C. 7%. D. 12%. 356. If Security A and Security B are positively correlated, and the price of Security A increases, the price of Security B: A. will increase. B. will decrease. C. is most likely to increase than to decrease. D. may decrease or remain unchanged, but will not increase. 357. The lowest level of risk, measured by volatility, of a portfolio containing two perfectly negatively correlated investments with volatilities of 3 percent and 10 percent respectively is: A. 0% B. 3% C. 10% D. 13% 358. Stock A has a standard deviation of 0.30 and Stock B has a standard deviation of 0.45. If the

http://64.106.152.240/inmarkets/passpro/index.cfm?event=questions....

2004-08-02

PASS PRO - Copyright 2004 Inmarkets Ltd - Licensed to Raymond chen

2/150 9

covariance is 0.1013 what is the correlation coefficient? A. 0.01. B. 0.23. C. 0.34. D. 0.75. 359. A positive covariance means that: A. there is no relationship between asset returns. B. the correlation coefficient is zero. C. asset returns move in the opposite direction. D. asset returns move in the same direction. 360. Which of the following are no-arbitrage models for fixed income valuation? I. Vasicek. II. Black Scholes. III. Hull and White. IV. Heath, Jarrow and Morton. A. I and II. B. III and IV. C. I, III and IV. D. II, III and IV. 361. Which of the following are no-arbitrage models for fixed income valuation? I. Vasicek. II. Black Scholes. III. Hull and White. IV. Heath, Jarrow and Morton. A. I and II. B. III and IV. C. I, III and IV. D. II, III and IV. 362. Given two variables X and Y that are normally distributed, what of the following will also be normally distributed? A. X * Y.

http://64.106.152.240/inmarkets/passpro/index.cfm?event=questions....

2004-08-02

PASS PRO - Copyright 2004 Inmarkets Ltd - Licensed to Raymond chen


B. X + Y. C. log(X) + log(Y). D. exp(X) + exp(Y). 363. Which of the following model incorporates both mean reversion and no-arbitrage? A. Vasicek. B. Black Scholes. C. Cox, Ingersoll and Ross. D. Heath, Jarrow and Morton. 364.

3/150 9

Given two variables X and Y that follow geometric Brownian motion, which of the following variable will also follow geometric Brownian motion? A. X * Y. B. X + Y. C. log(X) + log(Y). D. exp(X) + exp(Y). 365. Which of the following are no-arbitrage models for fixed income valuation? I. Ho and Lee. II. Hull and White. III. Cox, Ingersoll and Ross. IV. Heath, Jarrow and Morton. A. I and II. B. III and IV. C. I, II and IV. D. II, III and IV. 366. Vasicek proposed the following model for interest rates: dr = a * (b - r) * dt + s * dz What is the long term mean of interest rates in this model? A. a B. b C. r D. s

http://64.106.152.240/inmarkets/passpro/index.cfm?event=questions....

2004-08-02

PASS PRO - Copyright 2004 Inmarkets Ltd - Licensed to Raymond chen

4/150 9

367. Which of the following models for interest rates would allow for mean reversion? A. dr = a * dt + s * dz B. dr = a * dt - b * dt + s * dz C. dr = a * (b - r) * dt + s * dz D. dr = a * (r - b) * dt + s * dz 368. Which of the following model incorporates both mean reversion and no-arbitrage? A. Vasicek. B. Ho and Lee. C. Black Scholes. D. Hull and White. 369. For which of the following process can the mean change be not equal to zero? I. Ito process. II. Wiener process. III. Generalized Wiener process. A. I and II. B. I and III. C. II and III. D. I, II and III. 370. Given two variables X and Y that are lognormally distributed, what is the distribution of X * Y? A. Normal. B. Lognormal. C. Exponential. D. None of the above.

http://64.106.152.240/inmarkets/passpro/index.cfm?event=questions....

2004-08-02

PASS PRO - Copyright 2004 Inmarkets Ltd - Licensed to Raymond chen

5/150 9

PASS PRO Solutions 1. Correct answer: B Continuously compounded rate = ln(FV/PV)/N = ln(10000 / 8455) / 2 = 8.39%. Study Session: 1 - RA: 1 2. Correct answer: B Using continuous compounding, PV = FV x exp(-Rate x Time period) = $700 x exp (- 9% x 9/12) = $654. Study Session: 1 - RA: 1 3. Correct answer: D Continuously compounded rate = ln(FV/PV)/N = ln(100 / 75) / 2 = 14.38%. Study Session: 1 - RA: 1 4. Correct answer: D Continuously compounded rate = ln(FV/PV)/N = ln(100 / 75) / 2 = 14.38%. Study Session: 1 - RA: 1 5. Correct answer: D Continuously compounded rate = ln(final value/initial value) / Number of periods = ln(70 / 60) / 4 = 3.85%. Study Session: 1 - RA: 1 6. Correct answer: A Continuously compounding return = ln(final value/initial value) / Number of periods = ln (35/13) / 5 = 20%. Study Session: 1 - RA: 1 7. Correct answer: C Using continuous compounding, PV = FV x exp(-Rate x Time period) = $800 x exp (- 20% x 12/12) = $655.

http://64.106.152.240/inmarkets/passpro/index.cfm?event=questions....

2004-08-02

PASS PRO - Copyright 2004 Inmarkets Ltd - Licensed to Raymond chen

6/150 9

Study Session: 1 - RA: 1 8. Correct answer: D Continuously compounded rate = ln(final value/initial value) / Number of periods = ln(70 / 60) / 4 = 3.85%. Study Session: 1 - RA: 1 9. Correct answer: A Continuously compounding return = ln(final value/initial value) / Number of periods = ln (35/13) / 5 = 20%. Study Session: 1 - RA: 1 10. Correct answer: B Continuously compounded rate = ln(final value/initial value) / Number of periods = ln(45 / 30) / 5 = 8.11%. Study Session: 1 - RA: 1 11. Correct answer: B Using continuous compounding, PV = FV x exp(-Rate x Time period) = $4,500 x exp (- 9% x 6/12) = $4,302. Study Session: 1 - RA: 1 12. Correct answer: C Continuously compounded rate = ln(FV/PV)/N = ln(700 / 400) / 3 = 18.65%. Study Session: 1 - RA: 1 13. Correct answer: C Using continuous compounding, PV = FV x exp(-Rate x Time period) = $800 x exp (- 20% x 12/12) = $655. Study Session: 1 - RA: 1 14. Correct answer: D Continuously compounded rate = ln(final value/initial value) / Number of periods = ln(40 / 15) / 2 = 49.04%. Study Session: 1 - RA: 1 15. Correct answer: C Continuously compounded rate = ln(FV/PV)/N = ln(800 / 500) / 6 = 7.83%. Study Session: 1 - RA: 1 16. Correct answer: B

http://64.106.152.240/inmarkets/passpro/index.cfm?event=questions....

2004-08-02

PASS PRO - Copyright 2004 Inmarkets Ltd - Licensed to Raymond chen

7/150 9

Using continuous compounding, PV = FV x exp(-Rate x Time period) = $700 x exp (- 9% x 9/12) = $654. Study Session: 1 - RA: 1 17. Correct answer: A This problem requires the use of binomial probability function, P(X=x) = fact(n) / fact(n - x) / fact(x) * p^x * (1-p)^(n-x) = fact(8) / fact(8 - 7) / fact(7) * 0.4^7 * 0.6^1 = 0.8%. Study Session: 1 - RA: 2 18. Correct answer: A Given that the daily standard deviation is $4.27 million, the standard deviation over 7 days = $4.27 million x (7/1)^0.5 = $11.29 million. Using this standard deviation, the level of $40 million is 3.1 standard deviations [= (75 40)/11.29] from the mean value. Given that the returns are normally distributed the probability of value falling more than 3.1 standard deviations from the mean value is 0.1% (since 100% of the probability falls between 3.1 standard deviations of the mean). Study Session: 1 - RA: 2 19. Correct answer: C The tail ends of a normal distribution stretch to infinity, although the area under these tails becomes insignificant past standard deviation from mean. 3 Study Session: 1 - RA: 2 20. Correct answer: D Covariance = Correlation x Standard Deviation_A x Standard Deviation_B. Therefore Standard Deviation_B = Covariance / Correlation / Standard Deviation_A = 4 / 0.5 / 16^0.5 = 2. Therefore Variance_B = 2^2 = 4. Study Session: 1 - RA: 2 21. Correct answer: D The characteristics of a normal distribution are: It is a continuous distribution. It is bell shaped. It is symmetrical about the mean. It peaks at the mean expected value. It extends theoretically from negative infinity to positive infinity (the probability asymptotically approaches zero at plus and minus infinity). It has a skewness of zero (i.e. it is symmetric). It has a kurtosis (the level of peakedness) of three. Below three the distribution is platykurtic

http://64.106.152.240/inmarkets/passpro/index.cfm?event=questions....

2004-08-02

PASS PRO - Copyright 2004 Inmarkets Ltd - Licensed to Raymond chen

8/150 9

(too flat) and above three it is leptokurtic (too tall). Study Session: 1 - RA: 2 22. Correct answer: C Confidence interval = 1 - CND (Quantile) = Area under the normal distribution to the right of the given Quantile point. Some of these points are rather important due to their use in VAR calculations. Confidence Level 90% 95% 99% Quantile (alpha) -1.282 -1.645 -2.326 Note: CND = Cumulative Normal Distribution function. Study Session: 1 - RA: 2 23. Correct answer: C A return of 0 percent and 10 percent is between -2 standard deviation and -1 standard deviation of the mean, i.e. a quartile of -2 to -1. The confidence levels corresponding to these quartiles are 97.7% and 84.1%. Hence the probability that the return falls between these limits = 97.7% - 84.1% = 13.6% Study Session: 1 - RA: 2 24. Correct answer: A Standard deviation = 100^0.5 = 10. Number of SD that 77 is away from mean = (77-100)/10 = -2.3. SD of 2.3 has a one-sided probability of 1.00%. Note: you can eliminate other alternatives as being too high without solving the problem. Study Session: 1 - RA: 2 25. Correct answer: C The movement of S&P500 as described by the analyst is a binomial process, i.e. the S&P500 can go up with a probability of 0.375 or go down with a probability of 0.625. Carrying out this process for 285 days the variance = n x p x (1 - p) = 285 x 0.375 x 0.625 = 66.797. Therefore, the standard deviation = 66.797^0.5 = 8.173. Study Session: 1 - RA: 2 26. Correct answer: A Given that the daily standard deviation is $9 million, the standard deviation over 10 days = $9 million x (10/1)^0.5 = $28.46 million. Given that the returns are normally distributed, we know that 99.9% of the outcomes will be above 3.1 standard deviations below the mean, i.e. above $1.8 million. Study Session: 1 - RA: 2

http://64.106.152.240/inmarkets/passpro/index.cfm?event=questions....

2004-08-02

PASS PRO - Copyright 2004 Inmarkets Ltd - Licensed to Raymond chen

9/150 9

27. Correct answer: D Covariance = Correlation x Standard Deviation_A x Standard Deviation_B. Therefore Standard Deviation_B = Covariance / Correlation / Standard Deviation_A = 4.2 / 0.6 / 5^0.5 = 3.1305. Therefore, Variance_B = 3.1305^2 = 9.80. Study Session: 1 - RA: 2 28. Correct answer: B Confidence interval = 1 - CND (Quantile) = Area under the normal distribution to the right of the given Quantile point. Some of these point are rather important due to their use in VAR calculations. Confidence Level 90% 95% 99% Quantile (alpha) 1.282 1.645 2.326 Note: CND = Cumulative Normal Distribution function Study Session: 1 - RA: 2 29. Correct answer: D The characteristics of a normal distribution are: It is a continuous distribution. It is bell shaped. It is symmetrical about the mean. It peaks at the mean expected value. It extends theoretically from negative infinity to positive infinity (the probability asymptotically approaches zero at plus and minus infinity). It has a skewness of zero (i.e. it is symmetric). It has a kurtosis (the level of peakedness) of three. Below three the distribution is platykurtic (too flat) and above three it is leptokurtic (too tall). Study Session: 1 - RA: 2 30. Correct answer: B The range of the 95 percent confidence interval = mean 1.96 x standard deviation / Number of employees^0.5 = 35 1.96 x 15 / 80^0.5 = 35 3.3 = between 31.7 and 38.3 Study Session: 1 - RA: 2 31. Correct answer: D Lognormal distribution is most appropriate for stock prices, since it does not allow for negative values and implies that the returns are normally distributed. Study Session: 1 - RA: 2

http://64.106.152.240/inmarkets/passpro/index.cfm?event=questions....

2004-08-02

PASS PRO - Copyright 2004 Inmarkets Ltd - Licensed to Raymond chen

00/150 1

32. Correct answer: B The characteristics of a normal distribution are: It is a continuous distribution. It is bell shaped. It is symmetrical about the mean. It peaks at the mean expected value. It extends theoretically from negative infinity to positive infinity (the probability asymptotically approaches zero at plus and minus infinity). It has a skewness of zero (i.e. it is symmetric). It has a kurtosis (the level of peakedness) of three. Below three the distribution is platykurtic (too flat) and above three it is leptokurtic (too tall). Study Session: 1 - RA: 2 33. Correct answer: A This problem requires the use of binomial probability function, P(X=x) = fact(n) / fact(n - x) / fact(x) * p^x * (1-p)^(n-x) = fact(8) / fact(8 - 7) / fact(7) * 0.4^7 * 0.6^1 = 0.8%. Study Session: 1 - RA: 2 34. Correct answer: D Covariance = Correlation x Standard Deviation_A x Standard Deviation_B. Therefore Standard Deviation_B = Covariance / Correlation / Standard Deviation_A = 4 / 0.5 / 16^0.5 = 2. Therefore Variance_B = 2^2 = 4. Study Session: 1 - RA: 2 35. Correct answer: D Covariance = Correlation x Standard Deviation_A x Standard Deviation_B. Therefore Standard Deviation_B = Covariance / Correlation / Standard Deviation_A = 15 / 0.5 / 25^0.5 = 6. Therefore Variance_B = 6^2 = 36. Study Session: 1 - RA: 2 36. Correct answer: A The movement of S&P500 as described by the analyst is a binomial process, i.e. the S&P500 can go up with a probability of 0.714 or go down with a probability of 0.286. Carrying out this process for 280 days the variance = n x p x (1 - p) = 280 x 0.714 x 0.286 = 57.177. Therefore the standard deviation = 57.177^0.5 = 7.562. Study Session: 1 - RA: 2 37. Correct answer: C

http://64.106.152.240/inmarkets/passpro/index.cfm?event=questions....

2004-08-02

PASS PRO - Copyright 2004 Inmarkets Ltd - Licensed to Raymond chen

01/150 1

The tail ends of a normal distribution stretch to infinity, although the area under these tails becomes insignificant past standard deviation from mean. 3 Study Session: 1 - RA: 2 38. Correct answer: C Given that the daily standard deviation is $1.5 million, the standard deviation over 250 days = $1.5 million x (250/1)^0.5 = $23.72 million. Given that the returns are normally distributed, we know that 95% of the outcomes will be above 1.645 standard deviations below the mean, i.e. above $101 million. Study Session: 1 - RA: 2 39. Correct answer: D Lognormal distribution is most appropriate for stock prices, since it does not allow for negative values and implies that the returns are normally distributed. Study Session: 1 - RA: 2 40. Correct answer: C Given that the daily standard deviation is $19 million, the standard deviation over 5 days = $19 million x (5/1)^0.5 = $42.49 million. Given that the returns are normally distributed, we know that 95% of the outcomes will be above 1.645 standard deviations below the mean, i.e. above $30.1 million. Study Session: 1 - RA: 2 41. Correct answer: D Covariance = Correlation x Standard Deviation_A x Standard Deviation_B. Therefore Standard Deviation_B = Covariance / Correlation / Standard Deviation_A = 22 / 0.8 / 25^0.5 = 5.5. Therefore Variance_B = 5.5^2 = 30.25. Study Session: 1 - RA: 2 42. Correct answer: B FV of this payment = $6,000 x (1 + 4%)^4 = $7,019. Study Session: 1 - RA: 3 43. Correct answer: B Using continuous compounding, PV = FV x exp(-Rate x Time period) = $8,400 x exp (- 8% x 6/12) = $8,071. Study Session: 1 - RA: 3 44. Correct answer: A

http://64.106.152.240/inmarkets/passpro/index.cfm?event=questions....

2004-08-02

PASS PRO - Copyright 2004 Inmarkets Ltd - Licensed to Raymond chen

02/150 1

Continuously compounded rate = ln(FV/PV)/N = ln(21000 / 16000) / 2 = 13.6%. Study Session: 1 - RA: 3 45. Correct answer: A The value of this annuity at the start of the year 8 = Coupon x [1 - 1/(1 + i)^N]/i = 4,000 x [1 - 1/(1 + 5.25%)^10] / 5.25% = $30,515. To find the value of this amount in today's money, we need to discount it back 7 years. Thus, present value of this annuity = $30,515 / (1 + 5.25%)^7 = $21,328. Study Session: 1 - RA: 3 46. Correct answer: C The mean of this set is 6. Therefore, the variance is [(6-2)^2 + (6-4)^2 + (6-6)^2 + (6-8)^2 + (6-10)^2]/4 = 10. Standard deviation = (variance)^0.5 = 3.2 Study Session: 1 - RA: 3 47. Correct answer: D Effective rate = (1 + Stated rate / Periodicity)^Periodicity -1 = (1 + 9%/12)^12 -1 = 9.38%. Study Session: 1 - RA: 3 48. Correct answer: B Continuously compounded rate = ln(FV/PV)/N = ln(10,000 / 7,500) / 5 = 5.75%. Study Session: 1 - RA: 3 49. Correct answer: D Without any negative cash flow in the stream, the rate of return is infinite. So, the IRR of this investment cannot be determined. Study Session: 1 - RA: 3 50. Correct answer: C FV of this payment = $4,000 x (1 + 7%)^10 = $7,869. Study Session: 1 - RA: 3 51. Correct answer: A The value of this annuity at the start of the year 2 = Coupon x [1 - 1/(1 + i)^N]/i = 900 x [1 1/(1 + 8%)^25] / 8% = $9,607. To find the value of this amount in today's month we need to discount it back 1 years. Thus, present value of this annuity = $9,607 / (1 + 8%)^1 = $8,895. Study Session: 1 - RA: 3

http://64.106.152.240/inmarkets/passpro/index.cfm?event=questions....

2004-08-02

PASS PRO - Copyright 2004 Inmarkets Ltd - Licensed to Raymond chen

03/150 1

52. Correct answer: A The value of this annuity at the start of the year 15 = Coupon x [1 - 1/(1 + i)^N]/i = 1500 x [1 - 1/(1 + 8%)^10] / 8% = $10,065. To find the value of this amount in today's month, we need to discount it back 14 years. Thus, present value of this annuity = $10,065 / (1 + 8%)^14 = $3,427. Study Session: 1 - RA: 3 53. Correct answer: C FV of this payment = $7,000 x (1 + 6%)^8 = $11,157. Study Session: 1 - RA: 3 54. Correct answer: C Continuously compounded rate = ln(FV/PV)/N = ln(8,000 / 7,000) / 4 = 3.34%. Study Session: 1 - RA: 3 55. Correct answer: C The mean of this set is 6. Therefore, the variance is [(6-2)^2 + (6-4)^2 + (6-6)^2 + (6-8)^2 + (6-10)^2]/4 = 10. Standard deviation = (variance)^0.5 = 3.2 Study Session: 1 - RA: 3 56. Correct answer: A Continuously compounded rate = ln(FV/PV)/N = ln(21000 / 16000) / 2 = 13.6%. Study Session: 1 - RA: 3 57. Correct answer: A The value of this annuity at the start of the year 2 = Coupon x [1 - 1/(1 + i)^N]/i = 2,000 x [1 - 1/(1 + 4%)^8] / 4% = $13,465. To find the value of this amount in today's month, we need to discount it back one year. Thus, present value of this annuity = $13,465 / (1 + 4%)^1 = $12,947. Study Session: 1 - RA: 3 58. Correct answer: C Effective rate = (1 + Stated rate / Periodicity)^Periodicity -1 = (1 + 7.79%/12)^12 -1 = 8.07%. Study Session: 1 - RA: 3 59. Correct answer: C Continuously compounded rate = ln(FV/PV)/N = ln(8,000 / 7,000) / 4 = 3.34%.

http://64.106.152.240/inmarkets/passpro/index.cfm?event=questions....

2004-08-02

PASS PRO - Copyright 2004 Inmarkets Ltd - Licensed to Raymond chen

04/150 1

Study Session: 1 - RA: 3 60. Correct answer: B 1,250 / r = 1,500 / (r + 1%) or, 1,250 x (r + 1%) = 1,500 x r or, r = 12.5 / (1,500 - 1,250) = 5%. Study Session: 1 - RA: 3 61. Correct answer: C FV of this payment = $7,000 x (1 + 6%)^8 = $11,157. Study Session: 1 - RA: 3 62. Correct answer: B FV of this payment = $6,000 x (1 + 4%)^4 = $7,019. Study Session: 1 - RA: 3 63. Correct answer: B Rate of inflation = (Rise in prices)^(1/Number of years) - 1 = (2)^(1/8) - 1 = 9.05%. Study Session: 1 - RA: 3 64. Correct answer: B The price of this bond is approximately equal to 100 / (1 + 0.05) ^ 2, which is equal to about 91. Study Session: 1 - RA: 3 65. Correct answer: B To achieve equivalence, 10,000 / (1 + r/2)^2 = 11,000 / (1 + r/2)^6 or, (1 + r/2)^4 = 1.1, or r = (1.1^(1/4) - 1) x 2 = 4.8%. Study Session: 1 - RA: 3 66. Correct answer: C Black Scholes framework uses continuously compounded rate. Given the semi-annual rate, this can be calculated as = ln(1.025) x 2 = 4.9385%. Study Session: 1 - RA: 3 67. Correct answer: A Correlation coefficient is a measure of the linear relationship between two random variables. It can be calculated by scaling the covariance between them and varies between -1 (perfect negative correlation) to +1 (perfect positive correlation). If the variables are independent, they will have a correlation coefficient of zero, but the reverse does not always hold true (i.e. a correlation coefficient of zero does not necessarily mean that they are independent).

http://64.106.152.240/inmarkets/passpro/index.cfm?event=questions....

2004-08-02

PASS PRO - Copyright 2004 Inmarkets Ltd - Licensed to Raymond chen

05/150 1

Covariance between the variables = Correlation coefficient x Standard deviation of the first variable x Standard deviation of the second variable. This calculation does not require the use of mean at all. Study Session: 1 - RA: 4 68. Correct answer: C The long position on negatively correlated index X does not hedge the short S&P 500, but rather the two add to it each other. In order to profit from the negative correlation, one needs to go long on both assets. Study Session: 1 - RA: 4 69. Correct answer: D Covariance = Correlation x Standard Deviation_A x Standard Deviation_B. Therefore Standard Deviation_B = Covariance / Correlation / Standard Deviation_A = 5 / 0.5 / 8^0.5 = 3.5355. Therefore Variance_B = 3.5355^2 = 12.5. Study Session: 1 - RA: 4 70. Correct answer: D Mean = -2 x 60% + 10 x 20% + 20 x 20% = 4.8. Study Session: 1 - RA: 4 71. Correct answer: D First we calculate the Mean = -12 x 24% + 4 x 40% + 14 x 36% = 3.76 Then, Variance = (3.76 - -12)^2 x 24% + (3.76 - 4)^2 x 40% + (3.76 - 14)^2 x 36% = 97.38 Study Session: 1 - RA: 4 72. Correct answer: B Correlation coefficient is a measure of the linear relationship between two random variables. It can be calculated by scaling the covariance between them and varies between -1 (perfect negative correlation) to +1 (perfect positive correlation). If the variables are independent, they will have a correlation coefficient of zero, but the reverse does not always hold true (i.e. a correlation coefficient of zero does not necessarily mean that they are independent). Covariance between the variables = Correlation coefficient x Standard deviation of the first variable x Standard deviation of the second variable. This calculation does not require the use of mean at all. Study Session: 1 - RA: 4 73. Correct answer: D Covariance = Correlation x Standard Deviation_A x Standard Deviation_B. Therefore Standard Deviation_B

http://64.106.152.240/inmarkets/passpro/index.cfm?event=questions....

2004-08-02

PASS PRO - Copyright 2004 Inmarkets Ltd - Licensed to Raymond chen

06/150 1

= Covariance / Correlation / Standard Deviation_A = 5 / 0.5 / 8^0.5 = 3.5355. Therefore Variance_B = 3.5355^2 = 12.5. Study Session: 1 - RA: 4 74. Correct answer: B Volatility scales as the square root of time. Therefore, the 22-day volatility = 5-day volatility x (22 / 5)^0.5 = 0.89% x 2.0976 = 1.87%. Study Session: 1 - RA: 4 75. Correct answer: C Mean = -2 x 50% + 8 x 25% + 15 x 25% = 4.75. Study Session: 1 - RA: 4 76. Correct answer: B Firstly, we calculate the Mean = -2 x 60% + 10 x 20% + 20 x 20% = 4.8 Then, Variance = (4.8 - -2)^2 x 60% + (4.8 - 10)^2 x 20% + (4.8 - 20)^2 x 20% = 79.36 Finally, Standard deviation = 79.36^0.5 = 8.91. Study Session: 1 - RA: 4 77. Correct answer: D Mean = -10 x 30% + 5 x 40% + 25 x 30% = 6.5. Study Session: 1 - RA: 4 78. Correct answer: B Volatility scales as the square root of time. Therefore, the 23-day volatility = 10-day volatility x (23 / 10)^0.5 = 1.4% x 1.5166 = 2.12%. Study Session: 1 - RA: 4 79. Correct answer: B Volatility scales as the square root of time. Therefore the 22-day volatility = 5-day volatility x (22 / 5)^0.5 = 1.2% x 2.0976 = 2.52%. Study Session: 1 - RA: 4 80. Correct answer: B Firstly, we calculate the Mean = -15 x 30% + 5 x 40% + 25 x 30% = 5 Then, Variance = (5 - -15)^2 x 30% + (5 - 5)^2 x 40% + (5 - 25)^2 x 30% = 240

http://64.106.152.240/inmarkets/passpro/index.cfm?event=questions....

2004-08-02

PASS PRO - Copyright 2004 Inmarkets Ltd - Licensed to Raymond chen

07/150 1

Finally, Standard deviation = 240^0.5 = 15.49. Study Session: 1 - RA: 4 81. Correct answer: B Let x be the standard deviation of asset 1. Thus 13^2 = (0.5 x SD1)^2 + (0.5 x 19.5)^2 + 2 x 0.5 x 0.5 x 0.5 x SD1 x 19.5. Solving this problem requires trail and error. Always try the middle number because even if it is not the answer, you can judge whether to try a higher number or a lower number. Study Session: 1 - RA: 4 82. Correct answer: B Let x be the standard deviation of asset 1. Thus 13^2 = (0.5 x SD1)^2 + (0.5 x 19.5)^2 + 2 x 0.5 x 0.5 x 0.5 x SD1 x 19.5. Solving this problem requires trail and error. Always try the middle number because even if it is not the answer, you can judge whether to try a higher number or a lower number. Study Session: 1 - RA: 4 83. Correct answer: C Correlation coefficient is a measure of the linear relationship between two random variables. It can be calculated by scaling the covariance between them and varies between -1 (perfect negative correlation) to +1 (perfect positive correlation). If the variables are independent they will have a correlation coefficient of zero, but the reverse does not always hold true (i.e. a correlation coefficient of zero does not necessarily mean that they are independent). Covariance between the variables = Correlation coefficient x Standard deviation of the first variable x Standard deviation of the second variable. This calculation does not require the use of mean at all. Study Session: 1 - RA: 4 84. Correct answer: B Mean = -4 x 20% + 6 x 40% + 12 x 40% = 6.4. Study Session: 1 - RA: 4 85. Correct answer: C Correlation coefficient is a measure of linear relationship between two random variables. It can be calculated by scaling the covariance between them and varies between -1 (perfect negative correlation) to +1 (perfect positive correlation). If the variables are independent they will have a correlation coefficient of zero, but the reverse does not always hold true (i.e. a correlation coefficient of zero does not necessarily mean that they are independent). Covariance between the variables = Correlation coefficient x Standard deviation of the first variable x Standard deviation of the second variable. This calculation does not require the use of mean at all. Study Session: 1 - RA: 4 86. Correct answer: B

http://64.106.152.240/inmarkets/passpro/index.cfm?event=questions....

2004-08-02

PASS PRO - Copyright 2004 Inmarkets Ltd - Licensed to Raymond chen

08/150 1

Volatility scales as the square root of time. Therefore, the 22-day volatility = 5-day volatility x (22 / 5)^0.5 = 0.89% x 2.0976 = 1.87%. Study Session: 1 - RA: 4 87. Correct answer: D Covariance = Correlation x Standard Deviation_A x Standard Deviation_B. Therefore Standard Deviation_B = Covariance / Correlation / Standard Deviation_A = 5 / 0.5 / 8^0.5 = 3.5355. Therefore, Variance_B = 3.5355^2 = 12.5. Study Session: 1 - RA: 4 88. Correct answer: D Correlation coefficient is a measure of the linear relationship between two random variables. It can be calculated by scaling the covariance between them and varies between -1 (perfect negative correlation) to +1 (perfect positive correlation). If the variables are independent they will have a correlation coefficient of zero, but the reverse does not always hold true (i.e. a correlation coefficient of zero does not necessarily mean that they are independent). Covariance between the variables = Correlation coefficient x Standard deviation of the first variable x Standard deviation of the second variable. This calculation does not require the use of mean at all. Study Session: 1 - RA: 4 89. Correct answer: B Volatility scales as the square root of time. Therefore the 250-day volatility = 5-day volatility x (250 / 5)^0.5 = 1.5% x 7.0711 = 10.61%. Study Session: 1 - RA: 4 90. Correct answer: B Mean = -4 x 20% + 6 x 40% + 12 x 40% = 6.4. Study Session: 1 - RA: 4 91. Correct answer: C Mean = -2 x 50% + 8 x 25% + 15 x 25% = 4.75. Study Session: 1 - RA: 4 92. Correct answer: D Step 1. The returns for the three scenarios given are: 42.857% [= (50 - 35)/35], 20% [= (42 35) / 35], and -42.857% [= (20 - 35) / 35] Step 2. Calculate expected price = 33% x 42.857% + 20% x 20% + 47% x -42.857% = -2%.

http://64.106.152.240/inmarkets/passpro/index.cfm?event=questions....

2004-08-02

PASS PRO - Copyright 2004 Inmarkets Ltd - Licensed to Raymond chen

09/150 1

Step 3. Calculate Variance = 33% x (-2% - 42.857%)^2 + 20% x (-2% - 20%)^2 + 47% x (2% + 42.857%)^2 = 0.154538 Step 4. Calculate volatility/standard deviation = 0.154538^0.5 = 39.31%. Study Session: 1 - RA: 5 93. Correct answer: D Coefficient of variation (CV) is a measure of relative risk and is calculated as: CV = (standard deviation of returns) / (expected rate of return). The calculations for each investment are shown below: CV(A) = 0.008^0.5 / 0.15 = 0.596. CV(B) = 0.005^0.5 / 0.12 = 0.589. CV(C) = 0.03 / 0.08 = 0.375. CV(D) = 0.02 / 0.03 = 0.667. Study Session: 1 - RA: 5 94. Correct answer: D The null hypothesis for this test is that the coefficient is equal to zero. Therefore the test statistic = (Observed value - 0) / Standard error = (9% - 0) / 4% = 2.25. Study Session: 1 - RA: 5 95. Correct answer: A Coefficient of variation (CV) is a measure of relative risk and is calculated as: CV = (standard deviation of returns) / (expected rate of return). The calculations for each investment are shown below: CV(A) = 0.005^0.5 / 0.19 = 0.372. CV(B) = 0.002^0.5 / 0.15 = 0.298. CV(C) = 0.03 / 0.1 = 0.3. CV(D) = 0.01 / 0.03 = 0.333. Study Session: 1 - RA: 5 96. Correct answer: D Coefficient of variation (CV) is a measure of relative risk and is calculated as: CV = (standard deviation of returns) / (expected rate of return). The calculations for each investment are shown below: CV(A) = 0.008^0.5 / 0.15 = 0.596. CV(B) = 0.005^0.5 / 0.12 = 0.589. CV(C) = 0.03 / 0.08 = 0.375. CV(D) = 0.02 / 0.03 = 0.667. Study Session: 1 - RA: 5 97. Correct answer: C In this distribution, the lowest element is 1.5 percent below the mean whereas the highest element is 2.5 percent above the mean. Thus the distribution is skewed towards the right.

http://64.106.152.240/inmarkets/passpro/index.cfm?event=questions....

2004-08-02

PASS PRO - Copyright 2004 Inmarkets Ltd - Licensed to Raymond chen

10/150 1

Study Session: 1 - RA: 5 98. Correct answer: B The null hypothesis for this test is that the coefficient is equal to zero. Therefore, the test statistic = (Observed value - 0) / Standard error = (12% - 0) / 4% = 3. Study Session: 1 - RA: 5 99. Correct answer: D The null hypothesis for this test is that the coefficient is equal to zero. Therefore the test statistic = (Observed value - 0) / Standard error = (9% - 0) / 4% = 2.25. Study Session: 1 - RA: 5 100. Correct answer: D The null hypothesis for this test is that the coefficient is equal to zero. Therefore the test statistic = (Observed value - 0) / Standard error = (11.8% - 0) / 3% = 3.93. Study Session: 1 - RA: 5 101. Correct answer: C Standard error of the mean = s / n^0.5 = 3.5 / 30^0.5 = 0.64. Study Session: 1 - RA: 5 102. Correct answer: D The null hypothesis for this test is that the coefficient is equal to zero. Therefore the test statistic = (Observed value - 0) / Standard error = (12.25% - 0) / 8% = 1.53. Study Session: 1 - RA: 5 103. Correct answer: D Coefficient of variation (CV) is a measure of relative risk and is calculated as: CV = (standard deviation of returns) / (expected rate of return). The calculations for each investment are shown below: CV(A) = 0.008^0.5 / 0.15 = 0.596. CV(B) = 0.005^0.5 / 0.12 = 0.589. CV(C) = 0.03 / 0.08 = 0.375. CV(D) = 0.02 / 0.03 = 0.667. Study Session: 1 - RA: 5 104. Correct answer: B The null hypothesis for this test is that the coefficient is equal to zero. Therefore, the test statistic = (Observed value - 0) / Standard error = (12% - 0) / 4% = 3. Study Session: 1 - RA: 5 105. Correct answer: D

http://64.106.152.240/inmarkets/passpro/index.cfm?event=questions....

2004-08-02

PASS PRO - Copyright 2004 Inmarkets Ltd - Licensed to Raymond chen

11/150 1

Coefficient of variation (CV) is a measure of relative risk and is calculated as: CV = (standard deviation of returns) / (expected rate of return). The calculations for each investment are shown below: CV(A) = 0.005^0.5 / 0.12 = 0.589. CV(B) = 0.002^0.5 / 0.1 = 0.447. CV(C) = 0.03 / 0.15 = 0.2. CV(D) = 0.05 / 0.06 = 0.833. Study Session: 1 - RA: 5 106. Correct answer: A Coefficient of variation (CV) is a measure of relative risk and is calculated as: CV = (standard deviation of returns) / (expected rate of return). The calculations for each investment are shown below: CV(A) = 0.002^0.5 / 0.1 = 0.447. CV(B) = 0.003^0.5 / 0.15 = 0.365. CV(C) = 0.03 / 0.08 = 0.375. CV(D) = 0.01 / 0.05% = 0.2. Study Session: 1 - RA: 5 107. Correct answer: C Standard error of the mean = s /vn = 4/v40 = 0.63. Study Session: 1 - RA: 5 108. Correct answer: D Coefficient of variation (CV) is a measure of relative risk and is calculated as: CV = (standard deviation of returns) / (expected rate of return). The calculations for each investment are shown below: CV(A) = 0.008^0.5 / 0.15 = 0.596. CV(B) = 0.005^0.5 / 0.12 = 0.589. CV(C) = 0.03 / 0.08 = 0.375. CV(D) = 0.02 / 0.03 = 0.667. Study Session: 1 - RA: 5 109. Correct answer: C Standard error of the mean = s /n^0.5 = 6/90^0.5 = 0.63. Study Session: 1 - RA: 5 110. Correct answer: C Standard error of the mean = s /n^0.5 = 6/50^0.5 = 0.85. Study Session: 1 - RA: 5 111. Correct answer: C

http://64.106.152.240/inmarkets/passpro/index.cfm?event=questions....

2004-08-02

PASS PRO - Copyright 2004 Inmarkets Ltd - Licensed to Raymond chen

12/150 1

Standard error of the mean = s /n^0.5 = 7/60^0.5 = 0.9. Study Session: 1 - RA: 5 112. Correct answer: C Standard error of the mean = s /n^0.5 = 6/90^0.5 = 0.63. Study Session: 1 - RA: 5 113. Correct answer: D The null hypothesis for this test is that the coefficient is equal to zero. Therefore the test statistic = (Observed value - 0) / Standard error = (9% - 0) / 3.25% = 2.77. Study Session: 1 - RA: 5 114. Correct answer: B This problem requires a two-tailed F-test (with the null hypothesis Ho: variance of pharmaceutical industry returns = variance of the market). From the given data the test statistic = 0.13^2 / 0.092^2 = 2. The critical value of the F-statistic (for df1=40, df2=20 and p=0.025) is 2.29. Since the test statistic is lower than the critical statistic, we cannot reject the null hypothesis that the standard deviation of pharmaceutical stocks is same as that of the market. Study Session: 1 - RA: 6 115. Correct answer: C Since this is a one-tailed test with a 0.01 significance level the critical Z value is 2.33. Study Session: 1 - RA: 6 116. Correct answer: A A t-test is used when the sample size is small and the variance of the population is unknown. Study Session: 1 - RA: 6 117. Correct answer: D Tests of the variance (or standard deviation) of a population require the chi-squared test. Study Session: 1 - RA: 6 118. Correct answer: A The required test for testing the variance is the chi-squared test. The test statistic = (n - 1) x Sample variance / Hypothesized variance = 60 x 0.21^2 / 0.14^2 = 135. To test whether the standard deviation is higher (Ho: standard deviation is lower than or equal to 14%), the critical value of chi-squared will be 79.08 (using df = 60 and p= 0.05). Since the test statistic is higher than the critical value, the analyst can reject the null hypothesis and conclude that the standard deviation of returns is higher than 14%. Study Session: 1 - RA: 6

http://64.106.152.240/inmarkets/passpro/index.cfm?event=questions....

2004-08-02

PASS PRO - Copyright 2004 Inmarkets Ltd - Licensed to Raymond chen

13/150 1

119. Correct answer: C Tests of differences between variances (or standard deviation) of two populations require the Ftest. From the given data, the test statistic = 0.15^2 / 0.11^2 = 1.86. The critical value of the F-statistic (for df1= 40, df2= 20 and p= 0.05) is 1.99. Since the test statistic is lower than the critical statistic, we cannot reject the null hypothesis that the standard deviation of utility stocks is equal to or higher than that of the market. Study Session: 1 - RA: 6 120. Correct answer: B The required test for testing the variance is the chi-squared test. The test statistic = (n - 1) x Sample variance / Hypothesized variance = 19 x 0.12^2 / 0.18^2 = 8.44. To test whether the standard deviation is different (Ho: standard deviation is equal to 18%), the critical values of chi-squared will be 8.91 and 32.85(using df = 19, p-lower = 0.975 and p-higher = 0.025). Since the test statistic falls outside the critical range, the analyst can reject the null hypothesis and conclude that the standard deviation of returns is different from 18%. Study Session: 1 - RA: 6 121. Correct answer: C This is a two-tailed t-test for the difference between means of two independent populations. Step 1. The common variance = [(n1 -1) x Variance1 + (n2 - 1) x Variance2] / (n1 + n2 - 2) = [(21 -1) x 0.12^2 + (41 - 1) x 0.13^2] / (21 + 41 - 2) = 0.0161. Step 2. The value of the test statistic = (Mean1 - Mean2) / (Common variance/n1 + Common variance/n2)^0.5 = (0.15 0.12) / (0.0161/21 + 0.0161/41)^0.5 = 0.88. Using degrees of freedom = n1 + n2 - 2 = 21 + 41 - 2 = 60 and level of significance = alpha / 2 = 0.025 the critical value = 2.0. Since the test statistic is lower than the critical statistic, we cannot reject the null hypothesis that the mean returns are same. Study Session: 1 - RA: 6 122. Correct answer: C The required test for testing the variance is the chi-squared test. The test statistic = (n - 1) x Sample variance / Hypothesized variance = 14 x 0.1^2 / 0.15^2 = 6.22. To test whether the standard deviation is different (Ho: standard deviation is equal to 15%), the critical values of chi-squared will be 5.63 and 26.12 (using df=14, p-lower=0.975 and p-higher=0.025). Since the test statistic falls inside the critical range, the analyst cannot reject the null hypothesis . Study Session: 1 - RA: 6 123. Correct answer: A This is a t-test for the difference between means of two independent populations. Step 1. The common variance = [(n1 -1) x Variance1 + (n2 - 1) x Variance2] / (n1 + n2 - 2) = [(21 -1) x 0.13^2 + (41 - 1) x 0.15^2] / (21 + 41 - 2) = 0.0206. Step 2. The value of the test statistic = (Mean1 - Mean2) / (Common variance/n1 + Common variance/n2)^0.5 = (0.12 - 0.09) / (0.0206/21 + 0.0206/41)^0.5 = 0.78. Study Session: 1 - RA: 6 124. Correct answer: A Tests of the variance of a population require the chi-squared test. Since the analyst wants to show that the standard deviation is less than 14%, this will be chosen as the alternative

http://64.106.152.240/inmarkets/passpro/index.cfm?event=questions....

2004-08-02

PASS PRO - Copyright 2004 Inmarkets Ltd - Licensed to Raymond chen

14/150 1

hypothesis and the null hypothesis will be that the standard deviation is higher than or equal to 14%. Thus, using a probability in the right tail of 95% and degrees of freedom of 24, the critical value from the chi-squared table is 13.85. Study Session: 1 - RA: 6 125. Correct answer: C Tests of the variance of a population require the chi-squared test. For this data chi-squared = (n - 1) x Sample variance / Hypothesized variance = 14 x 0.13^2 / 0.18^2 = 7.3. Study Session: 1 - RA: 6 126. Correct answer: B The null hypothesis in this study is that the performance of active portfolio managers <= average for all portfolio managers. The value of test statistic for this, Z = (0.15 - 0.12)/(0.08 / 16^0.5) = 0.03 / (0.08 / 4) = 1.5. Since the analyst is using a 0.05 significance level the critical value of Z is 1.65 (the rejection region is above 1.65). As the test statistic is lower than the critical value the null hypothesis cannot be rejected, i.e. the performance of active portfolio managers is not significantly higher than the average for all portfolio managers. Study Session: 1 - RA: 6 127. Correct answer: D Theoretically, the z-statistic can be used when the population variance is known, but it can also be used for populations with unknown variances if the sample size is large. However, if the population variance is unknown and the sample size is small then it is necessary to use tstatistic. Study Session: 1 - RA: 6 128. Correct answer: C The significance level does not depend on the data. Rather it is chosen separately based on how much evidence is required before a null hypothesis will be rejected. The steps in the hypothesis testing process are: 1) State a hypothesis. 2) Identify the test statistic and its probability distribution. 3) Specify the significance level. 4) State the decision rule. 5) Collect data and perform the calculations. 6) Make the statistical decision. 7) Make the investment/economic decision based on the above. Study Session: 1 - RA: 6 129. Correct answer: A This problem requires a hypothesis test in which the null hypothesis, H0, is that Peter's performance is not significantly higher than average. The z statistic for this test = (Peter's performance - Average performance) / Standard deviation = (1,000,000 - 800,000) / 100,000 = 2. The critical value of the z statistic for a significance level of 0.05 is 1.645. Since the test statistic is larger than this critical value, we reject the null hypothesis and accept the alternate hypothesis that Peter's performance is significantly higher than average. Study Session: 1 - RA: 6 130. Correct answer: B

http://64.106.152.240/inmarkets/passpro/index.cfm?event=questions....

2004-08-02

PASS PRO - Copyright 2004 Inmarkets Ltd - Licensed to Raymond chen

15/150 1

Tests of differences between variances (or standard deviation) of two populations require the Ftest. Looking up the F-table for 0.05 significance level, with 40 degrees of freedom in the numerator and 20 in the denominator (note the population with the higher variance goes in the numerator), the critical value of F is 1.99. Study Session: 1 - RA: 6 131. Correct answer: D Tests of the variance of a population require the chi-squared test. For this data chi-squared = (n - 1) x Sample variance / Hypothesized variance = 59 x 0.22^2 / 0.045 = 63.46. Since the analyst wants to show that the variance is more than 0.045, this will be chosen as the alternative hypothesis and the null hypothesis will be that the variance is lower than or equal to 0.045. The critical value of the chi-squared statistic (for df = 59 and p= 5%) is 77.93. Since the test statistic is lower than the critical statistic, we cannot reject the null hypothesis that the variance is equal to or lower than 0.045. Study Session: 1 - RA: 6 132. Correct answer: B The required test for testing the variance is the chi-squared test. The test statistic = (n - 1) x Sample variance / Hypothesized variance = 19 x 0.09^2 / 0.16^2 = 6.01. To test whether the standard deviation is different (Ho: standard deviation is equal to 16%), the critical values of chi-squared will be 8.91 and 32.85(using df = 19, p-lower = 0.975 and p-higher = 0.025). Since the test statistic falls outside the critical range, the analyst can reject the null hypothesis and conclude that the standard deviation of returns is different from 16%. Study Session: 1 - RA: 6 133. Correct answer: D The alternate hypothesis is the statement which will be accepted if the null hypothesis is proven wrong. Therefore, we make whatever we are trying to test as the alternate hypothesis - in this case that the mean price of luxury cars is greater than $80,000, and the null hypothesis as the opposite (the mean price of luxury cars is less than or equal to $80,000). This problem is a common example of how statisticians establish hypotheses by proving that the opposite (i.e. the null hypothesis) is false. Study Session: 1 - RA: 6 134. Correct answer: A This is a paired comparison test because the two samples are not independent. The null hypothesis: Ho: mean difference in betas before and after the dot-com bust = 0. The test statistic = (Mean of differences - 0) / Standard error = (0.18 - 0) / (0.2 / 10^0.5) = 2.85. The critical value of t-statistic for 9 degrees of freedom and p = 0.025 (this is a two sided test) is 2.26. Since the test statistic is higher than the critical value of t-statistic, the analyst can reject the null hypothesis. Study Session: 1 - RA: 6 135. Correct answer: C Tests of the variance of a population require the chi-squared test. Since the analyst wants to show that the variance is more than 0.04, this will be chosen as the alternative hypothesis and the null hypothesis will be that the variance is lower than or equal to 0.04. Thus using a probability in the right tail of 5% and degrees of freedom of 30, the critical value from the chi-

http://64.106.152.240/inmarkets/passpro/index.cfm?event=questions....

2004-08-02

PASS PRO - Copyright 2004 Inmarkets Ltd - Licensed to Raymond chen

16/150 1

squared table is 54.57. Study Session: 1 - RA: 6 136. Correct answer: B Tests of the variance of a population require the chi-squared test. For this data chi-squared = (n - 1) x Sample variance / Hypothesized variance = 59 x 0.22^2 / 0.045 = 63.46. Study Session: 1 - RA: 6 137. Correct answer: B This is a paired comparison test because the two samples are not independent. The null hypothesis: Ho: mean difference in betas before and after the 1990 deregulation = 0. The test statistic = (Mean of differences - 0) / Standard error = (0.25 - 0) / (0.3 / 12^0.5) = 2.89. The critical value of t-statistic for 11 degrees of freedom and p = 0.005 (this is a two sided test) is 3.11. Since the test statistic is lower than the critical value of t-statistic, the analyst cannot reject the null hypothesis. Study Session: 1 - RA: 6 138. Correct answer: A In hypothesis testing we accept the alternate hypothesis if the null hypothesis has been rejected. Type I error happens if the null hypothesis is rejected when it is actually true. Type II error happens if the null hypothesis is accepted when it is actually false. The power of the test is the probability of correctly rejecting the null hypothesis (when it is false), so minimizing Type II errors would maximize the power of the test. Study Session: 1 - RA: 6 139. Correct answer: C Coefficient of determination = ESS / TSS = 3.312 / 3.617 = 0.916. Study Session: 1 - RA: 7 140. Correct answer: A This is a one-tailed test with 39 degrees of freedom and significance level of 0.1. Looking up the Student's t-distribution for df = 39 and p = 0.1, we get the critical value of 1.304. Study Session: 1 - RA: 7 141. Correct answer: D Correlation = (Coefficient of determination)^0.5 = (ESS / TSS)^0.5 = (761.562 / 865.333) ^0.5 = 0.938. Study Session: 1 - RA: 7 142. Correct answer: C Firstly, we calculate the slope coefficient = Sum_XY / Sum_X^2 = 9.08 / 24.9 = 0.365. Then we calculate the intercept = Avg_Y - Slope x Avg X = 4.343 - 0.365 x 3.5 = 3.07.

http://64.106.152.240/inmarkets/passpro/index.cfm?event=questions....

2004-08-02

PASS PRO - Copyright 2004 Inmarkets Ltd - Licensed to Raymond chen

17/150 1

Study Session: 1 - RA: 7 143. Correct answer: A Firstly, we calculate the slope coefficient = Sum_XY / Sum_X^2 = 133.302 / 23.333 = 5.713. Then we calculate the intercept = Avg_Y - Slope x Avg X = -15.667 - 5.713 x 6.333 = -51.85. Study Session: 1 - RA: 7 144. Correct answer: D Correlation = (Coefficient of determination)^0.5 = (ESS / TSS)^0.5 = (227,088.816 / 245,142.857)^0.5 = 0.962. Study Session: 1 - RA: 7 145. Correct answer: D The standard error = USS / (N-2) = (TSS - ESS) / (N-2) = (113,171.429 - 96,057.049) / (7-2) = 3,422.876. Study Session: 1 - RA: 7 146. Correct answer: A Firstly, we calculate the slope coefficient = Sum_XY / Sum_X^2 = 133.302 / 23.333 = 5.713. Then we calculate the intercept = Avg_Y - Slope x Avg X = -15.667 - 5.713 x 6.333 = -51.85. Study Session: 1 - RA: 7 147. Correct answer: D Correlation = (Coefficient of determination)^0.5 = (ESS / TSS)^0.5 = (3.312 / 3.617)^0.5 = 0.957. Study Session: 1 - RA: 7 148. Correct answer: D The percentage of explained variation = Correlation coefficient ^ 2 = 0.6 ^ 2 = 0.36 or 36%. Therefore, the unexplained variation is 64%. Study Session: 1 - RA: 7 149. Correct answer: D Correlation = (Coefficient of determination)^0.5 = (ESS / TSS)^0.5 = (761.562 / 865.333) ^0.5 = 0.938. Study Session: 1 - RA: 7 150. Correct answer: B The percentage of explained variation = Correlation coefficient ^ 2 = 0.75 ^ 2 = 0.5625 or

http://64.106.152.240/inmarkets/passpro/index.cfm?event=questions....

2004-08-02

PASS PRO - Copyright 2004 Inmarkets Ltd - Licensed to Raymond chen

18/150 1

56.25%. Therefore, the unexplained variation is 43.75%. Study Session: 1 - RA: 7 151. Correct answer: B The percentage of explained variation = Correlation coefficient ^ 2 = 0.85 ^ 2 = 0.7225 or 72.25%. Therefore, the unexplained variation is 27.75%. Study Session: 1 - RA: 7 152. Correct answer: D The percentage of explained variation = Correlation coefficient ^ 2 = 0.25 ^ 2 = 0.0625 or 6.25%. Therefore the unexplained variation is 93.75%. Study Session: 1 - RA: 7 153. Correct answer: C Firstly, we calculate the slope coefficient = Sum_XY / Sum_X^2 = 9.08 / 24.9 = 0.365. Then we calculate the intercept = Avg_Y - Slope x Avg X = 4.343 - 0.365 x 3.5 = 3.07. Study Session: 1 - RA: 7 154. Correct answer: B The percentage of explained variation = Correlation coefficient ^ 2 = 0.75 ^ 2 = 0.5625 or 56.25%. Therefore, the unexplained variation is 43.75%. Study Session: 1 - RA: 7 155. Correct answer: C The test to compare variances of two normally distributed populations is F-statistic. Study Session: 1 - RA: 7 156. Correct answer: C Coefficient of determination = ESS / TSS = 227,088.816 / 245,142.857 = 0.926. Study Session: 1 - RA: 7 157. Correct answer: D The standard error = USS / (N-2) = (TSS - ESS) / (N-2) = (3.617 - 3.312) / (7-2) = 0.061. Study Session: 1 - RA: 7 158. Correct answer: D The standard error = USS / (N-2) = (TSS - ESS) / (N-2) = (245,142.857 - 227,088.816) / (72) = 3,610.8082.

http://64.106.152.240/inmarkets/passpro/index.cfm?event=questions....

2004-08-02

PASS PRO - Copyright 2004 Inmarkets Ltd - Licensed to Raymond chen

19/150 1

Study Session: 1 - RA: 7 159. Correct answer: A The slope coefficient = Sum_XY / Sum_X^2 = 9.08 / 24.9 = 0.365. Study Session: 1 - RA: 7 160. Correct answer: D The percentage of explained variation = Correlation coefficient ^ 2 = 0.25 ^ 2 = 0.0625 or 6.25%. Therefore the unexplained variation is 93.75%. Study Session: 1 - RA: 7 161. Correct answer: C The slope coefficient = Sum_XY / Sum_X^2 = 1,640 / 28 = 58.571. Study Session: 1 - RA: 7 162. Correct answer: D Correlation = (Coefficient of determination)^0.5 = (ESS / TSS)^0.5 = (227,088.816 / 245,142.857)^0.5 = 0.962. Study Session: 1 - RA: 7 163. Correct answer: D Correlation = (Coefficient of determination)^0.5 = (ESS / TSS)^0.5 = (96,057.049 / 113,171.429)^0.5 = 0.921. Study Session: 1 - RA: 7 164. Correct answer: A Since the investments are perfectly negatively correlated the volatility of the portfolio = (w1 x v1 - w2 x v2). This can be reduced to zero by adjusting the amounts invested in the inverse of the ratios of the volatilities (i.e. w1/w2 = v2/v1). Study Session: 1 - RA: 8 165. Correct answer: C The positive correlation only means that the prices will tend to move in the same direction, not that they always will. Study Session: 1 - RA: 8 166. Correct answer: C Since the investments are perfectly negatively correlated the volatility of the portfolio = (w1 x v1 - w2 x v2). This can be reduced to zero by adjusting the amounts invested in the inverse of the ratios of the volatilities (i.e. w1/w2 = v2/v1) Study Session: 1 - RA: 8

http://64.106.152.240/inmarkets/passpro/index.cfm?event=questions....

2004-08-02

PASS PRO - Copyright 2004 Inmarkets Ltd - Licensed to Raymond chen

20/150 1

167. Correct answer: C Since the investments are perfectly negatively correlated the volatility of the portfolio = (w1 x v1 - w2 x v2). This can be reduced to zero by adjusting the amounts invested in the inverse of the ratios of the volatilities (i.e. w1/w2 = v2/v1) Study Session: 1 - RA: 8 168. Correct answer: A Since the investments are perfectly negatively correlated the volatility of the portfolio = (w1 x v1 - w2 x v2). This can be reduced to zero by adjusting the amounts invested in the inverse of the ratios of the volatilities (i.e. w1/w2 = v2/v1). Study Session: 1 - RA: 8 169. Correct answer: C The positive correlation only means that the prices will tend to move in the same direction, not that they always will. Study Session: 1 - RA: 8 170. Correct answer: A Since the investments are perfectly negatively correlated the volatility of the portfolio = (w1 x v1 - w2 x v2). This can be reduced to zero by adjusting the amounts invested in the inverse of the ratios of the volatilities (i.e. w1/w2 = v2/v1) Study Session: 1 - RA: 8 171. Correct answer: A Since the investments are perfectly negatively correlated the volatility of the portfolio = (w1 x v1 - w2 x v2). This can be reduced to zero by adjusting the amounts invested in the inverse of the ratios of the volatilities (i.e. w1/w2 = v2/v1) Study Session: 1 - RA: 8 172. Correct answer: C The positive correlation only means that the prices will tend to move in the same direction, not that they always will. Study Session: 1 - RA: 8 173. Correct answer: C Correlation coefficient = Covariance / (Standard deviation of stock A x Standard deviation of market index) = -0.002 / (0.018^0.5 x 0.025^0.5) = -0.09. Study Session: 1 - RA: 8 174. Correct answer: D Covariance of returns is an absolute measure of movement. The magnitude depends on size of the individual assets standard deviations and the relationship between co-movements.

http://64.106.152.240/inmarkets/passpro/index.cfm?event=questions....

2004-08-02

PASS PRO - Copyright 2004 Inmarkets Ltd - Licensed to Raymond chen

21/150 1

A positive covariance means that the asset returns move in the same direction, while a negative covariance means that asset returns move in the opposite direction. A zero covariance means that there is no relationship between asset returns. Study Session: 1 - RA: 8 175. Correct answer: D Of the models given here, only Heath, Jarrow and Morton is a no-arbitrage model that incorporates mean reversion. Cox, Ingersoll and Ross model and Vasicek model incorporate mean reversion but are not a no-arbitrage model. Black-Scholes does not meet either criteria. Study Session: 1 - RA: 10 176. Correct answer: A Since X and Y follow geometric Brownian motion, it follows that log(X) and log(Y) are normally distributed, which means that log(X) + log(Y) is also normally distributed. log(X) + log(Y) is equal to log(X * Y), so X * Y must follow geometric Brownian motion. Study Session: 1 - RA: 10 177. Correct answer: C Wiener process has a mean change of zero and variance proportional to the time interval. Study Session: 1 - RA: 10 178. Correct answer: B Wiener process has a mean change of zero and variance proportional to the time interval. Generalized Wiener process and Ito process incorporate a trend variable a * dt. Study Session: 1 - RA: 10 179. Correct answer: D Only Hull and White is a no-arbitrage model that incorporates mean reversion. Ho and Lee is a no-arbitrage model that does not incorporate mean reversion. Vasicek model incorporates mean reversion but is not a no-arbitrage model. Black-Scholes does not meet either criteria. Study Session: 1 - RA: 10 180. Correct answer: C Wiener process has a mean change of zero and variance proportional to the time interval. Study Session: 1 - RA: 10 181. Correct answer: B If X and Y are lognormally distributed, log(X) and log(Y) will be normally distributed, which means that log(X) + log(Y) = log(X * Y) is also normally distributed, implying that X * Y must be lognormally distributed.

http://64.106.152.240/inmarkets/passpro/index.cfm?event=questions....

2004-08-02

PASS PRO - Copyright 2004 Inmarkets Ltd - Licensed to Raymond chen

22/150 1

Study Session: 1 - RA: 10 182. Correct answer: C The Hull and White, the Ho and Lee, and the Heath, Jarrow and Morton are no-arbitrage models. Study Session: 1 - RA: 10 183. Correct answer: C dr = a * (b - r) * dt + s * dz, is the only model given above that allows for the interest rate r to mean revert to a level b. Study Session: 1 - RA: 10 184. Correct answer: B In the model: dr = a * (b - r) * dt + s * dz, the interest rate r will mean revert to a level b. Study Session: 1 - RA: 10 185. Correct answer: B If X and Y are normally distributed, X + Y will also be normally distributed. Study Session: 1 - RA: 10 186. Correct answer: B Using continuous compounding, PV = FV x exp(-Rate x Time period) = $8,500 x exp (- 6.5% x 5) = $8,008. Study Session: 1 - RA: 1 187. Correct answer: A Using continuous compounding, PV = FV x exp(-Rate x Time period) = $600 x exp (- 9% x 8) = $292. Study Session: 1 - RA: 1 188. Correct answer: A Using continuous compounding, PV = FV x exp(-Rate x Time period) = $1,500 x exp (- 9% x 4) = $1,047. Study Session: 1 - RA: 1 189. Correct answer: C Continuously compounded rate = ln(FV/PV)/N = ln(700 / 400) / 3 = 18.65%. Study Session: 1 - RA: 1 190. Correct answer: D

http://64.106.152.240/inmarkets/passpro/index.cfm?event=questions....

2004-08-02

PASS PRO - Copyright 2004 Inmarkets Ltd - Licensed to Raymond chen

23/150 1

Continuously compounded rate = ln(final value/initial value) / Number of periods = ln(40 / 15) / 2 = 49.04%. Study Session: 1 - RA: 1 191. Correct answer: B Using continuous compounding, PV = FV x exp(-Rate x Time period) = $4,500 x exp (- 9% x 6/12) = $4,302. Study Session: 1 - RA: 1 192. Correct answer: D Continuously compounded rate = ln(final value/initial value) / Number of periods = ln(40 / 15) / 2 = 49.04%. Study Session: 1 - RA: 1 193. Correct answer: A Using continuous compounding, PV = FV x exp(-Rate x Time period) = $500 x exp (- 9% x 9/12) = $467. Study Session: 1 - RA: 1 194. Correct answer: B Continuously compounded rate = ln(FV/PV)/N = ln(10000 / 8455) / 2 = 8.39%. Study Session: 1 - RA: 1 195. Correct answer: B Continuously compounded rate = ln(FV/PV)/N = ln(10000 / 8455) / 2 = 8.39%. Study Session: 1 - RA: 1 196. Correct answer: B Using continuous compounding, PV = FV x exp(-Rate x Time period) = $4,500 x exp (- 9% x 6/12) = $4,302. Study Session: 1 - RA: 1 197. Correct answer: D Continuously compounded rate = ln(FV/PV)/N = ln(100 / 75) / 2 = 14.38%. Study Session: 1 - RA: 1 198. Correct answer: C Continuously compounded rate = ln(FV/PV)/N = ln(800 / 500) / 6 = 7.83%. Study Session: 1 - RA: 1

http://64.106.152.240/inmarkets/passpro/index.cfm?event=questions....

2004-08-02

PASS PRO - Copyright 2004 Inmarkets Ltd - Licensed to Raymond chen

24/150 1

199. Correct answer: A Using continuous compounding, PV = FV x exp(-Rate x Time period) = $400 x exp (- 9% x 8) = $195. Study Session: 1 - RA: 1 200. Correct answer: B Using continuous compounding, PV = FV x exp(-Rate x Time period) = $8,500 x exp (- 6.5% x 5) = $8,008. Study Session: 1 - RA: 1 201. Correct answer: B Using continuous compounding, PV = FV x exp(-Rate x Time period) = $8,500 x exp (- 6.5% x 5) = $8,008. Study Session: 1 - RA: 1 202. Correct answer: D This problem requires the use of binomial probability function, P(X=x) = fact(n) / fact(n - x) / fact(x) * p^x * (1-p)^(n-x) = fact(12) / fact(12 - 8) / fact(8) * 0.6^8 * 0.4^4 = 21.3%. Study Session: 1 - RA: 2 203. Correct answer: A Using the multiplication rule the number of packages = Number of golf sets x Number of ways of choosing three golf balls out of the possible five types. The second variable in the above equation can be calculated by using combinations formula = n! / (n-r!) / r! = 5! / 2! / 3! = 10 Therefore, the number of packages = 2 x 10 = 20. Study Session: 1 - RA: 2 204. Correct answer: B The movement of S&P500 as described by the analyst is a binomial process, i.e. the S&P500 can go up with a probability of 0.467 or go down with a probability of 0.533. Carrying out this process for 275 days the variance = n x p x (1 - p) = 275 x 0.467 x 0.533 = 68.451. Therefore the standard deviation = 68.451^0.5 = 8.274. Study Session: 1 - RA: 2 205. Correct answer: D Lognormal distribution is most appropriate for stock prices since it does not allow for negative values and implies that the returns are normally distributed. Study Session: 1 - RA: 2 206. Correct answer: B

http://64.106.152.240/inmarkets/passpro/index.cfm?event=questions....

2004-08-02

PASS PRO - Copyright 2004 Inmarkets Ltd - Licensed to Raymond chen

25/150 1

A lognormal distribution cuts off at zero on the left-hand side and extends to infinity on the right-hand side. Study Session: 1 - RA: 2 207. Correct answer: B A lognormal distribution cuts off at zero on the left-hand side and extends to infinity on the right-hand side. Study Session: 1 - RA: 2 208. Correct answer: A The range of the 95 percent confidence interval = mean ? 1.96 x standard deviation / Number of employees^0.5 = 32 ? 1.96 x 12 = 32 ? 23.5 = between 8.5 and 55.5 Study Session: 1 - RA: 2 209. Correct answer: B Given that the daily standard deviation is $8 million, the standard deviation over five days = $8 million x (5/1)^0.5 = $17.89 million. Given that the returns are normally distributed, we know that 99% of the outcomes will be above 2.325 standard deviations below the mean, i.e. above $18.4 million. Study Session: 1 - RA: 2 210. Correct answer: D The characteristics of a normal distribution are: It is a continuous distribution. It is bell shaped. It is symmetrical about the mean. It peaks at the mean expected value. It extends theoretically from negative infinity to positive infinity (the probability asymptotically approaches zero at plus and minus infinity). It has a skewness of zero (i.e. it is symmetric). It has a kurtosis (the level of peakedness) of three. Below three the distribution is platykurtic (too flat) and above three it is leptokurtic (too tall). Study Session: 1 - RA: 2 211. Correct answer: C Confidence interval = 1 - CND (Quantile) = Area under the normal distribution to the right of the given Quantile point. Some of these points are rather important due to their use in VAR calculations. Confidence Level 90% 95% 99% Quantile (alpha) -1.282 -1.645 -2.326

http://64.106.152.240/inmarkets/passpro/index.cfm?event=questions....

2004-08-02

PASS PRO - Copyright 2004 Inmarkets Ltd - Licensed to Raymond chen

26/150 1

Note: CND = Cumulative Normal Distribution function. Study Session: 1 - RA: 2

212. Correct answer: D Covariance = Correlation x Standard Deviation_A x Standard Deviation_B. Therefore Standard Deviation_B = Covariance / Correlation / Standard Deviation_A = 16 / 0.5 / 8^0.5 = 11.3137. Therefore Variance_B = 11.3137^2 = 128. Study Session: 1 - RA: 2 213. Correct answer: B The range of the 95 percent confidence interval = mean 1.96 x standard deviation / Number of employees^0.5 = 35 1.96 x 15 / 80^0.5 = 35 3.3 = between 31.7 and 38.3 Study Session: 1 - RA: 2 214. Correct answer: D Covariance = Correlation x Standard Deviation_A x Standard Deviation_B. Therefore Standard Deviation_B = Covariance / Correlation / Standard Deviation_A = 16 / 0.5 / 8^0.5 = 11.3137. Therefore Variance_B = 11.3137^2 = 128. Study Session: 1 - RA: 2 215. Correct answer: A Confidence interval = 1 - CND (Quantile) = Area under the normal distribution to the right of the given Quantile point. Some of these point are rather important due to their use in VAR calculations. Confidence Level 90% 95% 99% Quantile (alpha) 1.282 1.645 2.326

http://64.106.152.240/inmarkets/passpro/index.cfm?event=questions....

2004-08-02

PASS PRO - Copyright 2004 Inmarkets Ltd - Licensed to Raymond chen

27/150 1

Note: CND = Cumulative Normal Distribution function Study Session: 1 - RA: 2

216. Correct answer: B The characteristics of a normal distribution are: It is a continuous distribution. It is bell shaped. It is symmetrical about the mean. It peaks at the mean expected value. It extends theoretically from negative infinity to positive infinity (the probability asymptotically approaches zero at plus and minus infinity). It has a skewness of zero (i.e. it is symmetric). It has a kurtosis (the level of peakedness) of three. Below three the distribution is platykurtic (too flat) and above three it is leptokurtic (too tall). Study Session: 1 - RA: 2 217. Correct answer: C A return of 0 percent and 10 percent is between -2 standard deviation and -1 standard deviation of the mean, i.e. a quartile of -2 to -1. The confidence levels corresponding to these quartiles are 97.7% and 84.1%. Hence the probability that the return falls between these limits = 97.7% - 84.1% = 13.6% Study Session: 1 - RA: 2 218. Correct answer: B Given that the daily standard deviation is $9.62 million, the standard deviation over 5 days = $9.62 million x (5/1)^0.5 = $21.51 million. Using this standard deviation, the level of $20 million is 2.32 standard deviations [= (70 20)/21.51] from the mean value. Given that the returns are normally distributed, the probability of the value falling more than 2.32 standard deviations from the mean value is 1% (since 98% of the probability falls between 2.32 standard deviations of the mean). Study Session: 1 - RA: 2 219. Correct answer: D

http://64.106.152.240/inmarkets/passpro/index.cfm?event=questions....

2004-08-02

PASS PRO - Copyright 2004 Inmarkets Ltd - Licensed to Raymond chen

28/150 1

Covariance = Correlation x Standard Deviation_A x Standard Deviation_B. Therefore Standard Deviation_B = Covariance / Correlation / Standard Deviation_A = 5 / (0.5 x 8^0.5) = 3.5355. Therefore Variance_B = 3.5355^2 = 12.5. Study Session: 1 - RA: 2 220. Correct answer: D Increasing volatility will accenuate the right skew of the lognormal price distribution. Thus if the mean is held constant the median must shift to the left. Study Session: 1 - RA: 2 221. Correct answer: D Covariance = Correlation x Standard Deviation_A x Standard Deviation_B. Therefore Standard Deviation_B = Covariance / Correlation / Standard Deviation_A = 4.2 / 0.6 / 5^0.5 = 3.1305. Therefore, Variance_B = 3.1305^2 = 9.80. Study Session: 1 - RA: 2 222. Correct answer: D Given that the daily standard deviation is $2.5 million, the standard deviation over 14 days = $2.5 million x (14/1)^0.5 = $9.35 million. Given that the returns are normally distributed, we know that 95% of the outcomes will be above 1.645 standard deviations below the mean, i.e. above $84.6 million. Study Session: 1 - RA: 2 223. Correct answer: A Using the multiplication rule the number of packages = Number of golf sets x Number of ways of choosing three golf balls out of the possible five types. The second variable in the above equation can be calculated by using combinations formula = n! / (n-r!) / r! = 5! / 2! / 3! = 10 Therefore the number of packages = 2 x 10 = 20. Study Session: 1 - RA: 2 224. Correct answer: A Using the multiplication rule the number of packages = Number of golf sets x Number of ways of choosing two golf balls out of the possible three types. The second variable in the above equation can be calculated by using combinations formula = n! / (n-r!) / r! = 3! / 1! / 2! = 3 Therefore the number of packages = 3 x 3 = 9.

http://64.106.152.240/inmarkets/passpro/index.cfm?event=questions....

2004-08-02

PASS PRO - Copyright 2004 Inmarkets Ltd - Licensed to Raymond chen

29/150 1

Study Session: 1 - RA: 2 225. Correct answer: B Step 1. Calculate the mean stock price = (33 + 43 + 45 + 48 + 46) / 5 = 43. Step 2. Calculate the mean S&P level = (1150 + 1125 + 1140 + 1160 + 1170) / 5 = 1149. Step 3. The covariance between the stock and S&P = [(33 - 43)(1150 - 1149) + (43 - 43)(1125 - 1149) + (45 - 43)(1140 - 1149) + (48 - 43)(1160 - 1149) + (46 - 43)(1170 - 1149)] / 5 = 18. Study Session: 1 - RA: 2 226. Correct answer: A Using the multiplication rule the number of packages = Number of golf sets x Number of ways of choosing three golf balls out of the possible five types. The second variable in the above equation can be calculated by using combinations formula = n! / (n-r!) / r! = 5! / 2! / 3! = 10 Therefore the number of packages = 2 x 10 = 20. Study Session: 1 - RA: 2 227. Correct answer: C The rate offered to the investor is the stated rate in nominal terms. The periodic rate will be 14.49% / 12 = 1.21%, and the effective rate will be (1 + 14.49%/12)^12 - 1 = 15.49%. Study Session: 1 - RA: 3 228. Correct answer: D The relationship between NPV and discount rate depends on the nature of the cash flows, i.e. whether they are initially negative and then positive or vice versa. Study Session: 1 - RA: 3 229. Correct answer: A The value of this annuity at the start of the year 5 = Coupon x [1 - 1/(1 + i)^N]/i = 6,000 x [1 - 1/(1 + 6%)^15] / 6% = $58,273. To find the value of this amount in today's month, we need to discount it back four years. Thus, present value of this annuity = $58,273 / (1 + 6%)^4 = $46,158. Study Session: 1 - RA: 3 230. Correct answer: A The value of this annuity at the start of the year 2 = Coupon x [1 - 1/(1 + i)^N]/i = 900 x [1 1/(1 + 8%)^25] / 8% = $9,607. To find the value of this amount in today's month we need to discount it back 1 years. Thus, present value of this annuity = $9,607 / (1 + 8%)^1 = $8,895.

http://64.106.152.240/inmarkets/passpro/index.cfm?event=questions....

2004-08-02

PASS PRO - Copyright 2004 Inmarkets Ltd - Licensed to Raymond chen

30/150 1

Study Session: 1 - RA: 3 231. Correct answer: B Using continuous compounding, PV = FV x exp(-Rate x Time period) = $4,700 x exp (- 7% x 9/12) = $4,460. Study Session: 1 - RA: 3 232. Correct answer: B The price of this bond is approximately equal to 100 / (1 + 0.05) ^ 2, which is equal to about 91. Study Session: 1 - RA: 3 233. Correct answer: A Using continuous compounding, PV = FV x exp(-Rate x Time period) = $7,000 x exp (- 8% x 8/12) = $6,636. Study Session: 1 - RA: 3 234. Correct answer: A The value of this annuity at the start of the year 2 = Coupon x [1 - 1/(1 + i)^N]/i = 2,000 x [1 - 1/(1 + 4%)^8] / 4% = $13,465. To find the value of this amount in today's month, we need to discount it back one year. Thus, present value of this annuity = $13,465 / (1 + 4%)^1 = $12,947. Study Session: 1 - RA: 3 235. Correct answer: B Rate of inflation = (Rise in prices)^(1/Number of years) - 1 = (2)^(1/15) - 1 = 4.73%. Study Session: 1 - RA: 3 236. Correct answer: D Without any negative cash flow in the stream, the rate of return is infinite. So, the IRR of this investment cannot be determined. Study Session: 1 - RA: 3 237. Correct answer: A The value of this annuity at the start of the year 5 = Coupon x [1 - 1/(1 + i)^N]/i = 6,000 x [1 - 1/(1 + 6%)^15] / 6% = $58,273. To find the value of this amount in today's month, we need to discount it back four years. Thus, present value of this annuity = $58,273 / (1 + 6%)^4 = $46,158. Study Session: 1 - RA: 3 238. Correct answer: A

http://64.106.152.240/inmarkets/passpro/index.cfm?event=questions....

2004-08-02

PASS PRO - Copyright 2004 Inmarkets Ltd - Licensed to Raymond chen

31/150 1

Continuously compounded rate = ln(FV/PV)/N = ln(15,000 / 12,000) / 2 = 11.16%. Study Session: 1 - RA: 3 239. Correct answer: D $1 million is a relatively small amount and the liquidity risk is not high in most markets. All other factors are crucial for the decision. Study Session: 1 - RA: 3 240. Correct answer: A The value of this annuity at the start of the year 15 = Coupon x [1 - 1/(1 + i)^N]/i = 1500 x [1 - 1/(1 + 8%)^10] / 8% = $10,065. To find the value of this amount in today's month, we need to discount it back 14 years. Thus, present value of this annuity = $10,065 / (1 + 8%)^14 = $3,427. Study Session: 1 - RA: 3 241. Correct answer: D $1 million is a relatively small amount and the liquidity risk is not high in most markets. All other factors are crucial for the decision. Study Session: 1 - RA: 3 242. Correct answer: D Effective rate = (1 + Stated rate / Periodicity)^Periodicity -1 = (1 + 7.75%/12)^12 -1 = 8.03%. Study Session: 1 - RA: 3 243. Correct answer: B Continuously compounded rate = ln(FV/PV)/N = ln(10,000 / 7,500) / 5 = 5.75%. Study Session: 1 - RA: 3 244. Correct answer: B Step 1. Calculate w = 45 / 180 = 0.25. Step 2. This bond has cash flows of 2.125, 2.125, 102.125 after 0.25, 1.25, 2.25 periods respectively. Discounting these cash flows at the yield of 3.975% / 2 = 1.9875%, we get the present values: 2.115, 2.073, 97.702. Adding these up, the full value of this bond is $101.89. Step 3. The accrued coupon = coupon x (1 - w) = 100 x 4.25% / 2 x (1 - 0.25) = $1.594. Step 4. Deducting this accrued coupon from the full price gives the clean price = $100.296.

http://64.106.152.240/inmarkets/passpro/index.cfm?event=questions....

2004-08-02

PASS PRO - Copyright 2004 Inmarkets Ltd - Licensed to Raymond chen

32/150 1

Study Session: 1 - RA: 3 245. Correct answer: D The relationship between NPV and discount rate depends on the nature of the cash flows, i.e. whether they are initially negative and then positive or vice versa. Study Session: 1 - RA: 3 246. Correct answer: B Using continuous compounding, PV = FV x exp(-Rate x Time period) = $8,400 x exp (- 8% x 6/12) = $8,071. Study Session: 1 - RA: 3 247. Correct answer: D Effective rate = (1 + Stated rate / Periodicity)^Periodicity -1 = (1 + 7.75%/12)^12 -1 = 8.03%. Study Session: 1 - RA: 3 248. Correct answer: C FV of this payment = $4,000 x (1 + 7%)^10 = $7,869. Study Session: 1 - RA: 3 249. Correct answer: D Effective rate = (1 + Stated rate / Periodicity)^Periodicity -1 = (1 + 9%/12)^12 -1 = 9.38%. Study Session: 1 - RA: 3 250. Correct answer: A Continuously compounded rate = ln(FV/PV)/N = ln(15,000 / 12,000) / 2 = 11.16%. Study Session: 1 - RA: 3 251. Correct answer: B Rate of inflation = (Rise in prices)^(1/Number of years) - 1 = (2)^(1/8) - 1 = 9.05%. Study Session: 1 - RA: 3 252. Correct answer: B Firstly, we calculate the Mean = -4 x 40% + 8 x 25% + 15 x 35% = 5.65 Then, Variance = (5.65 - -4)^2 x 40% + (5.65 - 8)^2 x 25% + (5.65 - 15)^2 x 35% = 69.23 Finally, Standard deviation = 69.23^0.5 = 8.32. Study Session: 1 - RA: 4

http://64.106.152.240/inmarkets/passpro/index.cfm?event=questions....

2004-08-02

PASS PRO - Copyright 2004 Inmarkets Ltd - Licensed to Raymond chen

33/150 1

253. Correct answer: B Volatility scales as the square root of time. Therefore the 250-day volatility = 10-day volatility x (250 / 10)^0.5 = 1.8% x 5 = 9%. Study Session: 1 - RA: 4 254. Correct answer: C Since the correlation is 1, the return on Asset B should be the same proportion as the ratio of volatility of B to A, i.e. Return on Asset B = (Volatility B / Volatility A) x Return on Asset = (25% /15%) x 1.50% = 2.50%. Study Session: 1 - RA: 4 255. Correct answer: C Correlation coefficient is a measure of the linear relationship between two random variables. It can be calculated by scaling the covariance between them and varies between -1 (perfect negative correlation) to +1 (perfect positive correlation). If the variables are independent they will have a correlation coefficient of zero, but the reverse does not always hold true (i.e. a correlation coefficient of zero does not necessarily mean that they are independent). Covariance between the variables = Correlation coefficient x Standard deviation of the first variable x Standard deviation of the second variable. This calculation does not require the use of mean at all. Study Session: 1 - RA: 4 256. Correct answer: B Firstly, we calculate the Mean = -6 x 15% + 6 x 40% + 12 x 45% = 6.9. Then, Variance = (6.9 - -6)^2 x 15% + (6.9 - 6)^2 x 40% + (6.9 - 12)^2 x 45% = 36.98 Finally, Standard deviation = 36.98^0.5 = 6.08. Study Session: 1 - RA: 4 257. Correct answer: D Mean = -10 x 30% + 5 x 40% + 25 x 30% = 6.5. Study Session: 1 - RA: 4 258. Correct answer: D Correlation coefficient is a measure of the linear relationship between two random variables. It can be calculated by scaling the covariance between them and varies between -1 (perfect negative correlation) to +1 (perfect positive correlation). If the variables are independent they will have a correlation coefficient of zero, but the reverse does not always hold true (i.e. a correlation coefficient of zero does not necessarily mean that they are independent). Covariance between the variables = Correlation coefficient x Standard deviation of the first variable x Standard deviation of the second variable. This calculation does not require the use of mean at all.

http://64.106.152.240/inmarkets/passpro/index.cfm?event=questions....

2004-08-02

PASS PRO - Copyright 2004 Inmarkets Ltd - Licensed to Raymond chen

34/150 1

Study Session: 1 - RA: 4 259. Correct answer: C Mean = -8 x 25% + 12 x 40% + 24 x 35% = 11.2. Study Session: 1 - RA: 4 260. Correct answer: D Covariance = Correlation x Standard Deviation_A x Standard Deviation_B. Therefore Standard Deviation_B = Covariance / Correlation / Standard Deviation_A = 5 / 0.5 / 8^0.5 = 3.5355. Therefore, Variance_B = 3.5355^2 = 12.5. Study Session: 1 - RA: 4 261. Correct answer: B Firstly, we calculate the Mean = -8 x 25% + 12 x 40% + 24 x 35% = 11.2 Then, Variance = (11.2 - -8)^2 x 25% + (11.2 - 12)^2 x 40% + (11.2 - 24)^2 x 35% = 149.76 Finally, Standard deviation = 149.76^0.5 = 12.24. Study Session: 1 - RA: 4 262. Correct answer: D Correlation coefficient is a measure of the linear relationship between two random variables. It can be calculated by scaling the covariance between them and varies between -1 (perfect negative correlation) to +1 (perfect positive correlation). If the variables are independent they will have a correlation coefficient of zero, but the reverse does not always hold true (a correlation coefficient of zero does not necessarily mean that they are independent). Covariance between the variables = Correlation coefficient x Standard deviation of the first variable x Standard deviation of the second variable. This calculation does not require the use of mean at all. Study Session: 1 - RA: 4 263. Correct answer: B First we calculate the Mean = -4 x 45% + 8 x 40% + 25 x 15% = 5.15 Then, Variance = (5.15 - -4)^2 x 45% + (5.15 - 8)^2 x 40% + (5.15 - 25)^2 x 15% = 100.03 Finally, Standard deviation = 100.03^0.5 = 10. Study Session: 1 - RA: 4 264. Correct answer: C

http://64.106.152.240/inmarkets/passpro/index.cfm?event=questions....

2004-08-02

PASS PRO - Copyright 2004 Inmarkets Ltd - Licensed to Raymond chen

35/150 1

Correlation coefficient is a measure of linear relationship between two random variables. It can be calculated by scaling the covariance between them and varies between -1 (perfect negative correlation) to +1 (perfect positive correlation). If the variables are independent they will have a correlation coefficient of zero, but the reverse does not always hold true (i.e. a correlation coefficient of zero does not necessarily mean that they are independent). Covariance between the variables = Correlation coefficient x Standard deviation of the first variable x Standard deviation of the second variable. This calculation does not require the use of mean at all. Study Session: 1 - RA: 4 265. Correct answer: C Mean = -8 x 25% + 12 x 40% + 24 x 35% = 11.2. Study Session: 1 - RA: 4 266. Correct answer: B Firstly, we calculate the Mean = -6 x 15% + 6 x 40% + 12 x 45% = 6.9. Then, Variance = (6.9 - -6)^2 x 15% + (6.9 - 6)^2 x 40% + (6.9 - 12)^2 x 45% = 36.98 Finally, Standard deviation = 36.98^0.5 = 6.08. Study Session: 1 - RA: 4 267. Correct answer: A Correlation coefficient is a measure of the linear relationship between two random variables. It can be calculated by scaling the covariance between them and varies between -1 (perfect negative correlation) to +1 (perfect positive correlation). If the variables are independent they will have a correlation coefficient of zero, but the reverse does not always hold true (i.e. a correlation coefficient of zero does not necessarily mean that they are independent). Covariance between the variables = Correlation coefficient x Standard deviation of the first variable x Standard deviation of the second variable. This calculation does not require the use of mean at all. Study Session: 1 - RA: 4 268. Correct answer: B Volatility scales as the square root of time. Therefore, the 23-day volatility = 10-day volatility x (23 / 10)^0.5 = 1.4% x 1.5166 = 2.12%. Study Session: 1 - RA: 4 269. Correct answer: A Correlation coefficient is a measure of the linear relationship between two random variables. It can be calculated by scaling the covariance between them and varies between -1 (perfect negative correlation) to +1 (perfect positive correlation). If the variables are independent they will have a correlation coefficient of zero, but the reverse does not always hold true (i.e. a correlation coefficient of zero does not necessarily mean that they are independent).

http://64.106.152.240/inmarkets/passpro/index.cfm?event=questions....

2004-08-02

PASS PRO - Copyright 2004 Inmarkets Ltd - Licensed to Raymond chen

36/150 1

Covariance between the variables = Correlation coefficient x Standard deviation of the first variable x Standard deviation of the second variable. This calculation does not require the use of mean at all. Study Session: 1 - RA: 4 270. Correct answer: C Since the correlation is 1, the return on Asset B should be the same proportion as the ratio of volatility of B to A, i.e. Return on Asset B = (Volatility B / Volatility A) x Return on Asset = (25% /15%) x 1.50% = 2.50%. Study Session: 1 - RA: 4 271. Correct answer: B Volatility scales as the square root of time. Therefore, the 30-day volatility = 7-day volatility x (30 / 7)^0.5 = 1.05% x 2.0702 = 2.17%. Study Session: 1 - RA: 4 272. Correct answer: D Mean = -2 x 60% + 10 x 20% + 20 x 20% = 4.8. Study Session: 1 - RA: 4 273. Correct answer: B Firstly, we calculate the Mean = -4 x 40% + 8 x 25% + 15 x 35% = 5.65 Then, Variance = (5.65 - -4)^2 x 40% + (5.65 - 8)^2 x 25% + (5.65 - 15)^2 x 35% = 69.23 Finally, Standard deviation = 69.23^0.5 = 8.32. Study Session: 1 - RA: 4 274. Correct answer: B Firstly, we calculate the Mean = -2 x 60% + 10 x 20% + 20 x 20% = 4.8 Then, Variance = (4.8 - -2)^2 x 60% + (4.8 - 10)^2 x 20% + (4.8 - 20)^2 x 20% = 79.36 Finally, Standard deviation = 79.36^0.5 = 8.91. Study Session: 1 - RA: 4 275. Correct answer: B Volatility scales as the square root of time. Therefore, the 30-day volatility = 7-day volatility x (30 / 7)^0.5 = 1.05% x 2.0702 = 2.17%. Study Session: 1 - RA: 4 276. Correct answer: B

http://64.106.152.240/inmarkets/passpro/index.cfm?event=questions....

2004-08-02

PASS PRO - Copyright 2004 Inmarkets Ltd - Licensed to Raymond chen

37/150 1

Firstly, we calculate the Mean = -15 x 30% + 5 x 40% + 25 x 30% = 5 Then, Variance = (5 - -15)^2 x 30% + (5 - 5)^2 x 40% + (5 - 25)^2 x 30% = 240 Finally, Standard deviation = 240^0.5 = 15.49. Study Session: 1 - RA: 4 277. Correct answer: A Coefficient of variation (CV) is a measure of relative risk and is calculated as: CV = (standard deviation of returns) / (expected rate of return). The calculations for each investment are shown below: CV(A) = 0.004^0.5 / 0.18 = 0.351. CV(B) = 0.005^0.5 / 0.25 = 0.283. CV(C) = 0.04 / 0.12 = 0.333. CV(D) = 0.025 / 0.08 = 0.313. Study Session: 1 - RA: 5 278. Correct answer: D Coefficient of variation (CV) is a measure of relative risk and is calculated as: CV = (standard deviation of returns) / (expected rate of return). The calculations for each investment are shown below: CV(A) = 0.005^0.5 / 0.12 = 0.589. CV(B) = 0.002^0.5 / 0.1 = 0.447. CV(C) = 0.03 / 0.15 = 0.2. CV(D) = 0.05 / 0.06 = 0.833. Study Session: 1 - RA: 5 279. Correct answer: D The null hypothesis for this test is that the coefficient is equal to zero. Therefore, the test statistic = (Observed value - 0) / Standard error = (6% - 0) / 3% = 2. Study Session: 1 - RA: 5 280. Correct answer: C Coefficient of variation (CV) is a measure of relative risk and is calculated as: CV = (standard deviation of returns) / (expected rate of return). The calculations for each investment are shown below: CV(A) = 0.005^0.5 / 0.24 = 0.295. CV(B) = 0.002^0.5 / 0.18 = 0.248. CV(C) = 0.05 / 0.15 = 0.333. CV(D) = 0.08 / 0.25 = 0.32. Study Session: 1 - RA: 5 281. Correct answer: C Standard error of the mean = s /vn = 4/v40 = 0.63.

http://64.106.152.240/inmarkets/passpro/index.cfm?event=questions....

2004-08-02

PASS PRO - Copyright 2004 Inmarkets Ltd - Licensed to Raymond chen

38/150 1

Study Session: 1 - RA: 5 282. Correct answer: A The null hypothesis for this test is that the coefficient is equal to zero. Therefore the test statistic = (Observed value - 0) / Standard error = (15% - 0) / 9% = 1.67. Study Session: 1 - RA: 5 283. Correct answer: A Coefficient of variation (CV) is a measure of relative risk and is calculated as: CV = (standard deviation of returns) / (expected rate of return). The calculations for each investment are shown below: CV(A) = 0.004^0.5 / 0.18 = 0.351. CV(B) = 0.005^0.5 / 0.25 = 0.283. CV(C) = 0.04 / 0.12 = 0.333. CV(D) = 0.025 / 0.08 = 0.313. Study Session: 1 - RA: 5 284. Correct answer: D The null hypothesis for this test is that the coefficient is equal to zero. Therefore the test statistic = (Observed value - 0) / Standard error = (12.25% - 0) / 8% = 1.53. Study Session: 1 - RA: 5 285. Correct answer: C Coefficient of variation (CV) is a measure of relative risk and is calculated as: CV = (standard deviation of returns) / (expected rate of return). The calculations for each investment are shown below: CV(A) = 0.005^0.5 / 0.24 = 0.295. CV(B) = 0.002^0.5 / 0.18 = 0.248. CV(C) = 0.05 / 0.15 = 0.333. CV(D) = 0.08 / 0.25 = 0.32. Study Session: 1 - RA: 5 286. Correct answer: C Standard error of the mean = s / n^0.5 = 3.5 / 30^0.5 = 0.64. Study Session: 1 - RA: 5 287. Correct answer: A The null hypothesis for this test is that the coefficient is equal to zero. Therefore the test statistic = (Observed value - 0) / Standard error = (15% - 0) / 9% = 1.67. Study Session: 1 - RA: 5 288. Correct answer: D

http://64.106.152.240/inmarkets/passpro/index.cfm?event=questions....

2004-08-02

PASS PRO - Copyright 2004 Inmarkets Ltd - Licensed to Raymond chen

39/150 1

The null hypothesis for this test is that the coefficient is equal to zero. Therefore the test statistic = (Observed value - 0) / Standard error = (9% - 0) / 3.25% = 2.77. Study Session: 1 - RA: 5 289. Correct answer: C In this distribution, the lowest element is 1.5 percent below the mean whereas the highest element is 2.5 percent above the mean. Thus the distribution is skewed towards the right. Study Session: 1 - RA: 5 290. Correct answer: D Step 1. The returns for the three scenarios given are: 42.857% [= (50 - 35)/35], 20% [= (42 35) / 35], and -42.857% [= (20 - 35) / 35] Step 2. Calculate expected price = 33% x 42.857% + 20% x 20% + 47% x -42.857% = -2%. Step 3. Calculate Variance = 33% x (-2% - 42.857%)^2 + 20% x (-2% - 20%)^2 + 47% x (2% + 42.857%)^2 = 0.154538 Step 4. Calculate volatility/standard deviation = 0.154538^0.5 = 39.31%. Study Session: 1 - RA: 5 291. Correct answer: D The null hypothesis for this test is that the coefficient is equal to zero. Therefore the test statistic = (Observed value - 0) / Standard error = (11.8% - 0) / 3% = 3.93. Study Session: 1 - RA: 5 292. Correct answer: C Standard error of the mean = s /n^0.5 = 7/60^0.5 = 0.9. Study Session: 1 - RA: 5 293. Correct answer: C Standard error of the mean = s /n^0.5 = 6/50^0.5 = 0.85. Study Session: 1 - RA: 5 294. Correct answer: D The null hypothesis for this test is that the coefficient is equal to zero. Therefore, the test statistic = (Observed value - 0) / Standard error = (6% - 0) / 3% = 2. Study Session: 1 - RA: 5 295. Correct answer: A Coefficient of variation (CV) is a measure of relative risk and is calculated as: CV = (standard deviation of returns) / (expected rate of return). The calculations for each investment are

http://64.106.152.240/inmarkets/passpro/index.cfm?event=questions....

2004-08-02

PASS PRO - Copyright 2004 Inmarkets Ltd - Licensed to Raymond chen

40/150 1

shown below: CV(A) = 0.005^0.5 / 0.19 = 0.372. CV(B) = 0.002^0.5 / 0.15 = 0.298. CV(C) = 0.03 / 0.1 = 0.3. CV(D) = 0.01 / 0.03 = 0.333. Study Session: 1 - RA: 5 296. Correct answer: B The null hypothesis for this test is that the coefficient is equal to zero. Therefore the test statistic = (Observed value - 0) / Standard error = (13.6% - 0) / 5% = 2.72. Study Session: 1 - RA: 5 297. Correct answer: A Coefficient of variation (CV) is a measure of relative risk and is calculated as: CV = (standard deviation of returns) / (expected rate of return). The calculations for each investment are shown below: CV(A) = 0.002^0.5 / 0.1 = 0.447. CV(B) = 0.003^0.5 / 0.15 = 0.365. CV(C) = 0.03 / 0.08 = 0.375. CV(D) = 0.01 / 0.05% = 0.2. Study Session: 1 - RA: 5 298. Correct answer: B The null hypothesis for this test is that the coefficient is equal to zero. Therefore the test statistic = (Observed value - 0) / Standard error = (13.6% - 0) / 5% = 2.72. Study Session: 1 - RA: 5 299. Correct answer: B In hypothesis testing, Type I error is rejecting the null hypothesis when it is actually true. The probability of Type I error is called the "level of significance", so choosing a lower level of significance lowers the probability of Type I error. Type II error happens if the null hypothesis is not rejected when it is actually false. The power of the test is the probability of correctly rejecting the null hypothesis (when it is false), so minimizing Type II errors would maximize the power of the test. Study Session: 1 - RA: 6 300. Correct answer: C Economic significance and statistical significance are not the same. Economic significance takes into account statistical significance but also a number of other issues (externalities such as transaction costs, regulations etc.) Note: the wording of the choices is a bit tricky. Choice A is right in some cases, but in many situations statistical significance can suggest economic significance, although it cannot confirm significance./p> Study Session: 1 - RA: 6

http://64.106.152.240/inmarkets/passpro/index.cfm?event=questions....

2004-08-02

PASS PRO - Copyright 2004 Inmarkets Ltd - Licensed to Raymond chen

41/150 1

301. Correct answer: A Tests of the variance of a population require the chi-squared test. For this data chi-squared = (n - 1) x Sample variance / Hypothesized variance = 29 x 0.12^2 / 0.2^2 = 10.44. Since the analyst wants to show that the standard deviation is less than 20%, this will be chosen as the alternative hypothesis and the null hypothesis will be that the standard deviation is higher than or equal to 20%. The critical value of the chi-squared statistic (for df = 29 and p= 95%) is 17.71. Since the test statistic is lower than the critical statistic, we can reject the null hypothesis that the standard deviation is equal to or higher than 20%, and accept the alternative hypothesis that the standard deviation is lower than 20%. Study Session: 1 - RA: 6 302. Correct answer: D In hypothesis testing we accept the alternate hypothesis if the null hypothesis has been rejected. Type I error happens if the null hypothesis is rejected when it is actually true. The probability of Type I error is called the level of significance, so choosing a lower level of significance lowers the probability of Type I error. Type II error happens if the null hypothesis is not rejected when it is actually false. Study Session: 1 - RA: 6 303. Correct answer: A This is a hypothesis test with the null hypothesis: performance of firms with a P/B ratio 25 percent below average is "less than or equal to" the market. If the calculated value of the test statistic is higher than the critical value, we reject the null hypothesis and accept the alternative hypothesis. Study Session: 1 - RA: 6 304. Correct answer: C Tests of differences between variances of two populations require the F-test. The value of the test F-statistic = Higher variance / Lower variance = 0.13^2 / 0.1^2 = 1.69. Study Session: 1 - RA: 6 305. Correct answer: B Tests of the variance of a population require the chi-squared test. For this data, chi-squared = (n - 1) x Sample variance / Hypothesized variance = 50 x 0.24^2 / 0.04 = 72. Since the analyst wants to show that the variance is more than 0.04, this will be chosen as the alternative hypothesis and the null hypothesis will be that the variance is lower than or equal to 0.04. The critical value of the chi-squared statistic (for df=50 and p=0.05) is 67.505. Since the test statistic is higher than the critical statistic, we can reject the null hypothesis (variance <= 0.04), and accept the alternative hypothesis (variance > 0.04). Study Session: 1 - RA: 6 306. Correct answer: B Population mean and probability distribution are usually unknown, which is why hypothesis testing has to be undertaken. The steps in the hypothesis testing process are: 1) State a hypothesis. 2) Identify the test statistic and its probability distribution. 3) Specify the significance level. 4) State the decision rule. 5) Collect data and perform the calculations. 6) Make the statistical decision. 7) Make the investment/economic decision based on the above.

http://64.106.152.240/inmarkets/passpro/index.cfm?event=questions....

2004-08-02

PASS PRO - Copyright 2004 Inmarkets Ltd - Licensed to Raymond chen

42/150 1

Study Session: 1 - RA: 6 307. Correct answer: A Tests of the variance of a population require the chi-squared test. Since the analyst wants to show that the standard deviation is less than 20%, this will be chosen as the alternative hypothesis and the null hypothesis will be that the standard deviation is higher than or equal to 20%. Thus using a probability in the right tail of 95% and degrees of freedom of 29, the critical value from the chi-squared table is 17.71. Study Session: 1 - RA: 6 308. Correct answer: C This is a two-tailed t-test for the difference between means of two independent populations. Step 1. The common variance = [(n1 -1) x Variance1 + (n2 - 1) x Variance2] / (n1 + n2 - 2) = [(21 -1) x 0.12^2 + (41 - 1) x 0.13^2] / (21 + 41 - 2) = 0.0161. Step 2. The value of the test statistic = (Mean1 - Mean2) / (Common variance/n1 + Common variance/n2)^0.5 = (0.15 0.12) / (0.0161/21 + 0.0161/41)^0.5 = 0.88. Using degrees of freedom = n1 + n2 - 2 = 21 + 41 - 2 = 60 and level of significance = alpha / 2 = 0.025 the critical value = 2.0. Since the test statistic is lower than the critical statistic, we cannot reject the null hypothesis that the mean returns are same. Study Session: 1 - RA: 6 309. Correct answer: A Tests of the variance of a population require the chi-squared test. For this data chi-squared = (n - 1) x Sample variance / Hypothesized variance = 39 x 0.11^2 / 0.2^2 = 11.8. Study Session: 1 - RA: 6 310. Correct answer: C The required test for testing the variance is the chi-squared test. The test statistic = (n - 1) x Sample variance / Hypothesized variance = 14 x 0.1^2 / 0.15^2 = 6.22. To test whether the standard deviation is different (Ho: standard deviation is equal to 15%), the critical values of chi-squared will be 5.63 and 26.12 (using df=14, p-lower=0.975 and p-higher=0.025). Since the test statistic falls inside the critical range, the analyst cannot reject the null hypothesis . Study Session: 1 - RA: 6 311. Correct answer: D Tests of the variance of a population require the chi-squared test. For this data chi-squared = (n - 1) x Sample variance / Hypothesized variance = 60 x 0.22^2 / 0.045 = 64.53. Since the analyst wants to show that the variance is more than 0.045, this will be chosen as the alternative hypothesis and the null hypothesis will be that the variance is lower than or equal to 0.045. The critical value of the chi-squared statistic (for df = 60 and p = 5%) is 79.08. Since the test statistic is lower than the critical statistic, we cannot reject the null hypothesis that the variance is equal to or lower than 0.045. Study Session: 1 - RA: 6 312. Correct answer: D This problem requires a two-tailed F-test. Looking up the F-table for 0.025 significance level,

http://64.106.152.240/inmarkets/passpro/index.cfm?event=questions....

2004-08-02

PASS PRO - Copyright 2004 Inmarkets Ltd - Licensed to Raymond chen

43/150 1

with 40 degrees of freedom in the numerator and 20 in the denominator (note the population with the higher variance goes in the numerator and that this is a two-sided test so p = 0.025), the critical value of F is 2.29. Study Session: 1 - RA: 6 313. Correct answer: C In hypothesis testing, we accept the alternate hypothesis if the null hypothesis has been rejected. Type I error happens if the null hypothesis is rejected when it is actually true. Type II error happens if the null hypothesis is not rejected when it is actually false. The power of the test is the probability of correctly rejecting the null hypothesis (when it is false), so minimizing Type II errors would maximize the power of the test. Study Session: 1 - RA: 6 314. Correct answer: B The 95% confidence interval = Mean 1.96 x Standard error = Mean 1.96 x Standard deviation / Sample size^0.5 = 20 1.96 x 8.5 / 50^0.5 = 17.64 to 22.36. Study Session: 1 - RA: 6 315. Correct answer: C This problem requires an F-test in which the null hypothesis Ho: variance of stock A = variance of stock B. The test statistic = higher variance / lower variance = 0.012 / 0.005 = 2.4. The critical F with 14 and 19 degrees of freedom and p=0.025 is 2.65. Since the test statistic is lower, the analyst cannot reject the null hypothesis. Study Session: 1 - RA: 6 316. Correct answer: A In hypothesis testing we accept the alternate hypothesis if the null hypothesis has been rejected. Type I error happens if the null hypothesis is rejected when it is actually true. Type II error happens if the null hypothesis is accepted when it is actually false. The power of the test is the probability of correctly rejecting the null hypothesis (when it is false), so minimizing Type II errors would maximize the power of the test. Study Session: 1 - RA: 6 317. Correct answer: A The required test for testing the variance is the chi-squared test. The test statistic = (n - 1) x Sample variance / Hypothesized variance = 100 x 0.1^2 / 0.18^2 = 30.86. To test whether the standard deviation is lower (Ho: standard deviation is higher than or equal to 18%), the critical value of chi-squared will be 77.93 (using df = 100 and p= 0.95). Since the test statistic is lower than the critical value, the analyst can reject the null hypothesis and conclude that the standard deviation of returns is lower than 18%. Study Session: 1 - RA: 6 318. Correct answer: C Since this is a one-tailed test with a 0.05 significance level and 14 degrees of freedom, the critical value from the t-table is 1.76.

http://64.106.152.240/inmarkets/passpro/index.cfm?event=questions....

2004-08-02

PASS PRO - Copyright 2004 Inmarkets Ltd - Licensed to Raymond chen

44/150 1

Study Session: 1 - RA: 6 319. Correct answer: C The null hypothesis is usually set to the opposite of what we are trying to proves, so that discrediting the null hypothesis confirms our belief. In this case the null hypothesis H0: performance of active portfolio managers < or = average for all portfolio managers. Study Session: 1 - RA: 6 320. Correct answer: A The 95% confidence interval = Mean 1.96 x Standard error = 18 1.96 x 7.25 = 3.79 to 32.21. Study Session: 1 - RA: 6 321. Correct answer: A The hypothesis test that the analyst will construct for this study is H0: Spending is equal to or less than $2,000; and Ha = Spending is more than $2,000. Thus we have only one rejection region (if the test statistic is higher than the critical statistic), i.e. this is a one-tailed test. Study Session: 1 - RA: 6 322. Correct answer: C This is a two-tailed t-test for the difference between means of two independent populations. Step 1. The common variance = [(n1 -1) x Variance1 + (n2 - 1) x Variance2] / (n1 + n2 - 2) = [(21 -1) x 0.13^2 + (41 - 1) x 0.15^2] / (21 + 41 - 2) = 0.0206. Step 2. The value of the test statistic = (Mean1 - Mean2) / (Common variance/n1 + Common variance/n2)^0.5 = (0.12 0.09) / (0.0206/21 + 0.0206/41)^0.5 = 0.78. Using degrees of freedom = n1 + n2 - 2 = 21 + 41 - 2 = 60 and level of significance = alpha / 2 = 0.025 the critical value = 2.0. Since the test statistic is lower than the critical statistic, we cannot reject the null hypothesis that the mean returns are same. Study Session: 1 - RA: 6 323. Correct answer: C The null hypothesis is usually set to the opposite of what we are trying to proves, so that discrediting the null hypothesis confirms our belief. In this case the null hypothesis H0: performance of active portfolio managers < or = average for all portfolio managers. Study Session: 1 - RA: 6 324. Correct answer: C Coefficient of determination = ESS / TSS = 761.562 / 865.333 = 0.88. Study Session: 1 - RA: 7 325. Correct answer: D The standard error = USS / (N-2) = (TSS - ESS) / (N-2) = (113,171.429 - 96,057.049) / (7-2) = 3,422.876.

http://64.106.152.240/inmarkets/passpro/index.cfm?event=questions....

2004-08-02

PASS PRO - Copyright 2004 Inmarkets Ltd - Licensed to Raymond chen

45/150 1

Study Session: 1 - RA: 7 326. Correct answer: D The percentage of explained variation = Correlation coefficient ^ 2 = 0.6 ^ 2 = 0.36 or 36%. Therefore, the unexplained variation is 64%. Study Session: 1 - RA: 7 327. Correct answer: B The percentage of explained variation = Correlation coefficient ^ 2 = 0.85 ^ 2 = 0.7225 or 72.25%. Therefore, the unexplained variation is 27.75%. Study Session: 1 - RA: 7 328. Correct answer: A The slope coefficient = Sum_XY / Sum_X^2 = -9,152.23 / 368.857 = -24.812. Study Session: 1 - RA: 7 329. Correct answer: D The percentage of explained variation = Correlation coefficient ^ 2 = 0.33 ^ 2 = 0.1089 or 10.89%. Therefore, the unexplained variation is 89.11%. Study Session: 1 - RA: 7 330. Correct answer: D The slope coefficient = Sum_XY / Sum_X^2 = 133.302 / 23.333 = 5.713. Study Session: 1 - RA: 7 331. Correct answer: D Correlation = (Coefficient of determination)^0.5 = (ESS / TSS)^0.5 = (96,057.049 / 113,171.429)^0.5 = 0.921. Study Session: 1 - RA: 7 332. Correct answer: C Coefficient of determination = ESS / TSS = 761.562 / 865.333 = 0.88. Study Session: 1 - RA: 7 333. Correct answer: C Coefficient of determination = ESS / TSS = 96,057.049 / 113,171.429 = 0.849. Study Session: 1 - RA: 7 334. Correct answer: D

http://64.106.152.240/inmarkets/passpro/index.cfm?event=questions....

2004-08-02

PASS PRO - Copyright 2004 Inmarkets Ltd - Licensed to Raymond chen

46/150 1

The standard error = USS / (N-2) = (TSS - ESS) / (N-2) = (865.333 - 761.562) / (6-2) = 25.94. Study Session: 1 - RA: 7 335. Correct answer: D The standard error = USS / (N-2) = (TSS - ESS) / (N-2) = (865.333 - 761.562) / (6-2) = 25.94. Study Session: 1 - RA: 7 336. Correct answer: D Correlation = (Coefficient of determination)^0.5 = (ESS / TSS)^0.5 = (3.312 / 3.617)^0.5 = 0.957. Study Session: 1 - RA: 7 337. Correct answer: D First we calculate the slope coefficient = Sum_XY / Sum_X^2 = -9,152.23 / 368.857 = 24.812. Then we calculate the intercept = Avg_Y - Slope x Avg X = 627.143 - -24.812 x 14.857 = 995.77. Study Session: 1 - RA: 7 338. Correct answer: D Standard error of the mean = Standard deviation / (Number of observations)^0.5 = 15 / 60^0.5 = 1.94. Study Session: 1 - RA: 7 339. Correct answer: C Coefficient of determination = ESS / TSS = 3.312 / 3.617 = 0.916. Study Session: 1 - RA: 7 340. Correct answer: D The standard error = USS / (N-2) = (TSS - ESS) / (N-2) = (245,142.857 - 227,088.816) / (72) = 3,610.8082. Study Session: 1 - RA: 7 341. Correct answer: D Standard error of the mean = Standard deviation / (Number of observations)^0.5 = 15 / 60^0.5 = 1.94. Study Session: 1 - RA: 7 342. Correct answer: D

http://64.106.152.240/inmarkets/passpro/index.cfm?event=questions....

2004-08-02

PASS PRO - Copyright 2004 Inmarkets Ltd - Licensed to Raymond chen

47/150 1

The standard error = USS / (N-2) = (TSS - ESS) / (N-2) = (3.617 - 3.312) / (7-2) = 0.061. Study Session: 1 - RA: 7 343. Correct answer: D The percentage of explained variation = Correlation coefficient ^ 2 = 0.33 ^ 2 = 0.1089 or 10.89%. Therefore, the unexplained variation is 89.11%. Study Session: 1 - RA: 7 344. Correct answer: A The slope coefficient = Sum_XY / Sum_X^2 = 9.08 / 24.9 = 0.365. Study Session: 1 - RA: 7 345. Correct answer: D First we calculate the slope coefficient = Sum_XY / Sum_X^2 = -9,152.23 / 368.857 = 24.812. Then we calculate the intercept = Avg_Y - Slope x Avg X = 627.143 - -24.812 x 14.857 = 995.77. Study Session: 1 - RA: 7 346. Correct answer: D The slope coefficient = Sum_XY / Sum_X^2 = 133.302 / 23.333 = 5.713. Study Session: 1 - RA: 7 347. Correct answer: B Firstly, we calculate the slope coefficient = Sum_XY / Sum_X^2 = 1,640 / 28 = 58.571. Then, we calculate the intercept = Avg_Y - Slope x Avg X = 455.714 - 58.571 x 4 = 221.43. Study Session: 1 - RA: 7 348. Correct answer: C The test to compare variances of two normally distributed populations is F-statistic. Study Session: 1 - RA: 7 349. Correct answer: D Correlation coefficient = Covariance / (Standard deviation of stock A x Standard deviation of stock B) = 0.1013 / (0.30 x 0.45) = 0.75. Study Session: 1 - RA: 8 350. Correct answer: C

http://64.106.152.240/inmarkets/passpro/index.cfm?event=questions....

2004-08-02

PASS PRO - Copyright 2004 Inmarkets Ltd - Licensed to Raymond chen

48/150 1

Correlation coefficient = Covariance / (Standard deviation of stock A x Standard deviation of market index) = -0.002 / (0.018^0.5 x 0.025^0.5) = -0.09. Study Session: 1 - RA: 8 351. Correct answer: A Covariance = Standard deviation of stock A x Standard deviation of stock B x Correlation coefficient = 0.30 x 0.45 x 0.75 =0.1013. Study Session: 1 - RA: 8 352. Correct answer: D Since the investments are perfectly negatively correlated the volatility of the portfolio = (w1 x v1 - w2 x v2). This can be reduced to zero by adjusting the amounts invested in the inverse of the ratios of the volatilities (i.e. w1/w2 = v2/v1) Study Session: 1 - RA: 8 353. Correct answer: D Since the investments are perfectly negatively correlated the volatility of the portfolio = (w1 x v1 - w2 x v2). This can be reduced to zero by adjusting the amounts invested in the inverse of the ratios of the volatilities (i.e. w1/w2 = v2/v1) Study Session: 1 - RA: 8 354. Correct answer: A Covariance = Standard deviation of stock A x Standard deviation of stock B x Correlation coefficient = 0.30 x 0.45 x 0.75 =0.1013. Study Session: 1 - RA: 8 355. Correct answer: A Since the investments are perfectly negatively correlated the volatility of the portfolio = (w1 x v1 - w2 x v2). This can be reduced to zero by adjusting the amounts invested in the inverse of the ratios of the volatilities (i.e. w1/w2 = v2/v1) Study Session: 1 - RA: 8 356. Correct answer: C The positive correlation only means that the prices will tend to move in the same direction, not that they always will. Study Session: 1 - RA: 8 357. Correct answer: A Since the investments are perfectly negatively correlated the volatility of the portfolio = (w1 x v1 - w2 x v2). This can be reduced to zero by adjusting the amounts invested in the inverse of the ratios of the volatilities (i.e. w1/w2 = v2/v1)

http://64.106.152.240/inmarkets/passpro/index.cfm?event=questions....

2004-08-02

PASS PRO - Copyright 2004 Inmarkets Ltd - Licensed to Raymond chen

49/150 1

Study Session: 1 - RA: 8 358. Correct answer: D Correlation coefficient = Covariance / (Standard deviation of stock A x Standard deviation of stock B) = 0.1013 / (0.30 x 0.45) = 0.75. Study Session: 1 - RA: 8 359. Correct answer: D Covariance of returns is an absolute measure of movement. The magnitude depends on size of the individual assets standard deviations and the relationship between co-movements. A positive covariance means that the asset returns move in the same direction, while a negative covariance means that asset returns move in the opposite direction. A zero covariance means that there is no relationship between asset returns. Study Session: 1 - RA: 8 360. Correct answer: B Only the Hull and White and the Heath, Jarrow and Morton are no-arbitrage models. Study Session: 1 - RA: 10 361. Correct answer: B Only the Hull and White and the Heath, Jarrow and Morton are no-arbitrage models. Study Session: 1 - RA: 10 362. Correct answer: B If X and Y are normally distributed, X + Y will also be normally distributed. Study Session: 1 - RA: 10 363. Correct answer: D Of the models given here, only Heath, Jarrow and Morton is a no-arbitrage model that incorporates mean reversion. Cox, Ingersoll and Ross model and Vasicek model incorporate mean reversion but are not a no-arbitrage model. Black-Scholes does not meet either criteria. Study Session: 1 - RA: 10 364. Correct answer: A Since X and Y follow geometric Brownian motion, it follows that log(X) and log(Y) are normally distributed, which means that log(X) + log(Y) is also normally distributed. log(X) + log(Y) is equal to log(X * Y), so X * Y must follow geometric Brownian motion. Study Session: 1 - RA: 10 365. Correct answer: C

http://64.106.152.240/inmarkets/passpro/index.cfm?event=questions....

2004-08-02

PASS PRO - Copyright 2004 Inmarkets Ltd - Licensed to Raymond chen

50/150 1

The Hull and White, the Ho and Lee, and the Heath, Jarrow and Morton are no-arbitrage models. Study Session: 1 - RA: 10 366. Correct answer: B In the model: dr = a * (b - r) * dt + s * dz, the interest rate r will mean revert to a level b. Study Session: 1 - RA: 10 367. Correct answer: C dr = a * (b - r) * dt + s * dz, is the only model given above that allows for the interest rate r to mean revert to a level b. Study Session: 1 - RA: 10 368. Correct answer: D Only Hull and White is a no-arbitrage model that incorporates mean reversion. Ho and Lee is a no-arbitrage model that does not incorporate mean reversion. Vasicek model incorporates mean reversion but is not a no-arbitrage model. Black-Scholes does not meet either criteria. Study Session: 1 - RA: 10 369. Correct answer: B Wiener process has a mean change of zero and variance proportional to the time interval. Generalized Wiener process and Ito process incorporate a trend variable a * dt. Study Session: 1 - RA: 10 370. Correct answer: B If X and Y are lognormally distributed, log(X) and log(Y) will be normally distributed, which means that log(X) + log(Y) = log(X * Y) is also normally distributed, implying that X * Y must be lognormally distributed. Study Session: 1 - RA: 10

PASS PRO , Expected Score , Adaptive Practice , Targeted Practice and Pick'N'Mix Test Creator are trademarks of Inmarkets Ltd.

http://64.106.152.240/inmarkets/passpro/index.cfm?event=questions....

2004-08-02

You might also like